Primary FRCA: 450 MTFs and SBAs Kariem El-Boghdadly, Imran Ahmad, Craig Bailey
INDEX
×
Chapter Notes

Save Clear


Mock paper 1: 60 MTFs and 30 SBAs to be answered in three hoursChapter 1

Questions: MTFs
Answer each stem ‘True’ or ‘False’.
1. Regarding the cardiac cycle, which of the following are true:
  1. At the end of atrial systole, the mitral valve closes because the pressure in the left atrium exceeds that in the left ventricle
  2. At the end of isovolumetric contraction, the aortic valve opens at approximately 120 mmHg
  3. Isovolumetric relaxation signifies the beginning of diastole
  4. The pressure/volume trace of the right ventricle has the same morphology as the left ventricle
  5. The dicrotic notch appears on the left ventricle pressure trace
1.
  1. False
  2. False
  3. True
  4. True
  5. False
The atria and ventricles contract in sequence, which results in a cycle of pressure and volume changes (see Figure 1.1a and b). During atrial systole, blood flows from the atria through the atrioventricular valves and into the ventricles. The volume of blood in a ventricle at the end of the filling phase is the end-diastolic volume (about 120 mL in adults). As soon as the left ventricular pressure rises above the left atrial pressure, the mitral valve closes (A). This is the beginning of isovolumetric contraction.
zoom view
Figure 1.1: (a) The pressure/time curve for the left ventricle. (b) The flow/time curve for the left ventricle.
During isovolumetric contraction the ventricle is a closed chamber, allowing a steep pressure rise, generated as a result of increasing ventricular wall tension. When the ventricular pressure exceeds aortic pressure, usually approximately 80 mmHg, then the aortic valve opens, isovolumetric contraction ends and ejection into the aorta begins (B).
At the end of the ejection phase, the aortic and pulmonary valves close and once again the ventricles become closed chambers; this is isovolumetric relaxation. As the pressure in the ventricles falls below atrial pressure, the atrioventricular valves open and blood then flows from the atria into the ventricles. Both mitral and aortic valves are closed during isovolumetric relaxation, and there is a rapid fall in pressure; this is the first stage of diastole (C).
Both the left and right ventricles have the same pressure/volume traces or morphology, but the pressures are lower in the right ventricle (D).
The dicrotic notch is seen on the aortic pressure trace and is as a result of the elastic recoil of the aortic walls after the aortic valve closes. It is not seen in the left ventricular pressure trace (E).
2. Coronary blood flow:
  1. Is approximately 500 mL/min
  2. In the left coronary artery is constant throughout the cardiac cycle
  3. In the right coronary artery is always greater than the left coronary artery
  4. In the right coronary artery occurs throughout the cardiac cycle
  5. In the left coronary artery peaks during systole
2.
  1. False
  2. False
  3. False
  4. True
  5. False
Coronary blood flow is approximately 200–250 mL/min in an average adult and comprises about 5% of the total cardiac output (A).
Flow in the right coronary artery is greater than that in the left during systole, but less during diastole. The blood flow within the left coronary artery varies throughout the cardiac cycle (B).
Flow through the right coronary artery occurs throughout the cycle as the transmitted intracavity pressures are low compared with the left ventricle, where the higher pressures lead to compression of the coronary vessels and left coronary blood flow almost ceases during systole (see Figure 1.2) (C, D, E).
Coronary blood flow is least during systole and mostly occurs during diastole.
The coronary perfusion pressure (CPP) determines the maximum pressure perfusing the coronary arteries and can be quantified by:
CPP = aortic-diastolic pressure – left ventricular end-diastolic pressure.
The above equation demonstrates that the greatest perfusion occurs during diastole.
zoom view
Figure 1.2: Left and right coronary arterial blood flow in different phases of the cardiac cycle.
3. The left ventricle:
  1. Has a wall that is three times thicker than that of the right ventricle
  2. Takes about 2 seconds to fill during ventricular diastole at rest
  3. Is responsible for the ‘apex beat’
  4. Ejects blood into the right coronary artery
  5. Contains papillary muscles connected to the aortic valves
3.
  1. True
  2. False
  3. True
  4. True
  5. False
The left ventricular wall is about three times thicker than the right, as it has to generate much higher pressures to provide blood flow in the systemic circulation (A). Ejection of blood is produced by a reduction in both diameter and length of the chamber.
At rest, ventricular diastole typically lasts for approximately 0.5 second, which is about two-thirds of the cardiac cycle (B).
When the chamber of the left ventricle contracts, it twists forward and the apex taps against the chest wall; this can be palpated in the left fifth intercostal space as the ‘apex beat’ (C).
Both the right and left coronary arteries arise from the aorta immediately above the cusps of the aortic valve to receive blood from the left ventricle (D).
The papillary muscles of the left ventricle are connected to the cusps of the mitral valve by chordae tendineae, not to the aortic valves (E).
4. Regarding veins, which of the following are true:
  1. They contain a third of the circulating blood volume
  2. Their pressure/volume curve is initially very steep
  3. Blood enters the venules at a lower pressure than larger veins
  4. Central venous pressure is the pressure at the point where the venae cavae enter the right atrium
  5. Standing results in a decrease in the venous pressure of veins above and below the level of the heart
4.
  1. False
  2. True
  3. False
  4. True
  5. False
Peripheral veins and venules are thin-walled, voluminous vessels that hold roughly two-thirds of the circulating volume (A). Venules have a diameter of 0.01–0.2 mm with veins having a diameter of 0.2–5.0 mm, together collectively being known as capacitance vessels. In contrast, arterioles and small arteries are known as resistance vessels as they are responsive to autonomic supply and can constrict and dilate to control blood flow.
The effect of pressure on venous volume is very steep between 0 and 10 mmHg, due to the easily distensible walls (B). This allows the volume to increase relatively easily per unit rise in pressure.
Blood enters the venules at a pressure of about 12–20 mmHg, but by the time it reaches larger veins, such as the axillary or femoral vein, the pressure falls to about 10 mmHg (C).
Central venous pressure is usually measured directly by a catheter placed in the superior vena cava, via the internal jugular or subclavian veins (D).
On standing the pressure increases in any blood vessel below the level of the heart and decreases above the heart; this is due to gravity acting on the column of fluid between the heart and the vessel (E).
25. Regarding physiological dead space, which of the following are true:
  1. It decreases under general anaesthesia
  2. It increases with hypovolaemia
  3. It is the same as anatomical dead space
  4. It requires knowledge of the Peco2 and Pao2 to be calculated
  5. Alveolar dead space is normally 150 mL
5.
  1. False
  2. True
  3. False
  4. False
  5. False
Physiological dead space increases under general anaesthesia and in hypovolaemic patients as shown in Table 1.1 (A, B).
Table 1.1   The causes of increased and decreased dead space
Anatomical dead space
Alveolar dead space
Increased
Decreased
Increased
Neck extension
Neck flexion
Pulmonary embolism
Jaw protrusion
Low tidal volumes
Pulmonary disease
Increased tidal volumes
General anaesthesia
Hypovolaemia
Neonates and the elderly
Intubation
Hypotension
Bronchodilation
Tracheostomy
General anaesthesia
Anticholinergics
5-hydroxytryptamine
Intermittent positive pressure ventilation
Catecholamines
Histamine
Positive end-expiratory pressure
Physiological dead space is the sum of anatomical dead space (normally 150 mL) and alveolar dead space (normally 0 mL); thus, it is not the same as the anatomical dead space (C, E). It is normally approximately 30% of tidal volume (i.e. Vd/Vt = 0.3) and is calculated by the Bohr equation (see Answer 3.61). The Bohr equation requires the knowledge of the arterial partial pressure of CO2 (Paco2) and the expired partial pressure of CO2 (Peco2) (D):
There are a number of factors that affect the anatomical dead space and alveolar dead space as classified in Table 1.1.
6. Regarding pulmonary blood flow, which of the following is true:
  1. It is greatest in the uppermost lung when a subject is in the lateral position
  2. Regional perfusion differences can be explained by the effects of hydrostatic pressure
  3. Measurement of pulmonary blood flow requires knowledge of arterial carbon dioxide content
  4. In West zone 1, alveolar pressure exceeds pulmonary arterial pressure
  5. Hypoxic areas of lung have increased perfusion
6.
  1. False
  2. True
  3. False
  4. True
  5. False
Pulmonary blood flow is greatest in the dependent areas of the lung. Thus, when standing, perfusion is greatest at the bases, and in the lateral position perfusion is greatest in the lower lung (A). This is due to the effect of hydrostatic pressure whereby gravity increases perfusion pressure in the lungs by 1 cmH2O for every centimetre in height below the level of the heart (B).
The Fick principle may be utilised in the measurement of pulmonary blood flow. This states that O2 consumption per unit time (V̇o2) is equal to the amount of O2 taken up by the blood in the lungs per unit time (i.e. the blood flow times the arterial oxygen content (Cao2) – venous oxygen content (CV̄o2) difference) (C).
Therefore:
The lung is divided into three zones defined by the relationship between pulmonary arterial (Pa), pulmonary venous (Pv) and alveolar pressures (Pa). In zone 1, Pa>Pa>Pv, thus capillaries are closed and no flow occurs (D). This does not occur normally but in areas where it does occur it is defined as an area of alveolar dead space. Zone 2 may be a region in the middle of the lung, where Pa>Pa>Pv and blood flow is determined by the arterial-alveolar pressure difference. Finally, zone 3 is often at the bottom of the lung, where Pa>Pv>Pa and blood flow is determined by the arterial-venous pressure difference.
Hypoxic areas of the lung undergo vasoconstriction (hypoxic pulmonary vasoconstriction, HPV) to prevent blood flow to poorly ventilated alveoli, thus reducing shunt (E).
7. The respiratory centre receives afferents from:
  1. Pulmonary stretch receptors
  2. Juxtaglomerular capillary receptors
  3. Dorsal medullary neurones
  4. Baroreceptors
  5. Carotid sinus receptors
7.
  1. True
  2. False
  3. False
  4. True
  5. False
The control of breathing involves sensors with afferent fibres to central controllers that then feed effector targets. The location and function of each of these areas can be summarised as follows:
  • Central controllers
    • The respiratory centre is responsible for the rhythmic inspiratory and expiratory patterns and includes:
      • The medullary respiratory centre
      • The apneustic centre in the lower pons
      • The pneumotaxic centre in the upper pons
  • Sensors
    • Central chemoreceptors
      • On the ventral medullary surface (C)
      • Stimulated by a reduction in CSF pH (i.e. an increase in H+ ion concentration) caused by metabolic acidosis or an increased Pco2
      • Not affected by Po2
    • Peripheral chemoreceptors
      • Aortic bodies giving vagal afferents and carotid bodies giving glossopharyngeal nerve afferents (E)
      • Stimulated by elevated Pco2 in a linear fashion, increase in H+ ions and a reduction in Po2 below 8–10 kPa
    • Lung receptors
      • Pulmonary stretch receptors (A)
      • Juxtapulmonary capillary (J) receptors (B)
      • Irritant receptors
      • Bronchial C fibres
    • Other receptors
      • Nose and upper airway receptors
      • Arterial baroreceptors (D)
      • Joint and muscle receptors
      • Higher centres: pain, temperature, anxiety
  • Effectors
    • Respiratory muscles
8. The following substances undergo metabolism in the lungs:
  1. Angiotensin II
  2. Acetylcholine
  3. Noradrenaline
  4. Leukotrienes
  5. Prostaglandin A2
8.
  1. False
  2. False
  3. True
  4. True
  5. False
Other than the heart, the lungs are the only other organ that receives the whole circulating blood volume. Therefore, the lungs are involved in a number of metabolic processes (see Table 1.2) including noradrenaline and leukotriene metabolism (C, D).
Table 1.2   Metabolic functions of the lungs
Removal/inactivation
No effect
Conversion
Bradykinin - 80%
Angiotensin II
Angiotensin I to II by angiotensin-converting enzyme (ACE)
5-hydroxytryptamine (5-HT; serotonin)
Vasopressin (antidiuretic hormone, ADH)
Noradrenaline
Dopamine
Prostaglandins E2 and F
Histamine
Leukotrienes
Prostaglandin A2 (E)
Although angiotensin II is formed in the lungs by the action of angiotensin-converting enzyme (ACE) on angiotensin I, it is metabolised to angiotensin III in red blood cells and vascular endothelium (A). It is therefore not metabolised in the lungs.
Acetylcholine is hydrolysed in the plasma and post-synaptic membranes, but not directly in the lungs (B).
Other functions of the lung include phospholipid synthesis (e.g. surfactant), involvement in immune and coagulant function, and acting as a reservoir of blood (between 500 and 900 mL at any given moment). This latter feature explains the decreased blood pressure on release of an elevated intrathoracic pressure, as seen during the Valsalva manoeuvre (see Question 2.89).
9. The following are true regarding the kidneys:
  1. They receive approximately 25% of the cardiac output
  2. Each kidney has two renal arteries
  3. Most of the renal blood flow is to the medulla
  4. The vasa recta arise from the afferent arterioles
  5. The renal arteries divide into interlobular and arcuate arteries
9.
  1. True
  2. False
  3. False
  4. False
  5. True
Despite the kidneys being small (weighing approximately 130 g each), together they receive nearly a quarter of cardiac output, or 500 mL/min/100 g (about 1.2 L/min) (A).
Blood is supplied to the kidneys via the renal artery, which is a branch of the aorta. There are typically two renal veins and one renal artery per kidney (B).
The distribution of blood flow to different regions varies, with 90% of renal blood flow to the cortex and only 10% to the medulla (C). Despite this the inner medulla still receives a higher blood flow per gram of tissue than most other organs.
The vasa recta arise from the inner cortical efferent arterioles, which also supply the peritubular capillaries (D). The vasa recta provide the only blood supply to the renal medulla.
The high-pressure renal arteries first divide into several interlobar arteries and these in turn divide into arcuate arteries, which give off the interlobular arteries at right angles (see Figure 1.3) (E).
zoom view
Figure 1.3: Divisions of renal arterial supply.
10. Functions of the kidney include:
  1. Direct maintenance of intracellular fluid volume
  2. Maintenance of extracellular potassium concentration
  3. Gluconeogenesis
  4. Production and secretion of angiotensinogen
  5. Production of prostaglandins
10.
  1. False
  2. True
  3. True
  4. False
  5. True
The kidneys maintain the constancy of the extracellular fluid volume (ECFV) and the extracellular osmolality by balancing the intake and excretion of sodium and water (A). The osmolality of the ECFV is strictly controlled to avoid large fluctuations in volume and osmolality within the intracellular compartment, as this is in osmotic equilibrium with the ECFV. They do not directly maintain intracellular fluid volume.
The kidneys maintain the concentration of extracellular potassium constant as well as the pH of the blood, by adjusting the excretion of hydrogen ions and bicarbonate (B).
Gluconeogenesis is the process of formation of glucose from substrate, including lactate and pyruvate, occurring predominantly in the liver with a smaller contribution from the kidneys (C). Other metabolic functions of the kidneys include arginine formation and peptide hydrolysis.
The kidneys produce the proteolytic enzyme renin from the juxtaglomerular apparatus, which it releases into the bloodstream. Renin acts on angiotensinogen, which is produced in the liver, to form angiotensin I (D). This is then converted to the highly active angiotensin II by cleaving two amino acids. Angiotensin II causes thirst and stimulates the release of aldosterone.
In addition to the regulation of the ECFV and ion concentration, the kidneys are a source of hormones, such as prostaglandins, angiotensin II and erythropoietin (E).
11. Glomerular filtration rate:
  1. Can be calculated using inulin and creatinine as indicators
  2. Can be calculated using the formula urine concentration (U) × plasma concentration/urinary flow rate (V)
    3
  3. Is about 125 L/min
  4. Is the same as the clearance of inulin
  5. Ceases when the mean systemic arterial pressure falls below 60 mmHg
11.
  1. True
  2. False
  3. False
  4. True
  5. False
Both inulin and creatinine have the suitable properties required to measure glomerular filtration rate (GFR) (A). Inulin is a carbohydrate which is infused into the bloodstream, whereas creatinine is already present in the blood as a breakdown product of muscle metabolism.
The formula for calculating GFR is UV/P, where U is the urine concentration of the indicator, V is the urine flow rate and P is the plasma concentration of the indicator (B). The GFR is normally approximately 125 mL/min for a normal adult (C).
Inulin, once injected, is completely filtered to enter the tubule and all of it enters the urine; therefore, the rate of filtration of inulin will equal the rate of excretion (D).
GFR is constant at mean systemic arterial pressures above 80 mmHg, so the filtration pressure is kept constant in this range by altering the pre-arteriolar and arteriolar resistance. Filtration significantly reduces but does not cease below a mean pressure of 60 mmHg (E).
12. Regarding potassium balance, which of the following is true:
  1. Approximately 60% is excreted in the urine and 40% in the faeces
  2. Approximately 99% is stored intracellularly
  3. Extracellular potassium is important for the regulation of potassium balance
  4. Acute regulation is controlled by aldosterone
  5. Most of the potassium filtered by the kidney is reabsorbed
12.
  1. False
  2. True
  3. True
  4. False
  5. True
About 50–150 mmol of potassium (K+) is resorbed daily in which about 90% of the K+ is excreted in the urine and about 10% in the faeces (A).
About 98–99% of the total body K+ stores are intracellular, mostly in muscle cells, but also in the liver and the erythrocytes (B).
Extracellular K+ only accounts for about 1% of the total body store; one of the main functions of this extracellular store is to mediate the regulation of the whole K+ balance of the body (C).
The acute regulation of extracellular K+ is achieved largely by the release of insulin, which promotes the uptake of K+ from the extracellular compartment in to the intracellular compartment. Aldosterone, adrenaline and alkalosis all promote the cellular uptake of K+, but are not as rapid in their management as insulin (D).
About 70% of K+ is reabsorbed by the end of the proximal convoluted tubule, about 20% at the loop of Henle and the rest is reabsorbed in the distal convoluted tubule (E).
13. Regarding cerebral blood flow (CBF), which of the following is true:
  1. The majority is in the white matter
  2. It is approximately 15% of cardiac output
  3. Two thirds are from the vertebral arteries
  4. Autoregulation maintains a constant CBF between a mean arterial pressure of 50 and 150 mmHg
  5. It is calculated using Fick's law
13.
  1. False
  2. True
  3. False
  4. True
  5. False
Cerebral blood flow (CBF) is approximately 15% of cardiac output, 700 mL/min (50 mL/100 g/min) of which the grey matter receives the majority (A, B). The blood supply is two thirds from the internal carotid arteries and one third from the two vertebral arteries (C). The two systems merge at the anterior and posterior communicating arteries to form the circle of Willis. CBF remains constant between mean arterial pressures (MAPs) of 50–150 mmHg – above this range there is a sharp increase in CBF, with a decrease in CBF below this range (D).
CBF can be measured using the Fick principle. This states that the uptake/release of a substance, e.g. O2 (V̇o2), by an organ is the product of the blood flow (Q̇) through that organ and the arteriovenous difference in content (Cao2–CV̄o2) (see Question 1.6). This is applied using the Kety-Schmidt technique where 10% N2O is inhaled for 10–15 minutes, and the jugular venous concentration is measured and assumed to be the same as the brain concentration. Fick's law, however, describes the rate of diffusion across a membrane being proportional to the concentration gradient (E).
14. Regarding cerebrospinal fluid (CSF), which of the following is true:
  1. 500 mL is produced every 24 hours
  2. At greater intracranial pressures, more CSF is produced
  3. Fluid passes from the 3rd ventricle via the foramen of Magendie
  4. Potassium concentrations are normally 2.5–3.5 mmol/L
  5. There is a lower concentration of protein in CSF than in the plasma
14.
  1. True
  2. False
  3. False
  4. True
  5. True
Cerebral spinal fluid (CSF) is a clear, colourless fluid surrounding the brain that protects it from traumatic damage and helps regulate intracranial pressure (ICP).
It has characteristics and properties as follows:
  • Production
    • Total volume is 100–150 mL (10% of intracranial volume), produced at a rate of 0.3 mL/min by choroid plexuses in lateral, third and fourth ventricles
    • 500 mL is produced a day (A)
    • Formed by plasma filtration and secretion
    • Production of CSF is independent of ICP (B)
  • Circulation
    • Lateral ventricles to the third ventricle via foramen of Monro
    • Third to fourth ventricle via aqueduct of Sylvius
    • Fourth ventricle down the spinal cord or over cerebral hemispheres via midline foramen of Magendie or lateral foramen of Luschka (C)
    • Absorbed from dural venous sinuses via arachnoid villi
  • Composition
    • Normal CSF reference values in Table 1.3 (D)
    • CSF proteins are only 1% of plasma levels, while calcium and glucose levels are 50–60% of plasma levels (E)
    • Chloride and magnesium levels, however, are greater in the CSF than the plasma
Table 1.3   Reference values for cerebrospinal fluid
Constituent
Concentration
Sodium
135–145 mmol/L
Potassium
2.5–3.5 mmol/L
Chloride
115–125 mmol/L
Calcium
1–1.5 mmol/L
Magnesium
1.2–1.5 mmol/L
Glucose
2.7–4.2 mmol/L
Urea
1.5–6.0 mmol/L
Lymphocytes
0–5 × 106/L
Protein
0.2–0.4 g/L
15. Parasympathetic stimulation causes:
  1. Lacrimation
  2. Increased insulin secretion
  3. Vasoconstriction
  4. Tachycardia
  5. Bronchodilation
15.
  1. True
  2. True
  3. False
  4. False
  5. False
Parasympathetic stimulation is responsible for the ‘rest and digest’ actions in the autonomic nervous system, of which the Vagus nerve carries approximately three quarters of the fibres. Vasoconstriction, tachycardia and bronchodilation are all sympathetically mediated effects, while lacrimation and insulin production are parasympathetic effects.
The following are effects of parasympathetic stimulation:
  • Ophthalmic: pupillary and ciliary constriction; lacrimation (A)
  • Cardiovascular: bradycardia, reduced contractility; vasodilation in skeletal muscle, coronary, pulmonary, renal and viscera (C, D)
  • Pulmonary: bronchoconstriction; increased secretions (E)
  • Gastrointestinal: increased motility and secretions; sphincteric relaxation
  • Metabolic: increased secretion of insulin and glucagon (B)
  • Genitourinary: detrusor muscle contraction, sphincteric relaxation; penile erection
16. Regarding muscle spindles, which of the of following are true:
  1. They are involved in maintenance of posture
  2. Efferent control is via γ-motor neurones
  3. Sensory supply involves only type II fibres
  4. They sense muscle tension
  5. The withdrawal reflex is polysynaptic
16.
  1. True
  2. True
  3. False
  4. False
  5. True
Muscle spindles are formed from intrafusal muscle fibres that respond to changes in length, rather than muscle tension, whereby contraction of muscles causes shortening of muscle spindles, an important mechanism in maintaining posture (A, D).
When muscle spindles are passively stretched, they transmit impulses directly to efferent γ-motor neurones via either type Ia or type II fibres (B, C). This is the monosynaptic stretch reflex. However, γ-motor neurone activity can be modulated by descending spinal cord pathways, which may alter the resting tone and sensitivity of muscles. These features make muscle spindles suitable for sensing and modifying posture. Golgi tendon organs are responsible for sensing muscle tension.
The withdrawal reflex is a polysynaptic reflex, as there are interneurones between sensory signals and motor elements of the reflex (E).
17. Regarding daily nutrition, which of the following are true:
  1. Energy requirements are mainly obtained from fats, minerals and vitamins
  2. The largest energy contribution as a percentage of intake is from dietary fats
  3. Per unit, proteins have the highest caloric value
  4. Essential amino acids are only synthesised in small quantities
  5. Fat-soluble vitamins include vitamins B, C, E and K
17.
  1. False
  2. False
  3. False
  4. True
  5. False
Energy requirements are primarily provided by proteins, fats and carbohydrates (A). Vitamins and minerals are required in small quantities but do not provide significant energy supplies.
The energy contribution of carbohydrates is about 60% of the total, while fats provide about 25–30% of the energy (B). The remaining energy is provided by proteins.
One gram of fat provides about 40 kJ of energy, while 1 g of protein provides about 17 kJ as does 1 g of carbohydrates (C).
Essential amino acids are those that cannot be synthesised by humans or only in very small amounts; therefore, they need to be part of a healthy diet (D). The essential amino acids include valine, leucine, isoleucine, threonine, methionine, phenylalanine, tryptophan and lysine.
Fats are largely superfluous in the diet, provided that there is a supply of the essential fatty acids, such as linoleic acid, and the fat-soluble vitamins, such as A, D, E and K (E).
418. Swallowing:
  1. Is a voluntary process
  2. Involves the soft palate being pulled up
  3. Causes the larynx to be pulled up by the pharyngeal muscles
  4. Takes about 3 seconds from start to finish
  5. Is initiated at the tonsillar pillars
18.
  1. False
  2. True
  3. False
  4. False
  5. True
Swallowing is an involuntary reflex (A). It is preceded by the voluntary act of collecting food on the tongue and propelling the bolus into the pharynx by raising the tongue.
As the bolus of food enters the pharynx, it stimulates the swallowing receptors. This triggers the swallowing reflex, which is a series of autonomic pharyngeal muscular contractions.
First, the soft palate is pulled upwards to close the posterior nares, thus preventing food from entering the nasal cavities (B).
The palatopharyngeal folds are pulled medially so that well-masticated food passes posteriorly into the pharynx, and larger objects are impeded.
The vocal cords are strongly approximated, the larynx is pulled upwards and anteriorly by the neck muscles, not the pharyngeal muscles, and the epiglottis swings backwards over the opening of the larynx (C). This all prevents food from entering the larynx, while enlarging the opening of the oesophagus.
At the same time the upper oesophageal sphincter relaxes, allowing food to easily pass into the upper oesophagus.
The whole process, which includes the trachea closing, the oesophagus opening and propelling of the food bolus from the pharynx to the oesophagus, takes about 1–2 seconds (D).
The most sensitive tactile areas of the pharynx for the initiation of swallowing lie around the pharyngeal opening, with the tonsillar pillars having the greatest sensitivity (E).
19. Regarding hormone production, which of the following are true:
  1. The kidneys produce calcitriol
  2. The thyroid gland produces calcitonin
  3. The stomach produces glucagon
  4. The liver produces cholecystokinin
  5. The anterior pituitary gland produces oxytocin
19.
  1. True
  2. True
  3. False
  4. False
  5. False
A hormone is a regulatory substance released by a single or group of cells or organ that has an effect at a distant site. Common hormones can be seen in Table 1.4.
Calcitriol is the active form of vitamin D that acts to increase plasma calcium levels and is produced in cells of the proximal tubule of the renal nephron (A).
Calcitonin is generated by the thyroid parafollicular (C) cells to reduce serum calcium concentrations (B).
Glucagon is a peptide hormone synthesised in the α-cells of the pancreatic islets of Langerhans that increase blood glucose concentrations (C).
Cholecystokinin is a peptide hormone that is produced in the small intestinal and duodenal mucosa in response to fat and protein in chime (D). It increases gastric transit time to allow further digestion of fats.
Both oxytocin and vasopressin (antidiuretic hormone, ADH) are released from the posterior pituitary but are produced in the hypothalamus (E).
Table 1.4   Sites of hormone production
Organ
Hormone
Adrenal cortex
Aldosterone, cortisol, androgens
Kidneys
Calcitriol
Testes
Testosterone
Ovaries
Oestrogen, progesterone
Thyroid
T3 (triiodothyronine), T4 (thyroxine), calcitonin
Adrenal medulla
Adrenaline, noradrenaline
Mast cells
Histamine
Pancreas
Insulin, glucagon
Parathyroids
Parathyroid hormone
Stomach and small intestine
Gastrin, secretin, cholecystokinin, gastric inhibitory peptide
Anterior pituitary
Human growth hormone, prolactin, adrenocorticotropic hormone, melanocyte stimulating hormone, thryoid stimulating hormone, follicle stimulating hormone, luteinising hormone
Hypothalamus
Oxytocin, antidiuretic hormone
20. Regarding haemoglobin, which of the following are true:
  1. Adult haemoglobin is composed of two α- and two γ-subunits
  2. It contains iron in its ferrous state
  3. It contains four iron atoms
  4. It can bind eight molecules of oxygen to each haemoglobin chain
  5. Cooperative binding is due to the Bohr effect
20.
  1. False
  2. True
  3. True
  4. False
  5. False
Haemoglobin is a molecule that is designed to increase the oxygen carrying capacity of blood >50 times, and is made of four polypeptide globin chains. The adult circulation contains HbA (two α-chains and two β-chains) as well as 2–3% HbA2 (two α-chains and two δ-chains) (A). Fetal haemoglobin contains two α-chains and two γ chains.
Each globin chain has a porphyrin-derived haem group with a central iron atom in its ferrous state (Fe2+) able to carry one O2 molecule; therefore, up to four molecules of O2 can be carried per haemoglobin (B, C, D). It is methaemoglobin that carries iron atoms in the ferric state (Fe3+). Haemoglobin has a number of characteristic effects that it demonstrates:
  • Cooperativity: binding of one O2 molecule breaks non-covalent salt links between globin chains that increase affinity for further O2 molecules to bind. This effect is more pronounced with the second and third molecule of O2 that bind and is the underlying cause of the sigmoid-shaped oxyhaemoglobin dissociation curve
  • Bohr effect: higher Pco2 reduces the affinity of haemoglobin for O2 (E)
  • Haldane effect: increases affinity of deoxygenated haemoglobin for CO2
21. Bioavailability:
  1. Is the fraction of administered substance that enters the systemic circulation
  2. Is higher for orally administered drugs than rectally administered drugs
  3. May be reduced by increasing first-pass metabolism
  4. Increases in the presence of hepatic enzyme inducers
  5. Is 100% for intravenous ketamine
21.
  1. True
  2. False
  3. True
  4. False
  5. True
Bioavailability is the fraction of a drug dose that reaches the systemic circulation compared with a standard, usually intravenous, route of administration (A). Thus, any drug administered via the intravenous route has a bioavailability of 100% (E). By plotting a plasma concentration over time curve for the same dose of oral and intravenous drug administration (see Figure 1.4), the bioavailability can be calculated by:
where AUC is area under the curve and IV is intravenous.
zoom view
Figure 1.4: Bioavailability of a drug from oral and intravenous administration.
Bioavailability can be affected by a number of factors, including:
  • First-pass metabolism: drugs absorbed from the gastrointestinal tract enter the portal vein and thence to the liver, where they undergo metabolism prior to reaching the systemic circulation. This is minimised with sublingual and rectal routes of administration which have a higher bioavailability than the oral route (B, C)
  • Hepatic enzymes: induction of hepatic enzymes increases first-pass metabolism thereby reducing bioavailability, while hepatic enzyme inhibition reduces metabolism, thus increasing bioavailability (D)
  • Physicochemical properties: small or liquid drugs are more readily absorbed when compared with large drug preparations. Interaction with other orally delivered substances may reduce absorption, thus reducing oral bioavailability
  • Gastrointestinal properties: any cause for delayed gastric emptying or malabsorption may reduce oral bioavailability
22. The following drugs can be administered transdermally:
  1. Procaine
  2. Glyceryl trinitrate
  3. Clonidine
  4. Alfentanil
  5. Diclofenac
22.
  1. False
  2. True
  3. True
  4. False
  5. True
Transdermal drug delivery depends on drug factors and patient factors as shown in Table 1.5.
Table 1.5   Factors affecting transdermal drug delivery
Drug factors
Patient factors
Lipophilic and hydrophilic
Good regional blood flow
Short half-life
Intact skin
Low melting point
Low molecular weight (<500 Da)
High potency
Unionised
Prilocaine, not procaine, is a component of the topical local anaesthetic cream EMLA (eutectic mixture of local anaesthetic) and can be delivered transdermally (A).
Glyceryl trinitrate is a vasodilator that can be administered as transdermal patches (B).
Clonidine is a versatile α2-adrenoceptor agonist that can be administered orally, transdermally, intramuscularly, intravenously or intrathecally (C).
Alfentanil is a synthetic piperidine opioid receptor agonist that is only delivered via the intravenous route (D).
Diclofenac is a non-steroidal anti-inflammatory drug that can be administered orally, intravenously, rectally or transdermally (E).
Commonly used transdermally applied drugs include:
  • Fentanyl
  • Buprenorphine
  • Diclofenac
  • EMLA (2.5% lidocaine, 2.5% prilocaine)
  • Amethocaine
23. The volume of distribution:
  1. Is greater for fentanyl than morphine
  2. Of propofol is greater than total body water
  3. Is greater than 1 L/kg for most non-depolarising neuromuscular blocking drugs
  4. Is higher for highly protein-bound drugs
  5. Is the apparent volume that a drug disperses into to produce observed plasma concentrations
23.
  1. True
  2. True
  3. False
  4. True
  5. True
The volume of distribution (Vd) is the theoretical volume in which a drug would have to disperse in order to achieve observed plasma concentrations (E). Because it is only a theoretical volume, it may potentially be larger than total body water. Highly protein bound drugs also have a high volume of distribution, while highly polar or charged drugs do not cross membranes easily and stay within the central compartment, leading to small volumes of distribution (D).
The Vd of fentanyl is 4.0 L/kg, while that of morphine is 3.5 L/kg, reflecting its lipid solubility that is 600 times greater than that of morphine (A).
As propofol is highly lipid soluble, it has a volume of distribution of 4 L/kg, which is greater than total body water (B).
Non-depolarising neuromuscular blocking drugs all have volumes of distribution of <0.30 L/kg because they are highly polar drugs (C).
24. The following drugs are excreted in the urine predominantly unchanged:
  1. Diclofenac
  2. Epinephrine
    5
  3. Lithium
  4. Ephedrine
  5. Digoxin
24.
  1. False
  2. False
  3. True
  4. True
  5. True
Diclofenac is significantly metabolised in the liver by phase I and II metabolism, while exogenous epinephrine (adrenaline) is metabolised in the liver by catechol-O-methyl transferase to metadrenaline and metnoradrenaline (A, B).
Lithium, ephedrine and digoxin are all renally excreted without undergoing a significant degree of metabolism (C, D, E).
The following drugs are excreted predominantly unchanged in the urine and can be remembered by the mnemonic ACED LMNOP G:
  • Aminoglycosides
  • Cephalosporins
  • Ephedrine
  • Digoxin
  • Lithium
  • Milrinone/Mannitol
  • Neostigmine
  • Oxytetracycline
  • Penicillins
Drugs that are excreted unchanged in the urine are likely to need dose adjustment in renal failure.
25. The ideal intravenous anaesthetic agent would have the following properties:
  1. High lipid solubility
  2. Lipid-soluble formulation
  3. Long half-life
  4. Analgesic at sub-anaesthetic concentrations
  5. Pre-prepared solution
25.
  1. True
  2. False
  3. False
  4. True
  5. True
The ideal intravenous anaesthetic agent would have the following properties:
  • High lipid solubility (A)
  • Water-soluble formulation (B)
  • Short half-life (C)
  • Analgesic properties at low doses (D)
  • Pre-prepared solution (E)
  • Rapid recovery, with no accumulation after infusion
  • Minimal cardiovascular and respiratory depression
  • Antiemetic properties
  • Painless on injection
  • No interaction with other drugs
  • No histamine release
  • Long shelf life at room temperature
  • No histamine release
  • No hypersensitivity reactions
26. Thiopentone:
  1. Is formulated as a sulphur salt
  2. Forms a neutral solution when dissolved in water
  3. Is stored in nitrogen
  4. Once administered, 80% is immediately available at pH 7.4
  5. Is safe to use in patients with porphyria
26.
  1. False
  2. False
  3. True
  4. False
  5. True
Thiopentone is the sulphur analogue of the oxybarbiturate pentobarbitone. It is formulated as a sodium salt, sodium thiopentone and appears as a yellow powder in a glass vial (A).
Sodium thiopentone is a weak acid with a pKa value of 7.6. When dissolved in water, it forms an alkaline solution (B).
The free acid that is formed in solution is highly insoluble and would precipitate out of solution. To prevent the formation of the free acid, sodium thiopentone is stored in glass vials containing nitrogen, while sodium carbonate is added to react with water to produce hydroxide ions (C). This forms an alkaline solution, thus preventing the accumulation of hydrogen ions and therefore the undissociated acid.
At physiological pH, only 12% of the administered sodium thiopentone is actually available in the active form, which is non-protein bound and unionised (D). The rapid onset is due to the high lipid solubility of the drug and the fact that the brain receives a relatively large blood flow. Emergence after a single bolus dose is due to rapid redistribution into well-perfused tissues such as the liver followed by the skin and muscle.
Sodium thiopentone may precipitate an acute porphyric crisis and is absolutely contraindicated in patients suffering from porphyria (E).
27. Xenon:
  1. Is a colourless gas with a pungent odour
  2. Is produced by fractional distillation of air
  3. Increases cardiac output by sympathetic stimulation
  4. Has a slower onset and offset time than desflurane
  5. Has a higher density and viscosity than nitrous oxide
27.
  1. False
  2. True
  3. False
  4. False
  5. True
Xenon is a non-toxic inert odourless gas, making up only 0.0000087% of the atmosphere and is produced by the fractional distillation of air (A, B).
It does not alter myocardial contractility and may cause a small reduction in heart rate and hence cardiac output (C). Xenon does not sensitise the myocardium to catecholamines.
Xenon has a minimum alveolar concentration of 71% and has a very low blood:gas partition coefficient of 0.14, as a result its onset and offset of action are faster than desflurane, sevoflurane and nitrous oxide (D).
Xenon is three times denser and nearly twice as viscous as nitrous oxide, although the Fink effect of diffusion hypoxia is not a feature of xenon (E).
Xenon has a molecular weight of 131.2, a boiling point of -108°C, is non-flammable and does not support combustion. It undergoes virtually no metabolism in the body.
28. Tachyphylaxis:
  1. Is synonymous with tolerance
  2. Occurs when larger doses are required to achieve a similar response
  3. Is seen with amphetamine
  4. May be due to a change in receptor structure
  5. Takes place over a short period of time
28.
  1. False
  2. False
  3. True
  4. True
  5. True
Tolerance occurs when larger doses of a drug are required to achieve a given response, often due to altered receptor sensitivity such as in chronic opioid use (A).
Tachyphylaxis, however, is the reversible, acute decrease in response to a given dose of drug after repeated administration and is synonymous with desensitisation, which takes place over a longer period of time (B, E). The underlying mechanisms include:
  • Change in receptor structure (D)
  • Loss of receptors
  • Reduction in mediators
  • Increased breakdown of drug
  • Physiological adaptation
Examples include ephedrine and amphetamine depleting amine stores (C).
29. The following local anaesthetics are esters:
  1. Prilocaine
  2. Etidocaine
  3. Amethocaine
  4. Cocaine
  5. Ropivacaine
29.
  1. False
  2. False
  3. True
  4. True
  5. False
Local anaesthetics can be classified as esters or amides depending on the intermediate chains (see Figure 1.5). Esters contain the —CO.O— linkage and undergo hydrolysis in the plasma. They include amethocaine, cocaine and procaine (C, D). Amides contain the amide (—NH.CO—) linkage chain and undergo hepatic metabolism. All amides contain an ‘i’ followed by the suffix ‘caine’, including bupivacaine, etidocaine, lignocaine, prilocaine and ropivacaine (A, B, E).
zoom view
Figure 1.5: Chemical structure of ester and amide classes of local anaesthetic.
30. Systemic vascular absorption of local anaesthetic is greater than epidural administration when given:
  1. To the brachial plexus
  2. Caudally
  3. Subcutaneously
  4. To the femoral nerve
  5. Intercostally
30.
  1. False
  2. True
  3. False
  4. False
  5. True
Systemic absorption of local anaesthetics is greater than epidural administration when given via the caudal injection or intercostal injection (B, E). Systemic concentrations are lowest when administered subcutaneously, to plexuses or to large distal nerves (see Figure 1.6).
Systemic vascular absorption is dependent on:
  • Local anaesthetic characteristics
  • Use of vasoconstrictor
  • Location of injection
zoom view
Figure 1.6: Systemic absorption of local anaesthetics.
631. The following drugs have a vasodilator action:
  1. Prilocaine
  2. Bendroflumethiazide
  3. Diazoxide
  4. Tramadol
  5. Lidocaine
31.
  1. True
  2. True
  3. True
  4. False
  5. True
Most local anaesthetic agents, including prilocaine and lidocaine, cause vasodilatation at low concentrations with vasoconstriction at higher concentrations (A, E). Cocaine however is a pure vasoconstrictor. Bendroflumethiazide has an antihypertensive effect by inhibiting Na+/Cl- co-transport in the distal convoluted tubule, leading to a diuresis, but also exerts a direct vasodilator effect (B).
Diazoxide is structurally similar to bendroflumethiazide and acts as a vasodilator by modulating cyclic adenosine monophosphate levels in arteriolar smooth muscle cells (C).
Tramadol has no significant vascular effects when administered via the intravenous route (D).
32. Antiarrhythmic drugs:
  1. Are classified according to their site of action
  2. Are classified according to their effects on the action potential
  3. May belong to more than one class in the Vaughan Williams classification
  4. Belong to class I as they slow phase 0 of the action potential
  5. Such as adenosine belong to class III
32.
  1. True
  2. True
  3. True
  4. True
  5. False
Antiarrhythmic drugs can be categorised according to the cardiac tissue that they affect (see Table 1.6) (A). This may be useful when specific arrhythmias need to be treated.
The Vaughan Williams classification is based on the electrophysiological action on isolated cardiac fibres (B). There are four classes in this classification, more recently a 5th class has been added, as shown in Table 1.7.
Some drugs have more than one action and can belong to more than one class, but they all have a dominant effect that allows them to be classed accordingly (C).
Class I antiarrhthymic drugs have local anaesthetic properties that exhibit membrane stabilising activity and affect conduction, refractoriness and the action potential (D). They are further subdivided into Ia, Ib and Ic, but they all slow phase 0 of the action potential to different degrees.
Table 1.6   Site of action of antiarrhythmic agents
Site of action
Drugs
Sinuatrial node
Beta-blockers
Class IV drugs
Digoxin
Atrioventricular node
Class Ic and IV drugs
Beta-blockers
Digoxin
Adenosine
Ventricles
Class I and III drugs
Atria
Class Ia and Ic drugs
Beta-blockers
Class III drugs
Accessory pathways
Class Ia and III drugs
Table 1.7   The modified Vaughan Williams classification
Class
Mechanism of action
Drugs
Ia
Block Na+ channels, prolong refractory period
Disopyramide, procainamide, quinidine
Ib
Block Na+ channels, shorten refractory period
Lidocaine, mexiletine, phenytoin
Ic
Block Na+ channels, no effect on refractory period
Flecainide, propafenone
II
β-adrenoceptor blockade
Atenolol, propranolol
III
K+ channel blockade
Amiodarone, bretylium, sotalol
IV
Ca2+ channel blockade
Diltiazem, verapamil
V*
Miscellaneous
Adenosine (E), digoxin, magnesium sulphate
* Antiarrhythmic agents that do not fit into any of the original classes described by Vaughan Williams in 1970 were later classified into class V.
Class Ic has the most potent sodium channel blocking action; therefore, it leads to reduction in phase 0 rapid depolarisation. The weakest sodium channel effects, thus the least effect on phase 0, are from class Ib drugs such as lidocaine.
33. Regarding vasoactive drugs, which of the following is true:
  1. Dopamine is a synthetic analogue of dobutamine
  2. Dopamine is a precursor of noradrenaline
  3. Dobutamine can cause hypotension
  4. Dobutamine acts on dopaminergic receptors
  5. Dopamine causes peripheral dilatation at high doses
33.
  1. False
  2. True
  3. True
  4. False
  5. False
Dobutamine is a synthetic analogue of dopamine and is a β-adrenoceptor-stimulating agent (A). Dopamine is a catecholamine-like agent used for the treatment of severe heart failure and cardiogenic shock.
Dopamine is the precursor of noradrenaline and releases noradrenaline from stores in the nerve endings in the heart (B). The conversion of dopamine to noradrenaline occurs in granulated vesicles and is dependent on the enzyme dopamine β-hydroxylase.
Dobutamine is a derivative of isoprenaline with predominantly β1-adrenoceptor activity, thus making it a potent inotrope. It does, however, also have β2-stimulating effect, which causes peripheral vasodilatation. This may result in hypotension and a fall in the diastolic pressure with a reflex tachycardia (C).
Dobutamine acts on adrenergic receptors in the following order: β1 > β2 > α. It has no action on dopaminergic receptors (D).
Dopamine stimulates the heart by both β- and α-adrenergic responses and causes vasodilatation through its action on dopaminergic receptors. At high doses (>10 μg/kg/min), dopamine causes predominant α-adrenoceptor stimulation with peripheral vasoconstriction, increased peripheral vascular resistance and reduced renal blood flow (E).
34. The following are true regarding the mechanism of action of nitrates:
  1. They preferentially dilate large coronary arteries and arterioles
  2. They redistribute blood from epicardial to endocardial regions
  3. They have no effect on myocardial oxygen demand
  4. The vascular endothelium should be intact for effective vasodilation
  5. They are better arteriolar than venous dilators
34.
  1. True
  2. True
  3. False
  4. False
  5. False
In preference to small coronary arteries, nitrates dilate the large coronary arteries and arterioles with diameters >100 μm (A).
Nitrates redistribute blood flow along collateral channels and from epicardial to endocardial regions (B). They also relieve coronary spasm and dynamic stenosis, especially at epicardial sites, including the coronary arterial constriction induced by exercise.
Nitrates increase the venous capacitance, causing pooling of blood in the peripheral veins and thereby reducing venous return and ventricular volume. There is less mechanical stress on the myocardial wall and the myocardial oxygen demand is reduced. In addition to this, there is a fall in aortic systolic pressure, which further reduces myocardial oxygen demand (C).
The basic mechanism of nitrates (NO3-) is conversion to nitrites (NO2-) followed by enzyme-mediated release of unstable nitric oxide (NO). For some vasoactive agents an intact vascular endothelium is required, but nitrates vasodilate whether or not the endothelium is physically intact or functional (D). The NO produced then stimulates the enzyme guanylate cyclase to produce cyclic guanosine monophosphate (cyclic GMP). This reduces the levels of calcium in the vascular monocyte and therefore vasodilatation occurs. Nitrates are better venous than arteriolar dilators (E).
35. Suxamethonium:
  1. Is hydrolysed at the neuromuscular junction by cholinesterases
  2. Has active metabolites
  3. Is predominantly excreted in the urine
  4. Produces a phase I block on repeated administration
  5. Consists of two acetylcholine molecules joined by an ester link
35.
  1. False
  2. True
  3. False
  4. False
  5. False
Suxamethonium is a depolarising muscle relaxant. It is hydrolysed by plasma cholinesterase to choline and succinylmonocholine, leaving only 20% to reach the neuromuscular junction (A). Succinylmonocholine is a weakly active ester of succinic acid with choline (B). Because of the rapid metabolism of suxamethonium, only 10% is excreted via the kidneys in the urine (C).
It produces a ‘phase I’ block in which there is a train-of-four ratio of >0.7, no fade to a 1 Hz stimulus with no post-tetanic facilitation. Repeated administration may produce a ‘phase II’ block similar to that of non-depolarising muscle relaxants, demonstrating a reduced train-of-four ratio of <0.7, fade on 1 Hz stimulation and post-tetanic facilitation present (D).
Suxamethonium has a structure of two acetylcholine molecules joined through acetyl groups (E). By binding to nicotinic acetylcholine receptors, suxamethonium depolarises the membrane; however, due to the lack of plasma cholinesterase at the neuromuscular junction, it remains attached to the acetylcholine receptors thereby preventing any further activation, and thus causing neuromuscular blockade.
36. Bisquaternary aminosteroid drugs include:
  1. Pancuronium
  2. Vecuronium
  3. Mivacurium
  4. Tubocurarine
  5. Rocuronium
36.
  1. True
  2. False
  3. False
  4. False
  5. False
The aminosteroid neuromuscular blocking drugs are large, bulky, polar molecules containing a steroid ring. They can be monoquaternary (containing a single N+—CH3group) such as vecuronium and rocuronium, or bisquaternary (containing two N+—CH3 groups) such as pancuronium. Of note, pancuronium is the bisquaternary analogue of vecuronium thereby making it more potent (A, B). Mivacurium is a benzylisoquinolinium, along with atracurium and cis-atracurium, while tubocurarine is a monoquaternary alkaloid (C, D, E).
37. The following are true of non-steroidal anti-inflammatory drugs:
  1. Paracetamol has anti-inflammatory properties
  2. Aspirin has antipyretic properties
  3. Diclofenac is 10% plasma protein bound
  4. Paracetamol is 90% plasma protein bound
  5. Ibuprofen has greater anti-inflammatory activity than analgesic activity
37.
  1. True
  2. True
  3. False
  4. False
  5. False
Non-steroidal anti-inflammatory drugs (NSAIDs) are a group of diverse drugs with variable analgesic, antipyretic and anti-inflammatory properties. The extent and effect of common NSAIDs can be seen in Table 1.8.
Table 1.8   Features of non-steroidal anti-inflammatory drugs
Thus, paracetamol has moderate anti-inflammatory properties, while aspirin has significant antipyretic action (A, B). Most NSAIDs are highly protein bound except paracetamol that is only 10% protein bound (C, D). Ibuprofen is an equipotent analgesic and anti-inflammatory agent (E).
738. Morphine:
  1. Is a naturally occurring phenanthrene derivative
  2. Relaxes the gut sphincters and the sphincter of Oddi
  3. Causes nausea and vomiting via serotonergic (5HT) and muscarinic receptors
  4. Can precipitate chest wall rigidity
  5. Inhibits the release of adrenocorticotropic hormone and prolactin
38.
  1. True
  2. False
  3. False
  4. True
  5. True
Morphine is a naturally occurring opiate with a phenanthrene structure comprising three fused benzene rings (A). It causes constriction of the gut sphincters and spastic immobility of the bowel, resulting in constipation. It also causes contraction of the sphincter of Oddi, causing an increase in the intraluminal pressure within the biliary tree (B).
Morphine stimulates the chemoreceptor trigger zone via 5-HT3 and dopaminergic receptors. This explains the nausea-inducing effect of morphine (C). Muscarinic receptors are not activated by morphine to cause nausea and vomiting.
Morphine and other opioids can cause chest wall rigidity (D). This is thought to be as a result of dopaminergic and GABA pathways in the substantia nigra interacting with the opioid receptors.
Morphine inhibits the release of adrenocorticotropic hormone, prolactin and gonadotrophic hormones (E). It may also increase the secretion of vasopressin (antidiuretic hormone, ADH) that can cause water retention and hyponatraemia.
39. Benzodiazepines:
  1. Bind to the α-subunit of gamma-aminobutyric acid (GABA)A receptors
  2. Act via second-messenger systems
  3. Are highly plasma protein bound
  4. Always have active metabolites
  5. Include zopiclone
39.
  1. True
  2. False
  3. True
  4. False
  5. False
Benzodiazepines are hypnotic drugs that act on the α-subunit of GABAA receptors to open the ligand-gated chloride (Cl-) channel causing hyperpolarisation of post-synaptic neuronal membranes predominantly (A). They therefore do not act via second messenger systems (B). Baclofen, however, acts via metabotropic GABABreceptors, activating second messenger systems.
Benzodiazepines are generally highly protein bound and usually have active metabolites; however, lorazepam is glucuronated to inactive metabolites (C, D).
Zopiclone, along with zolpidem and zaleplon, is non-benzodiazepine ‘Z-drug’ that lacks the benzodiazepine chemical structure but binds to GABAA receptors in a similar fashion (E).
40. These diuretics have the following unwanted effects:
  1. Furosemide and hypercalcaemia
  2. Bendroflumethiazide and hypocalcaemia
  3. Spironolactone and Conn's syndrome
  4. Mannitol and an increase in preload and cardiac output
  5. Bendroflumethiazide and leucopaenia and thrombocytopaenia
40.
  1. False
  2. False
  3. False
  4. True
  5. True
The biochemical effects of furosemide include hyponatraemia, hypokalaemia, hypochloraemic alkalosis, hypomagnesaemia and hyperuricaemia. These effects are also caused by the thiazide diuretics, but furosemide can also cause hypocalcaemia by increasing Ca2+ excretion (A).
Bendroflumethiazide causes hypercalcaemia as a result of reduced renal excretion (B).
Spironolactone is a competitive aldosterone antagonist; therefore, it not only reduces the excretion of K+ but also increases the excretion of Na+ and water. As a result of its action, it is used to treat ascites, Conn's syndrome (primary hyperaldosteronism) and nephrotic syndrome (C).
Mannitol is administered as an intravenous infusion and is usually given as a loading dose followed by an infusion. The loading dose for the treatment of raised intracranial pressure is about 1 g/kg, followed by an infusion of between 0.1 and 0.2 g/kg. As a result of these large infusion volumes, the initial circulating volume is increased which increases the preload and cardiac output (D).
The haematological effects of bendroflumethiazide include haemolytic anaemia, aplastic anaemia, leucopaenia, thrombocytopaenia and agranulocytosis (see Table 4.8) (E).
41. The following is true regarding electricity:
  1. Electrons can only pass from one atom to another under a potential difference
  2. Body fluids are generally bad conductors
  3. Insulators bind to their electrons more firmly then semiconductors
  4. Thermistors are insulators
  5. Diodes are semiconductors
41.
  1. False
  2. False
  3. True
  4. False
  5. True
Electricity is the flow of electrons from one atom to another under the influence of a potential difference or under the influence of a changing magnetic field (A).
The electrons in a conductor are loosely bound; therefore, they can readily move when influenced by a potential difference. Bodily fluids are good conductors as they contain positive and negative ions, and these can move when a potential difference is applied to them, thereby conducting electricity (B).
The electrons in insulators are firmly bound to their atoms; therefore, they do not conduct electricity; thermistors are semiconductors (C, D).
The outer electrons in semiconductors are more firmly bound to their atoms than in conductors, but less firmly bound than in insulators. Diodes and transistors are semiconductors (E).
42. Surgical diathermy:
  1. Typically uses frequencies of 1 kHz
  2. Is more likely to cause ventricular fibrillation than a direct current
  3. Relies on two connections to the patient, even with monopolar diathermy
  4. Contains a capacitor
  5. When bipolar, uses higher power than monopolar
42.
  1. False
  2. False
  3. True
  4. True
  5. False
Surgical diathermy typically uses high-frequency currents in the region of 1 MHz (A).
The passage of lower frequency currents through the body can cause muscular contractions and ventricular fibrillation. The lower frequency of alternating current or the use of direct current is more likely to cause ventricular fibrillation than using high-frequency diathermy currents (B).
When using monopolar diathermy two connections are made to the patient: one is the patient plate (or neutral plate) and the other is the active (or cutting) electrode. When using bipolar diathermy, two connections are also made, but this is through the two ends of the forceps; a diathermy plate is not required (C).
Isolating capacitors are placed within the diathermy apparatus to increase safety of the equipment and reduce the risk of accidental burns to the patient in the event of the diathermy pad becoming detached (D).
Bipolar diathermy uses lower power than monopolar diathermy (E).
43. Regarding the hazards of magnetic resonance imaging (MRI), which of the following is true:
  1. MRI involves the use of ionising radiation
  2. There is a risk of causing involuntary muscle contraction
  3. Patients may experience flashing lights and sensations of taste
  4. Patients with metal prosthetic joints should not undergo MRI
  5. Ferromagnetic objects are safe to use within the fringe field of the scanner
43.
  1. False
  2. True
  3. True
  4. False
  5. False
MRI does not involve the use of ionising radiation (A). There is little evidence of serious adverse effects as a result of the magnetic field. Various recommendations exist, but generally continuous exposure should be limited to a magnetic flux density of 0.2 T and short exposures to 5 T.
If the magnetic field gradient is switched, then eddy currents can be induced and this has the potential of inducing small currents in any biological conductors, especially nerve fibres, which can cause involuntary muscle contraction (B). There is also a small risk of causing breathing difficulties and inducing ventricular fibrillation as a result of these eddy currents.
In the presence of very strong fields, patients may experience flashing lights and various taste sensations (C).
Joint and dental prostheses are safe as they are usually non-ferromagnetic and are firmly fixed in the patient, thus cannot be dislodged by the magnetic field of the scanner (D). They may however distort MRI images.
Ferromagnetic objects are attracted to the scanner even in the fringe field, which can extend for a few metres (E).
44. Regarding the gas laws, which of the following is true:
  1. Boyle's law states that at a constant temperature, the volume of a given mass of gas varies inversely to the absolute pressure
  2. Gay Lussac's law is the third perfect gas law
    8
  3. The second gas law states that at a constant pressure, the volume of a gas is proportional to the absolute temperature
  4. They are corrected to a standard temperature of 273°F
  5. Dalton's law states that PV/T = constant
44.
  1. True
  2. False
  3. True
  4. False
  5. False
Boyle's law is the first perfect gas law. If, at a constant temperature, the volume (V) of a gas is decreased, then the pressure (P) will increase by the same proportion (A). This principle can be used to calculate the volume of gas remaining in a cylinder when the pressure on the pressure gauge is known.
The second perfect gas law is Charles's law or Gay Lussac's law (B). This states that at a constant pressure the volume of a given mass of gas varies directly with the absolute temperature (T) (C). The third perfect gas law states that at a constant volume the absolute pressure of a given mass of gas varies directly with the absolute temperature. Combining the three perfect gas laws gives us the formula:
The perfect gas laws are usually corrected to a standard temperature of 273.15 K (0°C) and a standard pressure of 101 kPa (760 mmHg) (D). This is because gases are affected by temperature and pressure.
Dalton's law of partial pressures is not one of the perfect gas laws but refers to a mixture of gasses within a container (E). It states that in a mixture of gases the pressure exerted by each gas is the same as that which it would exert if it alone were occupying the container.
45. The following statements about simple mechanics are true:
  1. Power is defined as the work done when a force is applied in the direction of the force
  2. Power is measured in joules
  3. When the point of application of a force moves in the direction of the force energy is expended
  4. The power of breathing can be calculated by the area under the pressure/volume curve divided by time
  5. In respiratory muscles, most of the chemical energy is converted to mechanical energy
45.
  1. False
  2. False
  3. True
  4. True
  5. False
Power is the rate of work, or the amount of work performed over time (A). It is measured in watts (W) with one watt equalling one joule (J) per second(s): 1 W = 1 J/s (B).
Energy is always expended and work is done when the point of application of a force moves in the direction of the force (C). It is measured in joules, the energy expended when applying a force of one newton (N) through a distance of one metre (m).
The area under a pressure/volume curve for ventilation is the work done. If power is the rate of work done, then the power of breathing can be calculated by dividing the work done (the area under the curve) by the time (D).
There are various energy changes that take place during the respiratory cycle. It must be remembered that energy cannot be created or lost, but it is converted from one form to another. So, most of the chemical energy generated by the respiratory muscles is converted to heat energy, with only a small proportion converted to mechanical energy (E).
46. Regarding humidity, which of the following is true:
  1. Relative humidity is expressed in milligrams of water per liter of gas
  2. Humidity may be expressed as the pressure exerted by water vapour in a gas mixture
  3. As humidity increases, hair length shortens
  4. If a fully saturated gas is cooled, both the absolute and the relative humidity will fall
  5. A wet and dry bulb hygrometer measures ambient and relative humidity
46.
  1. False
  2. True
  3. False
  4. False
  5. False
Humidity is the amount of water vapour present in gas, and it may be expressed in many ways. Absolute humidity is the mass of water vapour present in a volume of gas and is expressed in milligrams of water per litre of gas. Relative humidity is the amount of water vapour at a particular temperature, and this is expressed as a percentage of the amount that would be held if the gas were fully saturated (A).
Humidity may also be expressed as the pressure exerted by water vapour in a gas mixture (B).
If gas saturated with water vapour is cooled, then it will condense out water vapour. The amount of condensed water vapour will be the amount held at the original temperature minus the amount it can hold at the new lower temperature. The absolute humidity will therefore fall, but the relative humidity remains at 100% (D).
The hair hygrometer is one method used to measure relative humidity, and it operates on the principle that hair length increases if the humidity increases (C).
The wet and dry bulb hygrometer allows the measurement of the ambient humidity, while relative humidity is obtained from a set of tables and is not directly measured (E).
47. Regarding heat and temperature, which of the following is true:
  1. Heat is a measure of the tendency of an object to gain or lose heat
  2. The SI unit for temperature is Celsius
  3. Temperature is the energy which can be transferred from a hotter object to a cooler one
  4. Zero Kelvin is absolute zero
  5. The triple point of water is at 273.16 K
47.
  1. False
  2. False
  3. False
  4. True
  5. True
Heat is the energy that can be transferred from a hotter object to a cooler object down this temperature gradient (A).
The SI unit for temperature is the Kelvin (K) and not Celsius (B).
Temperature is a measure of the tendency of an object to gain or lose heat (C).
Absolute zero is the zero reference point on this scale and 0 K is equivalent to -273°C (D).
The triple point of water is the temperature at which it exists as a solid, liquid and gas simultaneously. This is at 273.16 K or 0.01°C (E).
48. Regarding latent heat in anaesthesia:
  1. Liquid ethyl chloride vapourises from the skin causing a cooling sensation
  2. Ethyl chloride is stored as a liquid in a glass ampoule under pressure
  3. Cooling of volatile anaesthetics makes them less volatile
  4. Carbon dioxide is stored in cylinders in gaseous form
  5. As nitrous oxide is emptied from a cylinder, the pressure remains constant until all the liquid has vapourised
48.
  1. True
  2. True
  3. True
  4. False
  5. False
Latent heat is the heat absorbed or released when a substance changes its state at a constant temperature. As liquid ethyl chloride vapourises from the skin surface, it requires the latent heat of vapourisation to do so, which is gained from the skin. This results in cooling of the skin as heat is taken away from its surface (A).
Within glass ampoules ethyl chloride is stored under pressure as a liquid to ensure it does not vapourise until it has left the ampoule and contacts skin (B).
When volatile anaesthetics are vapourised, they lose latent heat and the vapourised liquid takes heat from the remaining fluid as well as the vapouriser. This can cause a fall in temperature of the remaining volatile anaesthetic, which makes them less volatile (C).
Nitrous oxide, carbon dioxide and oxygen may all be stored in liquid form when in cylinders, oxygen being a liquid in the vacuum-insulated evaporator (D).
As nitrous oxide is emptied from a cylinder, the liquid form is converted to gas which requires latent heat of vapourisation. This is taken from the remaining liquid and cylinder walls, which cools and may cause the water vapour in the air to condense outside the cylinder. Because of this cooling, there is a rapid fall in vapour pressure inside the cylinder, which is indicated on the pressure gauge (E).
49. The following are true regarding fluids and flow:
  1. Liquids are the only fluids
  2. During laminar flow, the fluid nearest to the wall has no flow
  3. In the middle of a pipe, the velocity is twice than that of the average velocity across the pipe, during laminar flow
  4. Turbulent flow requires less driving pressures than laminar flow
  5. Turbulent flow wastes more energy than laminar flow
49.
  1. False
  2. True
  3. True
  4. False
  5. True
Fluids are any form of matter that change shape under shear forces and can either be liquids or gases (A).
The flow of fluids can be described either as laminar or turbulent. Laminar flow is flow in layers, or laminae, of fluid running parallel to each other. In a pipe, the layers nearest to the wall will have the lowest velocity, and the layer closest to the wall will probably have no flow (B). The velocity profile across the pipe during laminar flow is parabolic, with the highest flow in the centre. The peak velocity in the centre is twice the average across the pipe at equilibrium (C). There is minimum energy loss during laminar flow.
During turbulent flow, eddies, vortices and currents are developed which result in energy loss in the form of heat, friction and noise (E). The driving pressure required for turbulent flow is greater than that required for an equivalent laminar flow rate (D).
950. Regarding the principles of ultrasound, which of the following is true:
  1. It is a form of mechanical energy
  2. It is produced from piezoelectric crystals which are compressed and decompressed
  3. The frequency is dependent on the compression pressures
  4. The wavelength is the reciprocal of the frequency
  5. The wave can pass through a vacuum
50.
  1. True
  2. True
  3. False
  4. True
  5. False
Ultrasound is mechanical energy in the form of high-frequency vibrations (A).
It is generated by electrically deforming a piezoelectric crystal, which causes it to compress and decompress the medium to which the crystal is coupled (B). The wavelength is the distance occupied by one oscillation when seen graphically.
The changes in pressure created by the crystal travel through the medium; the distance between the points of maximum pressure is also the wavelength. The frequency is not dependent on the compression pressures; in fact it is the wavelength that is dependent on the compression pressures (C). The wavelength of ultrasound is generally the reciprocal of the frequency (D). The wave is propagated by the movement of particles; therefore, it is unable to travel through a vacuum (E).
51. The following are true regarding hazards of electrical equipment:
  1. Under single fault conditions, type 1 CF equipment should have a leakage current of the order of 4 mA
  2. Type BF equipment is safe because the patient circuit is earthed
  3. Class II equipment has power cables only containing ‘live’ and ‘neutral’ conductors
  4. Class III equipment is defined as that which operates at ‘safety extra low voltage’ of less than 12 V
  5. A current-operated earth-leakage circuit breaker relies on an unacceptable current causing disintegration of a fuse that then breaks the circuit
51.
  1. False
  2. False
  3. True
  4. False
  5. False
Under single-fault conditions, type 1 CF equipment should have leakage currents under 50 μA (A).
Type BF equipment is floating, and may be either class I, II or III, thus is not, by definition, earthed (B).
Safety classification of equipment can be based on either:
  • The means of protection it provides
    • Class I – any accessible conducting part of the equipment is connected to an earth wire. Live, neutral and earth wires never come in contact with each other
    • Class II – also called ‘double-insulated’ as all accessible parts have two layers of insulation. Power cables only contain ‘live’ and ‘neutral’ conductors with a single fuse (C)
    • Class III – the equipment does not need electrical supply >50 V DC or 24 V AC; called ‘safety extra-low voltage’ (SELV) and still provides a theoretical risk of microshock (D)
  • The maximum leakage currents permissible
    • Type B – safe for external patient connection. Under single-fault conditions, equipment has leakage currents of 500 mA if class I or 100 mA if class II
    • Type BF – like type B, but the part connected to the patient is isolated (floating)
    • Type CF – safe for direct connection to the heart with isolated circuits.
      Leakage currents must be under 50 mA if class I or 10 mA if class II. Includes electrocardiogram leads and pressure transducers
Current-operated earth leakage circuit breakers (COELCB) consist of a live wire and a neutral wire attached to a relay circuit breaker. Faulty currents (e.g. high leakage current) lead to a magnetic field between the live and neutral wires that induce the relay to break the circuit. There is no fuse in COELCBs (E).
52. Nitrous oxide cylinders:
  1. Have blue and white quartered shoulders
  2. Are pressurised to 4.4 bar
  3. Have a filling ratio of 0.75 in the UK
  4. Have pressure within them decreasing linearly
  5. Of size E have a capacity of 1800 L
52.
  1. False
  2. False
  3. True
  4. False
  5. True
Nitrous oxide is stored in cylinders with French blue bodies and shoulders; Entonox is stored in cylinders with white and blue quartered shoulders (A). With a boiling point of -88°C and a critical temperature of 36.5°C, nitrous oxide exists as a liquid in cylinders with a vapour layer above.
It is stored at a pressure of 4400 kPa (44 bar), which decreases slowly until all the liquid is utilised with only vapour remaining at which point the pressure decreases rapidly (B). The slow decrease in cylinder pressures is due to the latent heat of vapourisation cooling the cylinder, thereby reducing the vapour pressure. Thus, cylinder pressures do not decrease in a linear fashion (D).
If nitrous oxide cylinders were full of liquid, then increasing temperature would risk the expansion of liquid, thus creating the risk of potential explosion. Therefore, the mass of gas in a cylinder divided by the mass of water required to fill that cylinder is termed the ‘filling ratio’, and in the UK the filling ratio is 0.75 (C).
Size E cylinders are attached directly to the anaesthetic machine and have a capacity of 1800 L, while size J cylinders are used in cylinder manifold and hold 18,000 L of nitrous oxide (E).
53. With regard to vacuum-insulated evaporators:
  1. They store liquid oxygen at a pressure of 400 kPa
  2. They have an internal temperature of -118°C
  3. The mass of oxygen can be measured by weight
  4. If less oxygen is utilised, the internal temperature rises
  5. If pressure exceeds 17 bar, this causes opening of a safety valve
53.
  1. False
  2. False
  3. True
  4. True
  5. True
A vacuum-insulated evaporator supplies oxygen to piped gas supplies in hospitals. It is composed of a double-insulated steel tank with a vacuum to maintain internal temperatures. Oxygen is stored as a liquid at -160°C which is below its critical temperature of -118°C, with an internal pressure of approximately 7 bar (700 kPa), not 400 kPa (A, B).
Up to 1500 L of liquid oxygen may be stored and the mass of the liquid can be measured either by weighing the vessel or by differential pressure gauges comparing pressures at the liquid bottom and vapour top of the cylinder (C).
When oxygen evaporates, the temperature of the vessel reduces due to the latent heat of vapourisation, further contributing to the maintenance of a low temperature. However, with reduced use of oxygen, the contribution of latent heat of vapourisation is reduced, therefore temperatures rise (D). With a rise in temperature, the internal pressure rises, and when this exceeds 17 bar, a safety valve opens allowing escape of vapourised oxygen (E). This leads to a further vapourisation, and thus a desired reduction in the temperature once again.
54. Flowmeters such as rotameters:
  1. Depend on gas density
  2. Rely on laminar flow at low flow rates
  3. Are accurate within 1%
  4. Reduce piped gas pressure
  5. Are inaccurate in the presence of static charge
54.
  1. True
  2. True
  3. False
  4. False
  5. True
Rotameter is the trade name of flowmeters that are constant-pressure, variable orifice devices composed of a conical tube that is wider at the top and a bobbin that varies in height depending on the flow rates. Opening of a needle valve at the bottom of the flowmeter allows gas into the tube, with higher flow rates allowing the bobbin to float higher within the tube. As the bobbin is of fixed dimensions, the distance between the bobbin and the wall of the tube increases as flow increases (see Figure 1.7). Thus, at low flow rates, the orifice is narrow and long allowing laminar flow depending on gas viscosity, while at higher flow rates the orifice is wider and flow is turbulent depending on gas density (A, B).
zoom view
Figure 1.7: Rotameter.
Flowmeters are 2.5% accurate but are only calibrated for the individual gases they are to be used for (C). Accuracy can be reduced by dirt within the flowmeter tubes, static electricity build-up (up to 35% inaccurate) and back pressure from the common gas outlet of up to 10% (E). Flowmeters have no effect on piped gas pressures (D).
55. The emergency oxygen flush:
  1. Delivers flow that bypasses the vapourisers
  2. Delivers at a pressure of 137 bar
  3. Has pressure-reducing valves to reduce barotrauma
  4. May lead to awareness
  5. Can deliver flow rates of 40–50 L/min
55.
  1. True
  2. False
  3. False
  4. True
  5. True
The emergency oxygen flush is designed to provide 100% oxygen flow that bypasses flowmeters and vapourisers at piped gas pressures of 4 bar (400 kPa) and flow rates of 35–75 L/min to the common gas outlet (A, B, E). Because it bypasses both flowmeters and vapourisers, it may therefore dilute anaesthetic gases potentially leading to awareness (D). It is a non-locking device that does not have any pressure reducing or regulating valves; therefore, the delivered pressure of 4 bar may potentially lead to barotrauma (C).
1056. The following are maximum allowable environmental concentrations in the UK:
  1. Desflurane: 75 ppm
  2. Enflurane: 50 ppm
  3. Halothane: 25 ppm
  4. Isoflurane: 100 ppm
  5. Nitrous oxide: 25 ppm
56.
  1. False
  2. True
  3. False
  4. False
  5. False
In 1996, the Health and Safety Commission issued maximally accepted 8-hour time-weighted average concentrations for the UK:
  • Enflurane: 50 particles per million (ppm) (B)
  • Halothane: 10 ppm (C)
  • Isoflurane: 50 ppm (D)
  • Nitrous oxide: 100 ppm (E)
These levels vary in different countries. No definite acceptable level was set for desflurane at the time of publication (A).
57. The Lack breathing system:
  1. Is a Mapleson D arrangement
  2. Is most efficient for controlled ventilation
  3. Can be in coaxial or parallel arrangements
  4. Requires a fresh gas flow rate of two to three times the minute ventilation during spontaneous ventilation
  5. Vents exhaled gases through a 14 mm inner tube
57.
  1. False
  2. False
  3. True
  4. False
  5. True
The Lack system is a coaxial version of the Mapleson A (Magill) breathing system that can also be in a parallel arrangement (A, C). It consists of a 14 mm inner tube attached to an adjustable pressure-limiting valve for exhaled gases and a 30 mm outer tube supplying the fresh gas flow (E).
It is efficient for spontaneously ventilating patients, requiring 70 mL/kg/min fresh gas flow, but is inefficient for controlled ventilation requiring a fresh gas flow rate of two to three times the minute ventilation (B, D). The most efficient Mapleson breathing system for controlled ventilation is the Mapleson D system, or Bain circuit.
58. The 12-lead electrocardiogram:
  1. Has a signal output of 1–2 mV
  2. Uses six bipolar leads
  3. Has a frequency response in monitoring mode of between 0 and 100 MHz
  4. Prints from an oscilloscope running at 25 mm/s
  5. CM5 arrangement optimally detects arrhythmias
58.
  1. True
  2. True
  3. False
  4. True
  5. False
The electrocardiogram is a device monitoring cardiac electrical activity using silver/silver chloride gel covered electrodes at specific sites on the skin. At the cardiac level, electrical potentials in the range of 90 mV are generated, but this signal is reduced as it passes through tissues, leaving a signal output of 1–2 mV (A). The initial signal is increased by a differential amplifier that may be set at a monitoring mode (frequency of 0.5–40 Hz) or diagnostic mode (frequency of 0.05–100 Hz) (C). The differential amplifier minimises noise and signal interference using the principles of common mode rejection, whereby interference that is the same between two leads is eliminated. The signal output is displayed via an oscilloscope or on paper running at 25 mm/s with a signal of 1 mV/cm (D).
There are six bipolar leads: I, II, III, aVR, aVL and aVF (note that ‘aV’ stands for ‘augmented voltage’) and there are six unipolar leads: V1–V6 (B). In anaesthetic practice, the configurations of three skin electrodes used include:
  • Lead II: right arm, left arm and indifferent electrodes; arrhythmia detection
  • CM5: right arm electrode on manubrium, left arm electrode on the 5th intercostal space, anterior axillary line and the indifferent electrode is on the left shoulder; sensitive for detecting ischaemia (E)
  • CB5: right arm electrode on right scapula, left arm electrode on the 5th intercostal space, anterior axillary line and the indifferent electrode is on the left shoulder; used in thoracic anaesthesia
59. Regarding statistical tests:
  1. A type I error involves rejecting a true null hypothesis
  2. β-errors are the same as false negatives
  3. The power of a study must be calculated after data collection
  4. A power of 20% is acceptable
  5. The variance is the square root of the standard deviation
59.
  1. True
  2. True
  3. False
  4. False
  5. False
The null hypothesis (H0) states that no difference exists between two sample groups. The null hypothesis must be set prior to conducting any clinical trial, and if a difference between the two samples is detected, the null hypothesis must be rejected. It is liable to sources of error and two outcomes of error are possible: Type I or type II errors.
A type I error is also known as an α-error or a false positive. A difference is found when one does not truly exist causing incorrect rejection of the null hypothesis (A). It is most commonly due to a high p-value or a small sample size.
A type II error is also known as a β-error or a false negative (B). No difference is found when one does indeed exist causing incorrect acceptance of the null hypothesis. It is most commonly due to small sample sizes.
The power of a study should be calculated prior to conducting the study as it indicates what sample size is required (C). The power measures the probability of a difference being detected when one exists and is calculated as 1 – β. A power of 80% or above (i.e. β <20%) is sufficient for a study to be adequately powered (D).
Standard deviation (SD) is a measure of spread around central tendency of data:
where Σ is the sum of the difference of each value (x) from the mean (X̄) divided by the number of values (n). The mean ±1 SD should include 68% of data points, ±2 SD include 96% of all data points and ±3 SD include 99% of all data points.
Variance (SD2) is thus calculated by (E):
60. The following devices contain differential pressure transducers:
  1. Vacuum-insulated evaporators
  2. Paramagnetic analysers
  3. End-tidal carbon dioxide analysers
  4. Tec 6 vapourisers
  5. Pneumotachographs
60.
  1. True
  2. True
  3. False
  4. True
  5. True
Differential pressure transducers work by detecting a difference in pressure between two sides of a given sensor. Although many devices utilise this principle, it may be argued that all devices measuring pressure are differential pressure transducers, giving the difference between atmospheric pressure and device pressure. The overall design is one that compares a measurement sample with a reference sample to give a pressure difference, which can be analysed and interpreted (see Figure 1.8).
zoom view
Figure 1.8: Differential pressure transducer. The transducer detects the difference in pressures between measurement and reference samples.
The vacuum-insulated evaporator uses a differential pressure transducer between the top and bottom of the cylinder to calculate the mass of liquid oxygen remaining within the vessel (A).
Paramagnetic analysers are rendered particularly accurate using the principles of differential pressure (B). A reference gas of 21% oxygen is compared with a measured gas with a set concentration of oxygen. When exposed to a magnetic field, the paramagnetic properties of oxygen cause it to be attracted to the field, leading to a difference in pressure between both chambers that is proportional to the partial pressure of oxygen.
End-tidal carbon dioxide analysers do not incorporate differential pressure transducers in their design (C).
Tec 6 desflurane vapourisers contain a differential pressure transducer between the fresh gas flow channel and the vapourising channel, which is then interpreted and electronically alters the gas flow accordingly (D).
Pneumotachographs can be designed in a number of ways, one which uses the Pitot tube design. One tube faces the direction of gas flow, while the other faces the opposite direction, being the reference tube. The pressure difference between the two tubes is proportional to the flow rate squared (E).
11
Questions: SBAs
For each question, select the single best answer from the five options listed.
61. A 73-year-old man has had an asystolic cardiac arrest. You have successfully intubated him and cardiopulmonary resuscitation is underway.
Which is the most likely finding for this patient?
  1. Blood pressure of 52/26 mmHg
  2. End-tidal CO2 of 0.4 kPa
  3. A pH of 7.38
  4. A standard bicarbonate of 38.1 mmol/L
  5. A serum potassium of 7.8 mmol/L
61. B End-tidal CO2 of 0.4 kPa
In a cardiac arrest, there is no cardiac output, therefore no perfusion of the lungs. This is an extreme form of dead space involving both lungs. Although arterial CO2might be elevated, the end-tidal CO2 will be low if there is effective cardiopulmonary resuscitation (CPR) or zero if there is ineffective CPR. Asystolic patients will have no blood pressure and are more than likely going to have a metabolic and/or respiratory acidosis; therefore, the bicarbonate is likely to be low. Elevated serum potassium may cause cardiac arrest, but this is not necessarily true.
Weil MH, Bisera J, Trevino RP, Racklow EC. Cardiac output and end-tidal carbon dioxide. Crit Care Med 1985; 13:907–909.
62. A 68-year-old woman, who had a grade I intubation, is having an open reduction and internal fixation of an ankle fracture. She is being ventilated at a rate of 14 breaths per minute, with a peak inspiratory pressure of 36 cmH2O, achieving a tidal volume of 210 mL.
Which of the following is the most appropriate statement regarding lung compliance?
  1. The patient may have pulmonary oedema
  2. Normally lung compliance is 500 mL/cmH2O
  3. During an asthma attack lung compliance decreases
  4. The patient may have underlying emphysema
  5. It may be plotted on a flow/volume loop
62. A The patient may have pulmonary oedema
Compliance (C) is defined as the change in volume (ΔV) per unit change in pressure (Δρ) and it reflects the elastic recoil of an organ:
In the lung, ΔP is the difference in pressure between alveolar pressure measured at the mouth when there is no gas flow and intrapleural pressure measured by a balloon in the lower third of the oesophagus. Normal lung compliance (Clung) is 150–200 mL/cmH2O (1.5–2.0 L/kPa). Chest wall compliance (Cchest) is normally 200 mL/cmH2O (2.0 L/kPa), where ΔP is the difference between alveolar pressure and intrapleural pressure. Total thoracic compliance (Ctotal) is 85–100 mL/cmH2O (0.85–1 L/kPa) and is related to both Clung and Cchest:
Repeated pressure and volume readings can be plotted on a pressure/volume loop that is approximately linear during normal tidal volume breathing. However, the curves demonstrate hysteresis, meaning that the inflation and deflation curves are different (see Figure 1.9). This is because of the need to initially overcome surface tension upon inflation or inspiration.
Lung compliance can be either static or dynamic. Static compliance refers to the stiffness of the lung and chest wall, i.e. alveolar stretchability and is measured when there is no gas flow. Dynamic compliance is related to airway resistance during equilibration of gases at end-inspiration or end-expiration. Dynamic compliance is usually less than static compliance.
Because compliance is relative to body size, specific compliance is calculated as:
Pulmonary oedema increases lung compliance due to the interstitial oedema.
zoom view
Figure 1.9: Lung pressure/volume loops demonstrating hysteresis.
However, asthma has been shown to reduce lung compliance, a pathology related to a loss of elastic recoil at total lung capacity and increased compliance of airways in spite of airway oedema. Emphysema also increases lung compliance, again due to loss of elastic recoil.
Compliance can be affected by the following factors (see Table 2.1):
  • Increased compliance: surfactant, emphysema, old age, acute asthma (reason unclear)
  • Reduced compliance: pulmonary fibrosis, pulmonary venous engorgement, pulmonary oedema, ARDS, pneumonia, neonates, extremes of lung volumes, chronic bronchitis (dynamic)
Wenzel S. Severe asthma in adults. Am J Respir Crit Care Med 2005; 172:149–160.
West JB. Respiratory physiology: the essentials, 9th edn. Baltimore: Lippincott Williams & Wilkins; 2012.
63. A 23-year-old man has sustained a traumatic intracranial haemorrhage with evidence of raised intracranial pressure (ICP). He is intubated and requires transfer to a specialist neurosurgical centre.
Which of the following is most likely to reduce his ICP?
  1. Increasing his Pao2 from 9.0 to 12 kPa
  2. Sedation with ketamine
  3. Reducing Paco2 from 5.8 to 4.2 kPa
  4. 8 mg of dexamethasone
  5. Fluid restriction
63. C Reducing Paco2 from 5.8 to 4.2 kPa
There are a number of factors affecting intracranial pressure (ICP), and management of traumatic brain injury is aimed at reducing secondary injury. Target Paco2 should be 4.5–5.0 kPa, with a reduction in cerebral blood flow of approximately 2–4% for each 0.13 kPa reduction in Paco2. Avoidance of hypoxia is vital, but the ICP does not significantly increase above a Pao2 of 6.7 kPa. Another management target is avoidance of hypotension and fluid restriction in the immediate phase that reduces mean arterial pressure (MAP), thereby reducing cerebral perfusion pressure (CPP): CPP = MAP – (ICP + CVP). Dexamethasone may reduce cerebral oedema secondary to tumours, but there is no evidence that it affects traumatic brain injury. There are a number of anaesthetic drugs that affect ICP (see Table 1.9).
Curry P, Viernes D, Sharma D. Perioperative management of traumatic brain injury. Int J Crit Illn Inj Sci 2011; 1:27–35.
Table 1.9   Anaesthetic drugs affecting intracranial pressure (ICP)
Increased ICP
Decreased ICP
Ketamine
Barbiturates, etomidate, propofol
Volatile agents
Opioids
Suxamethonium (transient)
64. A 75-year-old man in the emergency department has a heart rate of 44 beats per minute and an unrecordable blood pressure. His pulses are not palpable.
What is the first drug this patient should receive?
  1. Atropine
  2. Adrenaline
  3. Amiodarone
  4. Ephedrine
  5. Metaraminol
64. B Adrenaline
This patient is in pulseless electrical activity (PEA) cardiac arrest. The Resuscitation Council (UK) published their latest guidelines in 2010 that modified the application of atropine in cardiac arrest situations. It is no longer recommended for use in PEA or asystolic cardiac arrests due to a poor evidence base. The first-line pharmacological therapy should be administration of 1 mg adrenaline via the intravenous or intraosseous routes.
Nolan J, Soar J, Lockey A. Advanced life support, 6th edn. London: Resuscitation Council; 2011.
1265. A 54-year-old woman is having a bunionectomy under general anaesthetic. You perform an ankle block using the landmark technique.
Which of the following nerves could a nerve stimulator be used to block?
  1. Deep peroneal nerve
  2. Saphenous nerve
  3. Superficial peroneal nerve
  4. Sural nerve
  5. Tibial nerve
65. E Tibial nerve
An ankle block may be performed to provide analgesia or anaesthesia for forefoot operations. It requires blockade of the deep and superficial peroneal nerves, sural nerve, saphenous nerve and tibial nerve. As the tibial nerve is the only one that has a motor component beyond the site of block, it is the only nerve that a nerve stimulator may identify. Stimulation of the tibial nerve leads to plantar flexing of the toes via contraction of the flexor digitorum longus and flexor hallucis longus. The tibial nerve also provides sensory supply to the medial aspect of the ankle and foot.
Kopka A, Serpell MG. Distal nerve blocks of the lower limb. Cont Educ Anaesth Crit Care Pain 2005; 5 (5): 166–170.
66. An 8-year-old girl (30 kg body weight) presents for tonsillectomy. You have prepared drugs and equipment.
Which calculation is correct?
  1. Length of a nasal tube at the nares is 19 cm
  2. Endotracheal tube size is 5.0 mm internal diameter
  3. Laryngeal mask size is 2.0
  4. Fentanyl dose is 15 µg
  5. Intravenous paracetamol dose is 800 mg
66. A Length of a nasal tube at the nares is 19 cm
Drug dosing and equipment calculation is an important aspect of preparation for paediatric anaesthesia. The important calculations for equipment and drug dosing are as follows:
Equipment
  • Endotracheal tube internal diameter size = (Age/4) + 4
  • Length of oral endotracheal tube at lips = (Age/2) + 12
  • Length of nasal endotracheal tube at nares = (Age/2) + 15
  • Laryngeal mask sizes as defined by Table 1.10.
Drug dosing: for details of drug dosing for paediatric anaesthesis see Table 1.11.
Therefore, this patient should have a size 6.0 mm endotracheal tube at 16 cm at the lips for an oral tube and 19 cm at the nares for a nasal endotracheal tube. The patient should have a size 2.5 or 3 laryngeal mask airway.
Table 1.10   Paediatric laryngeal mask airway (LMA) sizes
LMA size
Patient weight (kg)
1
<5
1.5
5–10
2
10–20
2.5
20–30
3
30–50
Table 1.11   Drug dosing for paediatric anaesthesia
Drug
Dose
Analgesics
Paracetamol
15 mg/kg intravenous (7.5 mg/kg if weight is < 10 kg)
40 mg/kg loading dose per rectum
15 mg/kg every 4–6 hours
Ibuprofen
10 mg/kg
Diclofenac
1 mg/kg
Codeine
0.5 mg/kg
Fentanyl
1 µg/kg
Morphine
0.1–0.2 mg/kg
Antiemetics
Dexamethasone
0.1–0.2 mg/kg
Ondansetron
0.1–0.2 mg/kg
Girgis, Sanders. The BADS Paediatric Dose Table. Journal of One-day Surgery 2004; 14:65–8.
South Thames Retrieval Service. Clinical Guidelines. http://www.strs.nhs.uk/educationandguidelines/guidelines.aspx (Last accessed 01/10/2012).
67. A 34-year-old man has a rapid sequence induction for a perforated duodenal ulcer. Twenty minutes later his heart rate rises to 125 beats per minute, blood pressure decreases to 75/40 mmHg, airway pressures increase to 36 cmH2O and cutaneous urticaria is rapidly spreading.
What is the most likely causative agent?
  1. Atracurium
  2. Succinylcholine
  3. Latex
  4. Morphine
  5. Co-amoxiclav
67. B Succinylcholine
The clinical features described are in keeping with a diagnosis of anaphylaxis. Anaphylaxis is an IgE-mediated type 1 hypersensitivity reaction caused under anaesthesia by the substances shown in Table 1.12.
Thus, in this patient, it is most likely to be due to the use of succinylcholine during rapid sequence induction. However, formal allergy testing would be mandatory.
Association of Anaesthetists of Great Britain and Ireland. Suspected anaphylactic reactions associated with anaesthesia. Anaesthesia 2009; 64:199–211.
Table 1.12   Anaesthetic causes of anaphylaxis
Agent
Frequency (%)
Notes
Neuromuscular blocking drugs
60
Most commonly succinylcholine and rocuronium, rarely atracurium or mivacurium
Latex
20
Most commonly in atopic patients, healthcare workers, fruit allergy
Antibiotics
15
Penicillins and cephalosporins account for 70% of antibiotic-related anaphylaxis
Colloids
4
Most commonly to gelatins, rare with starches
Opioids
Uncommon
Usually due to histamine release
Local anaesthetics
Rare
May be related to preservatives
Anaesthetic agents
Very rare
Rare with propofol, very rare with thiopental, never reported with volatile agents
68. A 68-year-old woman with chronic obstructive airways disease and rheumatoid arthritis presents for a total shoulder placement. You perform an interscalene nerve block using a nerve stimulator technique.
What is the most likely complication in this patient?
  1. Recurrent laryngeal nerve palsy
  2. Horner's syndrome
  3. Pneumothorax
  4. Vertebral artery puncture
  5. Phrenic nerve palsy
68. E Phrenic nerve palsy
An interscalene brachial plexus block is indicated for shoulder, humeral or elbow surgery. It may be performed using the landmark technique in the interscalene groove at the level of C6, ultrasound guidance or a nerve stimulator eliciting deltoid and biceps contraction.
Nearly 100% of patients have a phrenic nerve block; therefore, the block should only be performed unilaterally. Other complications include a stellate ganglion block causing Horner's syndrome in up to 25% of patients, recurrent laryngeal nerve palsy in up to 10% of patients and vagal nerve damage. Vertebral artery and other vessel puncture may occur, and intrathecal or epidural injunction may also take place. Pneumothorax is rare but more common in patients with chronic obstructive airways disease, thus care must be taken in this patient.
Nicholls B, Conn D, Roberts A. The Abbott pocket guide to practical peripheral nerve blockade.
Maidenhead: Abbott Anaesthesia; 2010.
Urmey WF, Talts KH, Sharrock NE. One hundred percent incidence of hemidiaphragmatic paresis associated with interscalene brachial plexus anesthesia as diagnosed by ultrasonography. Anesthesia and Analgesia 1991; 72: 498–503.
69. A 44-year-old man (100 kg body weight) in the intensive care unit with acute respiratory distress syndrome is being ventilated with volume-controlled ventilation at tidal volumes of 620 mL, peak inspiratory pressures of 29 cmH2O, 13positive end-expiratory pressure (PEEP) of 6 cmH2O and a Fio2 of 0.9. His arterial blood gas demonstrates a Pao2 of 6.9 kPa and a Paco2 of 7.3 kPa.
What intervention is most likely to increase his Pao2?
  1. Increase the tidal volume
  2. Increase the Fio2
  3. Increase the expiratory time
  4. Increase the respiratory frequency
  5. Increase the PEEP
69. E Increase the positive end-expiratory pressure (PEEP)
Acute respiratory distress syndrome (ARDS) is traditionally defined by the American-European Consensus Conference (1994) as:
  • Acute onset
  • Bilateral infiltrates on chest X-ray
  • Absence of left atrial hypertension
  • A Pao2:Fio2 ratio of <200 mmHg (26.7 kPa)
A more recent consensus definition of ARDS has now replaced this definition. The Berlin definition (2012) involves:
  • Onset of ARDS within 7 days of a defined event (e.g. sepsis)
  • Bilateral opacities consistent with pulmonary oedema on either chest X-ray or CT scan
  • Respiratory failure not fully explained by cardiac failure or fluid overload; objective assessment with echocardiography should be performed.
ARDS is caused by either pulmonary or extrapulmonary pathology, both producing hypoxia due to an increase in both shunt and dead space
Treating hypoxia is best achieved with alveolar recruitment. This can be done by increasing the mean airway pressures, either by increasing the positive end-expiratory pressure (PEEP) or prolonging the inspiratory time. The ARDSnet trial have recommended a lung-protective strategy for ventilating patients with ARDS, and the recommendations include:
  • Use of pressure-controlled ventilation
  • Aim for tidal volumes of 6 mL/kg of ideal body weight
  • Plateau pressures of <30 cmH2O
  • PEEP of 10–12 cmH2O
  • Titrate Fio2 to a Pao2 of 8 kPa
  • Permissive hypercapnia of 8 kPa, increasing the respiratory rate to reduce the Paco2
In this patient, increasing the tidal volumes will increase peak airway pressures and have minimal effect on mean airway pressures. The patient already has an Fio2 of 0.9; thus, increasing the Fio2 will have minimal effect on hypoxia and will not aid alveolar recruitment. Increasing the expiratory time will only shorten the inspiratory time, lead to higher peak airway pressures and reduce alveolar recruitment. Increasing the respiratory rate will have a greater effect on offloading CO2 than oxygenation and will be of limited benefit in this scenario.
The Acute Respiratory Distress Syndrome Network. Ventilation with lower tidal volumes as compared with traditional tidal volumes for acute lung injury and the acute respiratory distress syndrome. N Engl J Med 2000; 342:1301–1308.
The ARDS Definition Task Force. Ranieri VM, Rubenfeld GD, et al. Acute respiratory distress syndrome: the Berlin definition. JAMA 2012; 307:2526–2533.
70. A 22-year-old woman with a past medical history of asthma presents with difficulty in breathing, a respiratory rate of 28 breaths per minute, inability to complete sentences and wheeze. An arterial blood gas breathing room air reveals a Pao2 of 8.5 kPa and a Paco2 of 3.4 kPa. Five milligrams of nebulised salbutamol has been administered.
What is the next step in this patient's management?
  1. Nebulised antimuscarinic agents
  2. Check peak expiratory flow rates
  3. Intubation and ventilation
  4. Intravenous magnesium
  5. Request a chest radiograph
70. A Nebulised antimuscarinic agents
This patient has evidence of acute severe asthma, which is defined by the British Thoracic Society (BTS) as any one of:
  • Peak expiratory flow rate (PEFR) of 33–50% of best or predicted
  • Respiratory rate of 25 breaths per minute or more
  • Heart rate or 110 beats per minute or more
  • Inability to complete sentences in one breath
Life-threatening asthma as defined by BTS (2012) guidelines includes:
  • PEFR of <33% best or predicted
  • Spo2 <92%
  • Quiet chest, cyanosis or reduced respiratory effort
  • Arrhythmia or hypotension
  • Altered consciousness
  • ‘Normal’ Paco2 of 4.6–6.0 kPa
  • Severe hypoxia with Pao2 of <8.0 kPa
  • Acidosis
The immediate treatment of acute severe asthma in adults is:
  • Oxygen to maintain Spo2 94–98%
  • Nebulised salbutamol 5 mg or terbutaline 10 mg (β2-agonists)
  • Nebulised ipratropium bromide 0.5 mg (antimuscarinic)
  • Oral prednisolone 40–50 mg or intravenous hydrocortisone 100 mg
  • Chest radiograph only if pneumothorax or consolidation is suspected or patient requires intubation
  • If life-threatening asthma, then also:
    • Consider ventilation
    • Consider intravenous magnesium sulphate 1.2–2 g via intravenous infusion
    • More frequent salbutamol nebulisers
This patient does not yet require intubation and ventilation, and there is no evidence of requirement for chest radiography at this current stage. PEFR assessment should be performed after immediate treatment as the diagnosis of acute severe asthma is already made. Intravenous magnesium should only be used once conventional therapy has been used and not been successful.
British Thoracic Society/Scottish Intercollegiate Guidelines Network. British Guideline on the Management of Asthma. January 2012 (http://www.brit-thoracic.org.uk/Portals/0/Guidelines/AsthmaGuidelines/sign101%20Jan%202012.pdf) (Last accessed 01/10/2012).
71. While trying to attach a cylinder to the anaesthetic machine, you realise it does not fit appropriately. The pin index shows positions 3 and 5.
Which is the most likely cylinder?
  1. Nitrous oxide
  2. Oxygen
  3. Air
  4. Carbon dioxide
  5. Entonox
71. A Nitrous oxide
The pin index system is a safety mechanism designed to avoid attachment of the incorrect cylinder to the wrong yoke on the anaesthetic machine. Each gas has a specific pin index to ensure the valve block holes correspond to the machine yoke (see Figure 1.10). The pin index system positions for the common gases are shown in Table 1.13.
zoom view
Figure 1.10: The pin index system showing pin positions.
Table 1.13   Pin index system positions for the common gases
Gas
Pin index system positions
Nitrous oxide
3 and 5
Oxygen
2 and 5
Air
1 and 5
Carbon dioxide
1 and 6
Entonox
7
Al-Shaikh B, Stacey S. Essentials of anaesthetic equipment, 3rd edn. Edinburgh: Churchill Livingstone; 2007.
72. A 36-year-old woman has oxygenation saturations of 96% on pulse oximetry but 88% on arterial blood gas oximetry analysis.
What is the most likely cause for this discrepancy?
  1. Hyperbilirubinaemia
  2. Indocyanine green dye
  3. Methaemoglobinaemia
  4. Carboxyhaemoglobin
  5. Presence of nail varnish
72. D Carboxyhaemoglobin
Pulse oximetry uses the principle of light absorption based on Beer's and Lambert's laws. They are accurate above 70% to ±2%, below which the saturation is extrapolated from studies. Pulse oximeters under read in presence of:
  • Indocyanine green dye
  • Methaemoglobinaemia
  • Methylene blue dye
  • Nail varnish
It over-reads, as in this patient, in the presence of carboxyhaemoglobin because the absorption coefficient is similar to that of oxyhaemoglobin, causing the readings to be 96%.
Pedersen T, Moller AM, Pedersen BD. Pulse oximetry for perioperative monitoring: Systematic review of randomized, controlled trials. Anesth Analg 2003; 96:426–431.
Al-Shaikh B, Stacey S. Essentials of anaesthetic equipment, 3rd edn. Edinburgh: Churchill Livingstone; 2007.
73. While floating a pulmonary artery catheter, a pulmonary capillary wedge pressure of 15 mmHg is shown on the monitor.
This represents the pressure in which of the following?
14
  1. Right atrium
  2. Right ventricle
  3. Left atrium
  4. Left ventricle
  5. Pulmonary artery
73. C Left atrium
The tip of the catheter is placed in the pulmonary artery tree where it is wedged. Once wedged the pulsatile waveform is lost to continuous low pressure reading; this is the pulmonary artery occlusion pressure or the pulmonary capillary wedge pressure; and it is an accurate representation of the left atrial pressure. However, the presence of the mitral valve means that the pressure analysed is more directly related to the left atrium, although the left ventricular pressure can be inferred from this. Because the pressure being measured is beyond the right atrium, ventricle and pulmonary artery, it is not related to these three pressures.
Levick JR. An introduction to cardiovascular physiology, 5th edn. London: Hodder Arnold; 2010.
74. Using the flow of water through a pipe, driven by a mechanical pump as an analogy for electricity, the pressure difference between two points in the pipe would correspond to which of the following?
  1. Current
  2. Work
  3. Voltage
  4. Resistance
  5. Power
74. C Voltage
The water pressure difference between two points would correspond to the voltage difference in an electrical circuit. If there is a water pressure difference between two points, then water will flow from one point to the other and this would allow the water to do work, such as rotate the blades in a turbine. Similarly, work is done by the flow of an electric current in a circuit driven by a voltage difference; the current which is generated could then provide power to an electrical device such as a light bulb. If the water pump is not working, then there will be no pressure difference generated so the turbine blades will not rotate, similarly if there is no flow of current then the bulb will not light up.
Davis PD, Kenny GNC. Basic physics and measurement in anaesthesia, 5th edn. Boston: Butterworth-Heinemann; 2003.
75. A 20-year-old woman has arrived in the anaesthetic room for elective orthopaedic surgery. She is needle phobic and is insisting on a gaseous induction of anaesthesia.
Which of the following would most efficiently and rapidly allow you to perform a gas induction?
  1. Bain breathing system
  2. Mapleson A breathing system
  3. Mapleson B breathing system
  4. Mapleson C breathing system
  5. Mapleson D breathing system
75. B Mapleson A breathing system
Mapleson A breathing systems are the most efficient for spontaneous ventilation, and the fresh gas flows required are equal to alveolar minute ventilation. This system is not efficient for controlled ventilation, as it would require fresh gas flows three times the minute ventilation. Mapleson B and C systems are not efficient for spontaneous or controlled ventilation; however, the B system is more efficient than the A and C systems for controlled ventilation.
The Bain system is a coaxial version of the Mapleson D breathing system. This system is not efficient for spontaneous ventilation, requiring a fresh gas flow of about twice the alveolar minute ventilation, but is efficient during controlled ventilation.
Mapleson WW. Editorial I: Fifty years after – reflections on ‘The elimination of rebreathing in various semi-closed anaesthetic systems’. Br J Anaesth 2004; 93:319–321.
76. A 75-year-old man has had emergency vascular surgery. He presented with a full stomach, therefore required a rapid sequence induction to secure his airway followed by prolonged surgery requiring significant blood transfusion.
Which of the following extubation strategies is least appropriate?
  1. Deep extubation
  2. Laryngeal mask exchange
  3. Airway exchange catheter
  4. Remifentanil technique
  5. Awake extubation
76. A Deep extubation
This patient would be classed as an ‘at risk’ extubation; he has a full stomach, has undergone emergency surgery taking several hours and lost significant amount of blood requiring blood transfusion. The patient may not be able to maintain his own airway after the tracheal tube has been removed. An ‘at-risk’ extubation is characterised by the concern that once extubated, airway management may be difficult. A deep extubation should not be performed in such a patient. Deep extubation should only be performed in ‘low-risk’ extubation cases; these are routine extubations where reintubation could be managed without difficulty if required.
All the other options are appropriate techniques for ‘at-risk’ extubations, but must be performed by clinicians who are experienced in the technique.
Difficult Airway Society Extubation Guidelines Group. Popat M, Mitchell V, et al. Difficult Airway Society Guidelines for the management of tracheal extubation. Anaesthesia 2012; 67:318–340.
77. A 35-year-old man requires surgery to repair a lacerated biceps tendon. He had been assaulted and sustained a left-sided pneumothorax and a 5 cm stab wound to his right antecubital fossa.
Which of the following ultrasound-guided upper limb blocks would be the most appropriate for surgery?
  1. Interscalene
  2. Supraclavicular
  3. Infraclavicular
    15
  4. Axillary
  5. Peripheral nerve block
77. D Axillary
This patient has a pneumothorax on the contralateral side to where the block will be performed. Either a supraclavicular, infraclavicular or axillary nerve block will provide appropriate analgesic coverage for the area where surgery will be performed. The dermatomal distribution for this is region is C5, C6 and T1 where the sensory innervation will be from the median and ulnar nerves.
For a supraclavicular block, the brachial plexus is blocked at the level of the divisions and when performed under ultrasound guidance, the risk of pneumothorax is reduced, but not completely eliminated, even by experienced practitioners. Therefore, there is still a risk of pneumothorax, which must be avoided as the patient already has a pneumothorax on the contralateral side.
An interscalene block would not provide adequate cover for an incision over the antecubital fossa. There is also a high chance of phrenic nerve palsy on the ipsilateral side, thus the patient would be at risk of respiratory distress as he already has a pneumothorax on the left and a phrenic nerve palsy on the right could further compromise ventilation. This is a further reason for not choosing this block.
Axillary and infraclavicular blocks both cover a similar area, which will be adequate for surgery in the antecubital fossa. But there is still some associated risk of pneumothorax when performing an infraclavicular block, even if guided by ultrasound.
Therefore, the most appropriate block would be an axillary nerve block, this will provide adequate coverage to the antecubital fossa and has no risk of pneumothorax. A peripheral nerve block is less likely to be as effective as an axillary block in this circumstance.
Perlas A, Lobo G, Lo N, et al. Ultrasound-guided supraclavicular block: outcome of 510 consecutive cases. Reg Anesth Pain Med 2009; 34:171–176.
78. The components of a circle breathing system attached to a standard anaesthetic machine include soda lime in a canister.
Which of the following is incorrect regarding soda lime?
  1. It contains a pink dye
  2. The dye changes to white when the soda lime is exhausted
  3. The size of the granules should be 4–8 mesh
  4. It consists of 95% sodium hydroxide
  5. Silica is added
78. D It consists of 95% sodium hydroxide
In the circle breathing system soda lime is added to absorb exhaled carbon dioxide. As a result fresh gas flows may be low and, compared with all the Mapleson breathing systems, this makes the circle breathing system a very efficient and economical one. It also causes minimal pollution.
The soda lime is placed in a canister which has two ports: one to deliver fresh gases to the patient and another to receive exhaled gases via unidirectional valves. Soda lime is made of 94% calcium hydroxide, 5% sodium hydroxide and a small amount of potassium hydroxide.
A dye is added which indicates when the soda lime is exhausted; this can either be from pink to white or white to violet. Silica is added to stop the granules from disintegrating into dust particles, which could be inhaled by the patient. The soda lime granules are 4–8 mesh in size, but they can also be made into spherical shapes of 3–4 mm in size.
Al-Shaikh B, Stacey S. Essentials of anaesthetic equipment, 3rd edn. Edinburgh: Churchill Livingstone; 2007.
79. A 35-year-old woman is having laparoscopic gynaecological surgery. She is anaesthetised and has a blood pressure of 110/60 mmHg and heart rate of 60 beats per minute as the surgeon is insufflating the peritoneum with gas. There is an immediate complication as a result of the pneumoperitoneum.
Which of the following emergency drugs would you first administer?
  1. Adrenaline
  2. Atropine
  3. Ephedrine
  4. Suxamethonium
  5. Metaraminol
79. B Atropine
The patient has a low heart rate at this point, and one of the commonest complications of pneumoperitoneum is bradycardia as a result of stimulation of the Vagus nerve due to rapid peritoneal stretching. The combination of a pre-existing low heart rate and vagal stimulation will most likely cause a bradycardia requiring immediate atropine administration and temporary cessation of air insufflation.
Metaraminol would make the bradycardia worse, so should not be administered in this scenario.
Ephedrine and adrenaline could be administered, but there is nothing to suggest that the blood pressure has been compromised, and therefore the first line of treatment should be atropine, followed by ephedrine if the blood pressure is compromised.
There are many complications associated with gas insufflation for laparoscopic surgery including:
  • Trauma to intra-abdominal viscera and great vessels
  • Gas embolus if gas is insufflated into a blood vessel
  • Pneumothorax
  • Pneumomediastinum
  • Subcutaneous emphysema
  • Caval compression
Perrin M, Fletcher A. Laparoscopic abdominal surgery. Contin Educ Anaesth Crit Care Pain 2004; 4:107–110.
80. You have chosen to use an infusion of remifentanil and propofol to induce and maintain anaesthesia for a patient undergoing elective surgery for a plastics procedure.
Which of the following is least likely to guarantee drug delivery using this technique?
  1. Use of non-return valves
  2. Keeping the cannulation site visible at all times
  3. Being aware of the uses and limitations of the infusions pumps
  4. Infusing intravenous fluids through the same cannula as the drugs
  5. Using a large proximal vein for infusion
80. E Using a large proximal vein for infusion
The Safe Anaesthesia Liaison Group has published guidelines in 2009 to help guarantee the administration of drugs during total intravenous anaesthesia. This was after a number of incidences of awareness as a result of inadequate drug delivery.
Non-return or one-way valves prevent the backflow of the anaesthetic drugs into the intravenous fluid lines, therefore guaranteeing the delivery of these drugs to the patient.
Monitoring the cannulation site at regular intervals is a simple way to reduce the risk of inadequate drug delivery by ensuring that accidental disconnection, backtracking of anaesthetic drugs or displacement of the cannula are noted early.
Infusing fluids through the same cannula as the anaesthetic drugs will help ensure their delivery because if the fluid infusion abruptly stops or suddenly flows at a very fast rate, occlusion or disconnection of the cannula could be the cause and this can then be investigated and resolved.
The use of a large vein as opposed to a smaller vein does not further guarantee drug delivery.
Safe Anaesthesia Liaison Group. Guaranteeing drug delivery in total intravenous anaesthesia. London: National Patient Safety Agency; October 2009.
81. A 46-year-old man is being anaesthetised for functional endoscopic sinus surgery to remove nasal polyps as a day case procedure.
Which of the following is most likely to reduce intraoperative bleeding?
  1. Hypotensive anaesthesia
  2. Normocarbia
  3. Reverse Trendelenburg tilt
  4. Use of Moffat's solution
  5. Normothermia
81. A Hypotensive anaesthesia
Intraoperative bleeding is one of the major problems of this type of surgery and can lead to complications such as impairing the surgeon's vision, increasing the duration of surgery and increasing the risk of intracranial and orbital complications. Blood loss is primarily dependent on cut vessels, but can be reduced considerably by a reduction in the mean arterial pressure. Controlled hypotension deliberately and predictably decreases mean arterial pressure to limit intraoperative blood loss and is the method most likely to reduce intraoperative bleeding.
Normocarbia will prevent increased blood flow to the head and neck region as a result of vasodilation which would otherwise occur with hypercarbia. The reverse Trendelenburg position helps reduce local blood flow to the capillary bed by having the patient in a slightly head-up position. However, neither of these techniques is as effective as hypotensive anaesthesia.
Moffat's solution is a sprayed onto the nasal mucosa to induce vasoconstriction, reduce intraoperative bleeding, reduce congestion of the nose and act as a local anaesthetic. It consists of a combination of cocaine, adrenaline and sodium bicarbonate. This is often required within a multimodal approach for generating a bloodless field but is usually insufficient in isolation. Surgeons often supplement Moffat's solution with lignocaine and adrenaline injections intraoperatively because it rarely guarantees a bloodless field.
Hypothermia has been shown to increase intraoperative blood loss and the need for blood transfusion due to hypothermia-induced impairment of platelet function; this is more likely to occur with a core temperature of 35–35.7°C, therefore will have less influence on this short duration surgery.
Allman KG, Wilson IH. Oxford handbook of anaesthesia, 3rd edn. Oxford: Oxford University Press; 2011.
1682. You are about to perform a fibreoptic intubation in a patient with a known difficult airway.
Which of the following would be the most appropriate method of preventing cross-contamination with a fibreoptic scope?
  1. Cleaning
  2. Sterilisation
  3. Disinfection
  4. Decontamination
  5. Autoclaving
82. D Decontamination
Decontamination is the process of removing contaminants such that they are unable to reach a site in sufficient quantities to initiate an infection or other harmful reaction. The process of decontamination always starts with cleaning and is followed by disinfection or sterilisation. Fibrescopes are classed as intermediate risk equipment (do not penetrate the skin or enter sterile cavities) so decontamination by cleaning followed by disinfection is an acceptable method of cleaning this type of equipment.
Cleaning is the process of physically removing foreign material from an object without necessarily destroying any infective material. An example of manual cleaning is washing, and automated cleaning is performed using an ultrasonic bath or low-temperature steam.
Disinfection is the process of rendering an object free from all pathological organisms except bacterial spores. Chemicals used for disinfection are glutaraldehyde 2%, alcohol 60–80%, chlorhexidine 0.5–5% and hydrogen peroxide. Pasteurisation is the process of using heat to destroy pathogenic organisms without destroying bacterial spores. It is an alternative to chemical disinfection.
Sterilisation is the process of rendering an object completely free of all visible infectious agents including bacterial spores. This can be done using chemicals such as ethylene oxide and glutaraldehyde 2%, using radiation like gamma irradiation, or using heat such as the autoclave.
Sabir N, Ramachandra V. Decontamination of anaesthetic equipment. Contin Educ Anaesth Crit Care Pain 2004; 4:134–135.
83. A 43-year-old woman having a laparoscopic cholecystectomy has an oxygen saturation which rapidly decreases to 80% on 100% inspired oxygen, peak airway pressures increase to 38 cmH2O, and blood pressure reduces to 72/42 mmHg. Examination demonstrates absent breath sounds and a hyper-resonant percussion note on the left side of the chest.
What is the most appropriate immediate management?
  1. Increase tidal volumes
  2. 50–100 µg of adrenaline
  3. Needle decompression
  4. Increase respiratory rate
  5. Fluid bolus
83. C Needle decompression
This patient is demonstrating clinical features of a tension pneumothorax with hypoxia, elevated airway pressures and hypotension. This is a recognised risk of laparoscopic surgery and requires immediate recognition and treatment with insertion of a cannula in the second intercostal space, midclavicular line on the affected side. An intercostal chest drain should then be inserted for this life-threatening emergency.
Adrenaline may be used but would not treat the underlying cause. Optimising ventilatory parameters would not treat the cause and may worsen any tension by increasing the size of the pneumothorax. Administration of intravenous fluid boluses would be of benefit but would not be addressing the underlying cause.
Hayden P, Cowman S. Anaesthesia for laparoscopic surgery. Contin Educ Anaesth Crit Care Pain 2011; 11:177–180.
84. A 68-year-old woman presents with shortness of breath and weakness. A 12-lead electrocardiogram shows a rate of 42 beats per minute with complete dissociation of P waves from QRS complexes.
What is the most likely jugular venous pressure (JVP) waveform that will be seen?
  1. Absent JVP waveforms
  2. Absent a waves
  3. Raised JVP with normal waveforms
  4. Large v waves
  5. Cannon a waves
84. E Cannon a waves
The jugular venous pressure (JVP) is assessed by viewing the height and pulsations of the jugular veins with the patient sitting at 45° and the head turned slightly away. It has a characteristic double-pulsatile waveform pattern, distinguishing it from arterial pulsations (see Figure 1.11):
  • a wave: atrial contraction
  • c wave: bulging of the closed tricuspid valve at the start of isovolumetric right ventricular contraction
  • x descent: atrial relaxation
  • v wave: venous return against a closed tricuspid valve
  • y descent: atrial relaxation while emptying into the right ventricle
zoom view
Figure 1.11: The jugular venous pressure (JVP) waveform.
Abnormalities of the JVP waveform may reflect underlying pathology:
  • Absent JVP may be present in patients who are intravascularly deplete
  • Elevated JVP (>4 cm above the sternal angle) with normal waveforms may reflect right heart failure and fluid overload
  • Elevated JVP with absent waveforms may be caused by superior vena cava obstruction
  • Cannon a waves are caused by atrial contraction against a closed tricuspid valve such as in complete heart block
  • Absent a waves may be seen in atrial fibrillation
  • Large v waves is a feature of tricuspid regurgitation
  • Elevated JVP with deep x and y descents may be seen with constrictive pericarditis
The electrocardiographic findings in this patient are consistent with complete atrioventricular dissociation, as is seen in complete heart block. Thus, the most likely JVP waveforms will be cannon a waves.
Levick JR. An introduction to cardiovascular physiology, 5th edn. London: Hodder Arnold; 2010.
85. A 45-year-old man presents with a productive cough, shortness of breath and pyrexia. He has saturations of 91% on room air. An arterial blood gas sample (ABG) is performed but the plastic syringe is left for more than 60 minutes at room temperature prior to analysis.
What is the most likely erroneous result on the ABG analysis?
  1. Reduced pH
  2. Elevated Pao2
  3. Reduced Paco2
  4. Increased glucose
  5. Reduced lactate
85. A Reduced pH
Once an arterial blood gas (ABG) sample is collected, it should be analysed as soon as possible. Within the ABG syringe cellular metabolism continues. Dissolved oxygen is utilised; however, this is only of a limited quantity and anaerobic metabolism ensues and carbon dioxide is produced, thereby increasing the Paco2. Anaerobic metabolism persists utilising glucose stores and producing lactate. The overall effect of delayed analysis of an ABG sample can be summarised as follows:
  • Reduced pH
  • Reduced Pao2
  • Increased Paco2
  • Reduced glucose
  • Increased lactate
Other errors in analysis of an ABG sample include:
  • Air bubbles in syringe: may incorrectly increase Pao2, saturation and pH of the sample
  • Clotting of the sample: may incorrectly increase the potassium concentration of the sample
  • Dilution of the sample with arterial line saline flush or liquid heparin: may incorrectly increase Pao2 while decreasing Paco2, glucose, lactate, haemoglobin and potassium
  • Accidental venepuncture during ABG sample collection: may incorrectly decrease Po2 and saturations, while increasing Pco2
Wennecke G, Juel G. Avoiding pre-analytical errors in blood gas testing. Radiometer Medical ApS; 2008.
1786. A 46-year-old woman is undergoing an open reduction and internal fixation of a tibial plateau fracture. She is breathing spontaneously through a laryngeal mask airway. Ten minutes following prosthesis insertion, her respiratory rate increases to 34 breaths per minute, oxygen saturations decrease to 88% with an Fio2 of 0.5, end-tidal CO2 reduces to 2.4 kPa and a petechial rash is noted on her chest.
What is the most likely diagnosis?
  1. Haemorrhage
  2. Anaphylaxis
  3. Fat embolism syndrome
  4. Venous thromboembolism
  5. Myocardial ischaemia
86. C Fat embolism syndrome
This patient has clinical features of fat embolism syndrome. This condition occurs when fat from the bone marrow enters the circulation and restricts right ventricular outflow by increasing pulmonary artery pressures. It most frequently occurs after major trauma, but bone fixation may also cause it. The classical triad of clinical features includes:
  • Respiratory symptoms: tachypnoea, dyspnoea, pulmonary oedema, hypoxia
  • Neurological features: confusion, drowsiness
  • Petechial rash: usually on chest wall, axilla or conjunctiva
Other clinical features include tachycardia, pyrexia, retinal emboli on fundoscopy, renal dysfunction, thrombocytopaenia, anaemia, increased erythrocyte sedimentation rate and fat macroglobulinaemia. Treatment is predominantly supportive.
Haemorrhage would present with cardiovascular as well as respiratory changes, and a rash is not a common presentation for acute haemorrhage. Although anaphylaxis may have a similar presentation, the rash is not typically petechial but urticarial. In addition, cardiovascular changes usually predominate in anaphylaxis. Venous thromboembolism is possible, yet in the time frame described it is less likely. Finally myocardial ischaemia does not usually present with a rash.
Gupta A, Reilly CS. Fat embolism. Contin Educ Anaesth Crit Care Pain 2007; 7:148–151.
87. A 28-year-old man with a traumatic brain injury is to be transferred to a neurosurgical centre 90 minutes away. He is intubated and being ventilated with an Fio2 of 1.0 and a minute ventilation of 10 L per minute.
What is the minimum number of full size E oxygen cylinders that would be needed for transfer?
  1. One size E oxygen cylinders
  2. Two size E oxygen cylinders
  3. Three size E oxygen cylinders
  4. Four size E oxygen cylinders
  5. Five size E oxygen cylinders
87. B Two size E oxygen cylinders
Calculation of the volume of oxygen cylinders required for a transfer depends on two factors: the oxygen delivered and the duration of the transfer.
Oxygen delivered = Fio2× minute volume
Oxygen delivered = 1.0 ×10 L/min
Oxygen delivered = 10 L/min
A size E oxygen cylinder contains a volume of 680 L of oxygen, thus would last for 68 minutes in total. A journey of 90 minutes would therefore require a minimum of 2 size E oxygen cylinders, as this would provide enough oxygen to last 136 minutes.
Shouman YM. More information from the pressure gauge of oxygen cylinders. Anesth Analg 2004; 99;307–308.
88. A 44-year-old woman is undergoing free flap surgery for breast reconstruction. The surgeons are concerned about microvascular perfusion in the donor tissue. The blood pressure decreases to 85/44 mmHg.
What is the most important variable to consider in order to optimise flap perfusion?
  1. Blood pressure
  2. Heart rate
  3. Blood viscosity
  4. Vessel calibre
  5. Temperature
88. D Vessel calibre
Blood flow to the flap is determined predominantly by the Hagen-Poiseuille equation:
where η is fluid viscosity, l is length of the tube, r is the radius of the tube and ΔP is pressure gradient along the tube.
Although the blood pressure and viscosity play an important role in microvascular perfusion, a change in the calibre or radius of vessels leads to a change in blood flow to the power of 4. Therefore, it is the vessel radius that is the most important variable.
Temperature may affect a combination of variables including viscosity and vessel radius; however, it is ultimately the radius that is vital.
Although heart rate may affect the cardiac output, it may not necessarily increase flap perfusion to the extent of vessel diameter.
Adams J, Charlton P. Anaesthesia for microvascular free tissue transfer. BJA CEPD Reviews 2003; 3:33–37.
89. A 65-year-old man presents with a 12-hour history of fatigue, confusion and chest pain with a plasma normal troponin concentration. A 12-lead electrocardiogram shows a heart rate of 32 beats per minute with complete dissociation of P waves from QRS complexes.
Which drug is most likely to increase the heart rate in this patient?
  1. Isoprenaline
  2. Atropine
  3. Phenylephrine
  4. Salbutamol
  5. Noradrenaline
89. A Isoprenaline
This patient has symptomatic complete heart block that is due to complete atrioventricular (AV) dissociation. The ideal pharmacological agent for this patient would be one with positive chronotropic effects, mediated via β1-adrenoceptors. Anticholinergic drugs such as atropine would be ineffective because they would act to increase the atrial rate but have no effect on ventricular rate because of the AV dissociation.
Isoprenaline is a potent synthetic β1- and β2-adrenoceptor agonist with no α-adrenoceptor effects. Thus, it has positive inotropic and chronotropic effects, increasing the cardiac output. It was thought to be the ideal drug for patients with complete heart block.
Phenylephrine is a pure α1-adrenoceptor agonist with no β-effects, therefore would not have any effect on heart rate or contractility in this patient.
Salbutamol is predominantly a β2-adrenoceptor agonist; thus, it induces smooth muscle relaxation and vasodilatation. It will have no direct effects to increase myocardial contractility or heart rate.
Noradrenaline predominantly acts via α1-adrenoceptors to causes peripheral vasoconstriction, hypertension and, potentially, a reflex bradycardia, although this reflex may be absent in a patient with complete AV dissociation. Noradrenaline has minimal influence on increasing heart rate in patients with complete heart block, and therefore would not be indicated in this patient.
Sasada M, Smith S. Drugs in anaesthesia and intensive care, 4th edn. Oxford: Oxford University Press; 2011.
1890. A 66-year-old man with severe chronic obstructive airways disease presents for a laparoscopic hernia repair as a day case.
Which is the most appropriate anaesthetic agent to use in this patient?
  1. Bupivacaine
  2. Desflurane
  3. Sevoflurane
  4. Remifentanil
  5. Lidocaine
90. C Sevoflurane
Laparoscopic hernia repairs are most frequently done under general anaesthetic, although there a few reports of regional anaesthetic techniques being used but this is not common practice. General anaesthesia is required to ensure optimal ventilation, avoidance of excessive build-up of CO2 and abdominal discomfort despite adequate regional blockade. Therefore, lidocaine and bupivacaine would not be ideal agents in this patient.
Under general anaesthesia, the ideal agent in this patient would be sevoflurane because it provides bronchodilator properties that would be of benefit to patients with chronic obstructive airways disease (COAD). Desflurane is irritative to airways and can precipitate bronchospasm, particularly in patients with COAD.
Although a remifentanil infusion would provide intraoperative analgesia and blood pressure control, it is not an anaesthetic agent in itself, and therefore cannot be used in isolation.
Molinelli BM, Tagliavia A, Bernstein D. Total extraperitoneal preperitoneal laparoscopic hernia repair using spinal anesthesia. JSLS 2006; 10:341–344.
19
Answers: MTFs
1. Regarding the cardiac cycle, which of the following are true:
  1. At the end of atrial systole, the mitral valve closes because the pressure in the left atrium exceeds that in the left ventricle
  2. At the end of isovolumetric contraction, the aortic valve opens at approximately 120 mmHg
  3. Isovolumetric relaxation signifies the beginning of diastole
  4. The pressure/volume trace of the right ventricle has the same morphology as the left ventricle
  5. The dicrotic notch appears on the left ventricle pressure trace
1.
  1. False
  2. False
  3. True
  4. True
  5. False
The atria and ventricles contract in sequence, which results in a cycle of pressure and volume changes (see Figure 1.1a and b). During atrial systole, blood flows from the atria through the atrioventricular valves and into the ventricles. The volume of blood in a ventricle at the end of the filling phase is the end-diastolic volume (about 120 mL in adults). As soon as the left ventricular pressure rises above the left atrial pressure, the mitral valve closes (A). This is the beginning of isovolumetric contraction.
zoom view
Figure 1.1: (a) The pressure/time curve for the left ventricle. (b) The flow/time curve for the left ventricle.
20During isovolumetric contraction the ventricle is a closed chamber, allowing a steep pressure rise, generated as a result of increasing ventricular wall tension. When the ventricular pressure exceeds aortic pressure, usually approximately 80 mmHg, then the aortic valve opens, isovolumetric contraction ends and ejection into the aorta begins (B).
At the end of the ejection phase, the aortic and pulmonary valves close and once again the ventricles become closed chambers; this is isovolumetric relaxation. As the pressure in the ventricles falls below atrial pressure, the atrioventricular valves open and blood then flows from the atria into the ventricles. Both mitral and aortic valves are closed during isovolumetric relaxation, and there is a rapid fall in pressure; this is the first stage of diastole (C).
Both the left and right ventricles have the same pressure/volume traces or morphology, but the pressures are lower in the right ventricle (D).
The dicrotic notch is seen on the aortic pressure trace and is as a result of the elastic recoil of the aortic walls after the aortic valve closes. It is not seen in the left ventricular pressure trace (E).
2. Coronary blood flow:
  1. Is approximately 500 mL/min
  2. In the left coronary artery is constant throughout the cardiac cycle
  3. In the right coronary artery is always greater than the left coronary artery
  4. In the right coronary artery occurs throughout the cardiac cycle
  5. In the left coronary artery peaks during systole
2.
  1. False
  2. False
  3. False
  4. True
  5. False
Coronary blood flow is approximately 200–250 mL/min in an average adult and comprises about 5% of the total cardiac output (A).
Flow in the right coronary artery is greater than that in the left during systole, but less during diastole. The blood flow within the left coronary artery varies throughout the cardiac cycle (B).
Flow through the right coronary artery occurs throughout the cycle as the transmitted intracavity pressures are low compared with the left ventricle, where the higher pressures lead to compression of the coronary vessels and left coronary blood flow almost ceases during systole (see Figure 1.2) (C, D, E).
Coronary blood flow is least during systole and mostly occurs during diastole.
The coronary perfusion pressure (CPP) determines the maximum pressure perfusing the coronary arteries and can be quantified by:
CPP = aortic-diastolic pressure – left ventricular end-diastolic pressure.
The above equation demonstrates that the greatest perfusion occurs during diastole.
21
zoom view
Figure 1.2: Left and right coronary arterial blood flow in different phases of the cardiac cycle.
3. The left ventricle:
  1. Has a wall that is three times thicker than that of the right ventricle
  2. Takes about 2 seconds to fill during ventricular diastole at rest
  3. Is responsible for the ‘apex beat’
  4. Ejects blood into the right coronary artery
  5. Contains papillary muscles connected to the aortic valves
3.
  1. True
  2. False
  3. True
  4. True
  5. False
The left ventricular wall is about three times thicker than the right, as it has to generate much higher pressures to provide blood flow in the systemic circulation (A). Ejection of blood is produced by a reduction in both diameter and length of the chamber.
At rest, ventricular diastole typically lasts for approximately 0.5 second, which is about two-thirds of the cardiac cycle (B).
When the chamber of the left ventricle contracts, it twists forward and the apex taps against the chest wall; this can be palpated in the left fifth intercostal space as the ‘apex beat’ (C).
Both the right and left coronary arteries arise from the aorta immediately above the cusps of the aortic valve to receive blood from the left ventricle (D).
The papillary muscles of the left ventricle are connected to the cusps of the mitral valve by chordae tendineae, not to the aortic valves (E).
4. Regarding veins, which of the following are true:
  1. They contain a third of the circulating blood volume
  2. Their pressure/volume curve is initially very steep
  3. Blood enters the venules at a lower pressure than larger veins
  4. Central venous pressure is the pressure at the point where the venae cavae enter the right atrium
  5. Standing results in a decrease in the venous pressure of veins above and below the level of the heart
4.
  1. False
  2. True
    22
  3. False
  4. True
  5. False
Peripheral veins and venules are thin-walled, voluminous vessels that hold roughly two-thirds of the circulating volume (A). Venules have a diameter of 0.01–0.2 mm with veins having a diameter of 0.2–5.0 mm, together collectively being known as capacitance vessels. In contrast, arterioles and small arteries are known as resistance vessels as they are responsive to autonomic supply and can constrict and dilate to control blood flow.
The effect of pressure on venous volume is very steep between 0 and 10 mmHg, due to the easily distensible walls (B). This allows the volume to increase relatively easily per unit rise in pressure.
Blood enters the venules at a pressure of about 12–20 mmHg, but by the time it reaches larger veins, such as the axillary or femoral vein, the pressure falls to about 10 mmHg (C).
Central venous pressure is usually measured directly by a catheter placed in the superior vena cava, via the internal jugular or subclavian veins (D).
On standing the pressure increases in any blood vessel below the level of the heart and decreases above the heart; this is due to gravity acting on the column of fluid between the heart and the vessel (E).
5. Regarding physiological dead space, which of the following are true:
  1. It decreases under general anaesthesia
  2. It increases with hypovolaemia
  3. It is the same as anatomical dead space
  4. It requires knowledge of the Peco2 and Pao2 to be calculated
  5. Alveolar dead space is normally 150 mL
5.
  1. False
  2. True
  3. False
  4. False
  5. False
Physiological dead space increases under general anaesthesia and in hypovolaemic patients as shown in Table 1.1 (A, B).
Table 1.1   The causes of increased and decreased dead space
Anatomical dead space
Alveolar dead space
Increased
Decreased
Increased
Neck extension
Neck flexion
Pulmonary embolism
Jaw protrusion
Low tidal volumes
Pulmonary disease
Increased tidal volumes
General anaesthesia
Hypovolaemia
Neonates and the elderly
Intubation
Hypotension
Bronchodilation
Tracheostomy
General anaesthesia
Anticholinergics
5-hydroxytryptamine
Intermittent positive pressure ventilation
Catecholamines
Histamine
Positive end-expiratory pressure
23Physiological dead space is the sum of anatomical dead space (normally 150 mL) and alveolar dead space (normally 0 mL); thus, it is not the same as the anatomical dead space (C, E). It is normally approximately 30% of tidal volume (i.e. Vd/Vt = 0.3) and is calculated by the Bohr equation (see Answer 3.61). The Bohr equation requires the knowledge of the arterial partial pressure of CO2 (Paco2) and the expired partial pressure of CO2 (Peco2) (D):
There are a number of factors that affect the anatomical dead space and alveolar dead space as classified in Table 1.1.
6. Regarding pulmonary blood flow, which of the following is true:
  1. It is greatest in the uppermost lung when a subject is in the lateral position
  2. Regional perfusion differences can be explained by the effects of hydrostatic pressure
  3. Measurement of pulmonary blood flow requires knowledge of arterial carbon dioxide content
  4. In West zone 1, alveolar pressure exceeds pulmonary arterial pressure
  5. Hypoxic areas of lung have increased perfusion
6.
  1. False
  2. True
  3. False
  4. True
  5. False
Pulmonary blood flow is greatest in the dependent areas of the lung. Thus, when standing, perfusion is greatest at the bases, and in the lateral position perfusion is greatest in the lower lung (A). This is due to the effect of hydrostatic pressure whereby gravity increases perfusion pressure in the lungs by 1 cmH2O for every centimetre in height below the level of the heart (B).
The Fick principle may be utilised in the measurement of pulmonary blood flow. This states that O2 consumption per unit time (V̇o2) is equal to the amount of O2 taken up by the blood in the lungs per unit time (i.e. the blood flow times the arterial oxygen content (Cao2) – venous oxygen content (CV̄o2) difference) (C).
Therefore:
The lung is divided into three zones defined by the relationship between pulmonary arterial (Pa), pulmonary venous (Pv) and alveolar pressures (Pa). In zone 1, Pa>Pa>Pv, thus capillaries are closed and no flow occurs (D). This does not occur normally but in areas where it does occur it is defined as an area of alveolar dead space. Zone 2 may be a region in the middle of the lung, where Pa>Pa>Pv and blood flow is determined by the arterial-alveolar pressure difference. Finally, zone 3 is often at the bottom of the lung, where Pa>Pv>Pa and blood flow is determined by the arterial-venous pressure difference.
Hypoxic areas of the lung undergo vasoconstriction (hypoxic pulmonary vasoconstriction, HPV) to prevent blood flow to poorly ventilated alveoli, thus reducing shunt (E).
7. The respiratory centre receives afferents from:
  1. Pulmonary stretch receptors
  2. Juxtaglomerular capillary receptors
  3. Dorsal medullary neurones
  4. Baroreceptors
  5. Carotid sinus receptors
247.
  1. True
  2. False
  3. False
  4. True
  5. False
The control of breathing involves sensors with afferent fibres to central controllers that then feed effector targets. The location and function of each of these areas can be summarised as follows:
  • Central controllers
    • The respiratory centre is responsible for the rhythmic inspiratory and expiratory patterns and includes:
      • The medullary respiratory centre
      • The apneustic centre in the lower pons
      • The pneumotaxic centre in the upper pons
  • Sensors
    • Central chemoreceptors
      • On the ventral medullary surface (C)
      • Stimulated by a reduction in CSF pH (i.e. an increase in H+ ion concentration) caused by metabolic acidosis or an increased Pco2
      • Not affected by Po2
    • Peripheral chemoreceptors
      • Aortic bodies giving vagal afferents and carotid bodies giving glossopharyngeal nerve afferents (E)
      • Stimulated by elevated Pco2 in a linear fashion, increase in H+ ions and a reduction in Po2 below 8–10 kPa
    • Lung receptors
      • Pulmonary stretch receptors (A)
      • Juxtapulmonary capillary (J) receptors (B)
      • Irritant receptors
      • Bronchial C fibres
    • Other receptors
      • Nose and upper airway receptors
      • Arterial baroreceptors (D)
      • Joint and muscle receptors
      • Higher centres: pain, temperature, anxiety
  • Effectors
    • Respiratory muscles
8. The following substances undergo metabolism in the lungs:
  1. Angiotensin II
  2. Acetylcholine
  3. Noradrenaline
  4. Leukotrienes
  5. Prostaglandin A2
8.
  1. False
  2. False
  3. True
  4. True
  5. False
25Other than the heart, the lungs are the only other organ that receives the whole circulating blood volume. Therefore, the lungs are involved in a number of metabolic processes (see Table 1.2) including noradrenaline and leukotriene metabolism (C, D).
Table 1.2   Metabolic functions of the lungs
Removal/inactivation
No effect
Conversion
Bradykinin - 80%
Angiotensin II
Angiotensin I to II by angiotensin-converting enzyme (ACE)
5-hydroxytryptamine (5-HT; serotonin)
Vasopressin (antidiuretic hormone, ADH)
Noradrenaline
Dopamine
Prostaglandins E2 and F
Histamine
Leukotrienes
Prostaglandin A2 (E)
Although angiotensin II is formed in the lungs by the action of angiotensin-converting enzyme (ACE) on angiotensin I, it is metabolised to angiotensin III in red blood cells and vascular endothelium (A). It is therefore not metabolised in the lungs.
Acetylcholine is hydrolysed in the plasma and post-synaptic membranes, but not directly in the lungs (B).
Other functions of the lung include phospholipid synthesis (e.g. surfactant), involvement in immune and coagulant function, and acting as a reservoir of blood (between 500 and 900 mL at any given moment). This latter feature explains the decreased blood pressure on release of an elevated intrathoracic pressure, as seen during the Valsalva manoeuvre (see Question 2.89).
9. The following are true regarding the kidneys:
  1. They receive approximately 25% of the cardiac output
  2. Each kidney has two renal arteries
  3. Most of the renal blood flow is to the medulla
  4. The vasa recta arise from the afferent arterioles
  5. The renal arteries divide into interlobular and arcuate arteries
9.
  1. True
  2. False
  3. False
  4. False
  5. True
Despite the kidneys being small (weighing approximately 130 g each), together they receive nearly a quarter of cardiac output, or 500 mL/min/100 g (about 1.2 L/min) (A).
Blood is supplied to the kidneys via the renal artery, which is a branch of the aorta. There are typically two renal veins and one renal artery per kidney (B).
The distribution of blood flow to different regions varies, with 90% of renal blood flow to the cortex and only 10% to the medulla (C). Despite this the inner medulla still receives a higher blood flow per gram of tissue than most other organs.
The vasa recta arise from the inner cortical efferent arterioles, which also supply the peritubular capillaries (D). The vasa recta provide the only blood supply to the renal medulla.
26The high-pressure renal arteries first divide into several interlobar arteries and these in turn divide into arcuate arteries, which give off the interlobular arteries at right angles (see Figure 1.3) (E).
zoom view
Figure 1.3: Divisions of renal arterial supply.
10. Functions of the kidney include:
  1. Direct maintenance of intracellular fluid volume
  2. Maintenance of extracellular potassium concentration
  3. Gluconeogenesis
  4. Production and secretion of angiotensinogen
  5. Production of prostaglandins
10.
  1. False
  2. True
  3. True
  4. False
  5. True
The kidneys maintain the constancy of the extracellular fluid volume (ECFV) and the extracellular osmolality by balancing the intake and excretion of sodium and water (A). The osmolality of the ECFV is strictly controlled to avoid large fluctuations in volume and osmolality within the intracellular compartment, as this is in osmotic equilibrium with the ECFV. They do not directly maintain intracellular fluid volume.
The kidneys maintain the concentration of extracellular potassium constant as well as the pH of the blood, by adjusting the excretion of hydrogen ions and bicarbonate (B).
Gluconeogenesis is the process of formation of glucose from substrate, including lactate and pyruvate, occurring predominantly in the liver with a smaller contribution from the kidneys (C). Other metabolic functions of the kidneys include arginine formation and peptide hydrolysis.
The kidneys produce the proteolytic enzyme renin from the juxtaglomerular apparatus, which it releases into the bloodstream. Renin acts on angiotensinogen, which is produced in the liver, to form angiotensin I (D). This is then converted to the highly active angiotensin II by cleaving two amino acids. Angiotensin II causes thirst and stimulates the release of aldosterone.
27In addition to the regulation of the ECFV and ion concentration, the kidneys are a source of hormones, such as prostaglandins, angiotensin II and erythropoietin (E).
11. Glomerular filtration rate:
  1. Can be calculated using inulin and creatinine as indicators
  2. Can be calculated using the formula urine concentration (U) × plasma concentration/urinary flow rate (V)
  3. Is about 125 L/min
  4. Is the same as the clearance of inulin
  5. Ceases when the mean systemic arterial pressure falls below 60 mmHg
11.
  1. True
  2. False
  3. False
  4. True
  5. False
Both inulin and creatinine have the suitable properties required to measure glomerular filtration rate (GFR) (A). Inulin is a carbohydrate which is infused into the bloodstream, whereas creatinine is already present in the blood as a breakdown product of muscle metabolism.
The formula for calculating GFR is UV/P, where U is the urine concentration of the indicator, V is the urine flow rate and P is the plasma concentration of the indicator (B). The GFR is normally approximately 125 mL/min for a normal adult (C).
Inulin, once injected, is completely filtered to enter the tubule and all of it enters the urine; therefore, the rate of filtration of inulin will equal the rate of excretion (D).
GFR is constant at mean systemic arterial pressures above 80 mmHg, so the filtration pressure is kept constant in this range by altering the pre-arteriolar and arteriolar resistance. Filtration significantly reduces but does not cease below a mean pressure of 60 mmHg (E).
12. Regarding potassium balance, which of the following is true:
  1. Approximately 60% is excreted in the urine and 40% in the faeces
  2. Approximately 99% is stored intracellularly
  3. Extracellular potassium is important for the regulation of potassium balance
  4. Acute regulation is controlled by aldosterone
  5. Most of the potassium filtered by the kidney is reabsorbed
12.
  1. False
  2. True
  3. True
  4. False
  5. True
About 50–150 mmol of potassium (K+) is resorbed daily in which about 90% of the K+ is excreted in the urine and about 10% in the faeces (A).
About 98–99% of the total body K+ stores are intracellular, mostly in muscle cells, but also in the liver and the erythrocytes (B).
Extracellular K+ only accounts for about 1% of the total body store; one of the main functions of this extracellular store is to mediate the regulation of the whole K+ balance of the body (C).
The acute regulation of extracellular K+ is achieved largely by the release of insulin, which promotes the uptake of K+ from the extracellular compartment in to the intracellular compartment. Aldosterone, adrenaline and alkalosis all promote the cellular uptake of K+, but are not as rapid in their management as insulin (D).
About 70% of K+ is reabsorbed by the end of the proximal convoluted tubule, about 20% at the loop of Henle and the rest is reabsorbed in the distal convoluted tubule (E).
13. Regarding cerebral blood flow (CBF), which of the following is true:
  1. The majority is in the white matter
  2. It is approximately 15% of cardiac output
  3. Two thirds are from the vertebral arteries
  4. Autoregulation maintains a constant CBF between a mean arterial pressure of 50 and 150 mmHg
  5. It is calculated using Fick's law
2813.
  1. False
  2. True
  3. False
  4. True
  5. False
Cerebral blood flow (CBF) is approximately 15% of cardiac output, 700 mL/min (50 mL/100 g/min) of which the grey matter receives the majority (A, B). The blood supply is two thirds from the internal carotid arteries and one third from the two vertebral arteries (C). The two systems merge at the anterior and posterior communicating arteries to form the circle of Willis. CBF remains constant between mean arterial pressures (MAPs) of 50–150 mmHg – above this range there is a sharp increase in CBF, with a decrease in CBF below this range (D).
CBF can be measured using the Fick principle. This states that the uptake/release of a substance, e.g. O2 (V̇o2), by an organ is the product of the blood flow (Q̇) through that organ and the arteriovenous difference in content (Cao2–CV̄o2) (see Question 1.6). This is applied using the Kety-Schmidt technique where 10% N2O is inhaled for 10–15 minutes, and the jugular venous concentration is measured and assumed to be the same as the brain concentration. Fick's law, however, describes the rate of diffusion across a membrane being proportional to the concentration gradient (E).
14. Regarding cerebrospinal fluid (CSF), which of the following is true:
  1. 500 mL is produced every 24 hours
  2. At greater intracranial pressures, more CSF is produced
  3. Fluid passes from the 3rd ventricle via the foramen of Magendie
  4. Potassium concentrations are normally 2.5–3.5 mmol/L
  5. There is a lower concentration of protein in CSF than in the plasma
14.
  1. True
  2. False
  3. False
  4. True
  5. True
Cerebral spinal fluid (CSF) is a clear, colourless fluid surrounding the brain that protects it from traumatic damage and helps regulate intracranial pressure (ICP).
It has characteristics and properties as follows:
  • Production
    • Total volume is 100–150 mL (10% of intracranial volume), produced at a rate of 0.3 mL/min by choroid plexuses in lateral, third and fourth ventricles
    • 500 mL is produced a day (A)
    • Formed by plasma filtration and secretion
    • Production of CSF is independent of ICP (B)
  • Circulation
    • Lateral ventricles to the third ventricle via foramen of Monro
    • Third to fourth ventricle via aqueduct of Sylvius
    • Fourth ventricle down the spinal cord or over cerebral hemispheres via midline foramen of Magendie or lateral foramen of Luschka (C)
    • Absorbed from dural venous sinuses via arachnoid villi
  • Composition
    • Normal CSF reference values in Table 1.3 (D)
      29
    • CSF proteins are only 1% of plasma levels, while calcium and glucose levels are 50–60% of plasma levels (E)
    • Chloride and magnesium levels, however, are greater in the CSF than the plasma
Table 1.3   Reference values for cerebrospinal fluid
Constituent
Concentration
Sodium
135–145 mmol/L
Potassium
2.5–3.5 mmol/L
Chloride
115–125 mmol/L
Calcium
1–1.5 mmol/L
Magnesium
1.2–1.5 mmol/L
Glucose
2.7–4.2 mmol/L
Urea
1.5–6.0 mmol/L
Lymphocytes
0–5 × 106/L
Protein
0.2–0.4 g/L
15. Parasympathetic stimulation causes:
  1. Lacrimation
  2. Increased insulin secretion
  3. Vasoconstriction
  4. Tachycardia
  5. Bronchodilation
15.
  1. True
  2. True
  3. False
  4. False
  5. False
Parasympathetic stimulation is responsible for the ‘rest and digest’ actions in the autonomic nervous system, of which the Vagus nerve carries approximately three quarters of the fibres. Vasoconstriction, tachycardia and bronchodilation are all sympathetically mediated effects, while lacrimation and insulin production are parasympathetic effects.
The following are effects of parasympathetic stimulation:
  • Ophthalmic: pupillary and ciliary constriction; lacrimation (A)
  • Cardiovascular: bradycardia, reduced contractility; vasodilation in skeletal muscle, coronary, pulmonary, renal and viscera (C, D)
  • Pulmonary: bronchoconstriction; increased secretions (E)
  • Gastrointestinal: increased motility and secretions; sphincteric relaxation
  • Metabolic: increased secretion of insulin and glucagon (B)
  • Genitourinary: detrusor muscle contraction, sphincteric relaxation; penile erection
16. Regarding muscle spindles, which of the of following are true:
  1. They are involved in maintenance of posture
  2. Efferent control is via γ-motor neurones
  3. Sensory supply involves only type II fibres
  4. They sense muscle tension
  5. The withdrawal reflex is polysynaptic
16.
  1. True
  2. True
  3. False
  4. False
  5. True
30Muscle spindles are formed from intrafusal muscle fibres that respond to changes in length, rather than muscle tension, whereby contraction of muscles causes shortening of muscle spindles, an important mechanism in maintaining posture (A, D).
When muscle spindles are passively stretched, they transmit impulses directly to efferent γ-motor neurones via either type Ia or type II fibres (B, C). This is the monosynaptic stretch reflex. However, γ-motor neurone activity can be modulated by descending spinal cord pathways, which may alter the resting tone and sensitivity of muscles. These features make muscle spindles suitable for sensing and modifying posture. Golgi tendon organs are responsible for sensing muscle tension.
The withdrawal reflex is a polysynaptic reflex, as there are interneurones between sensory signals and motor elements of the reflex (E).
17. Regarding daily nutrition, which of the following are true:
  1. Energy requirements are mainly obtained from fats, minerals and vitamins
  2. The largest energy contribution as a percentage of intake is from dietary fats
  3. Per unit, proteins have the highest caloric value
  4. Essential amino acids are only synthesised in small quantities
  5. Fat-soluble vitamins include vitamins B, C, E and K
17.
  1. False
  2. False
  3. False
  4. True
  5. False
Energy requirements are primarily provided by proteins, fats and carbohydrates (A). Vitamins and minerals are required in small quantities but do not provide significant energy supplies.
The energy contribution of carbohydrates is about 60% of the total, while fats provide about 25–30% of the energy (B). The remaining energy is provided by proteins.
One gram of fat provides about 40 kJ of energy, while 1 g of protein provides about 17 kJ as does 1 g of carbohydrates (C).
Essential amino acids are those that cannot be synthesised by humans or only in very small amounts; therefore, they need to be part of a healthy diet (D). The essential amino acids include valine, leucine, isoleucine, threonine, methionine, phenylalanine, tryptophan and lysine.
Fats are largely superfluous in the diet, provided that there is a supply of the essential fatty acids, such as linoleic acid, and the fat-soluble vitamins, such as A, D, E and K (E).
18. Swallowing:
  1. Is a voluntary process
  2. Involves the soft palate being pulled up
  3. Causes the larynx to be pulled up by the pharyngeal muscles
  4. Takes about 3 seconds from start to finish
  5. Is initiated at the tonsillar pillars
18.
  1. False
  2. True
  3. False
  4. False
  5. True
Swallowing is an involuntary reflex (A). It is preceded by the voluntary act of collecting food on the tongue and propelling the bolus into the pharynx by raising the tongue.
31As the bolus of food enters the pharynx, it stimulates the swallowing receptors. This triggers the swallowing reflex, which is a series of autonomic pharyngeal muscular contractions.
First, the soft palate is pulled upwards to close the posterior nares, thus preventing food from entering the nasal cavities (B).
The palatopharyngeal folds are pulled medially so that well-masticated food passes posteriorly into the pharynx, and larger objects are impeded.
The vocal cords are strongly approximated, the larynx is pulled upwards and anteriorly by the neck muscles, not the pharyngeal muscles, and the epiglottis swings backwards over the opening of the larynx (C). This all prevents food from entering the larynx, while enlarging the opening of the oesophagus.
At the same time the upper oesophageal sphincter relaxes, allowing food to easily pass into the upper oesophagus.
The whole process, which includes the trachea closing, the oesophagus opening and propelling of the food bolus from the pharynx to the oesophagus, takes about 1–2 seconds (D).
The most sensitive tactile areas of the pharynx for the initiation of swallowing lie around the pharyngeal opening, with the tonsillar pillars having the greatest sensitivity (E).
19. Regarding hormone production, which of the following are true:
  1. The kidneys produce calcitriol
  2. The thyroid gland produces calcitonin
  3. The stomach produces glucagon
  4. The liver produces cholecystokinin
  5. The anterior pituitary gland produces oxytocin
19.
  1. True
  2. True
  3. False
  4. False
  5. False
A hormone is a regulatory substance released by a single or group of cells or organ that has an effect at a distant site. Common hormones can be seen in Table 1.4.
Calcitriol is the active form of vitamin D that acts to increase plasma calcium levels and is produced in cells of the proximal tubule of the renal nephron (A).
Calcitonin is generated by the thyroid parafollicular (C) cells to reduce serum calcium concentrations (B).
Glucagon is a peptide hormone synthesised in the α-cells of the pancreatic islets of Langerhans that increase blood glucose concentrations (C).
Cholecystokinin is a peptide hormone that is produced in the small intestinal and duodenal mucosa in response to fat and protein in chime (D). It increases gastric transit time to allow further digestion of fats.
Both oxytocin and vasopressin (antidiuretic hormone, ADH) are released from the posterior pituitary but are produced in the hypothalamus (E).
32
Table 1.4   Sites of hormone production
Organ
Hormone
Adrenal cortex
Aldosterone, cortisol, androgens
Kidneys
Calcitriol
Testes
Testosterone
Ovaries
Oestrogen, progesterone
Thyroid
T3 (triiodothyronine), T4 (thyroxine), calcitonin
Adrenal medulla
Adrenaline, noradrenaline
Mast cells
Histamine
Pancreas
Insulin, glucagon
Parathyroids
Parathyroid hormone
Stomach and small intestine
Gastrin, secretin, cholecystokinin, gastric inhibitory peptide
Anterior pituitary
Human growth hormone, prolactin, adrenocorticotropic hormone, melanocyte stimulating hormone, thryoid stimulating hormone, follicle stimulating hormone, luteinising hormone
Hypothalamus
Oxytocin, antidiuretic hormone
20. Regarding haemoglobin, which of the following are true:
  1. Adult haemoglobin is composed of two α- and two γ-subunits
  2. It contains iron in its ferrous state
  3. It contains four iron atoms
  4. It can bind eight molecules of oxygen to each haemoglobin chain
  5. Cooperative binding is due to the Bohr effect
20.
  1. False
  2. True
  3. True
  4. False
  5. False
Haemoglobin is a molecule that is designed to increase the oxygen carrying capacity of blood >50 times, and is made of four polypeptide globin chains. The adult circulation contains HbA (two α-chains and two β-chains) as well as 2–3% HbA2 (two α-chains and two δ-chains) (A). Fetal haemoglobin contains two α-chains and two γ chains.
Each globin chain has a porphyrin-derived haem group with a central iron atom in its ferrous state (Fe2+) able to carry one O2 molecule; therefore, up to four molecules of O2 can be carried per haemoglobin (B, C, D). It is methaemoglobin that carries iron atoms in the ferric state (Fe3+). Haemoglobin has a number of characteristic effects that it demonstrates:
  • Cooperativity: binding of one O2 molecule breaks non-covalent salt links between globin chains that increase affinity for further O2 molecules to bind. This effect is more pronounced with the second and third molecule of O2 that bind and is the underlying cause of the sigmoid-shaped oxyhaemoglobin dissociation curve
  • Bohr effect: higher Pco2 reduces the affinity of haemoglobin for O2 (E)
  • Haldane effect: increases affinity of deoxygenated haemoglobin for CO2
21. Bioavailability:
  1. Is the fraction of administered substance that enters the systemic circulation
  2. Is higher for orally administered drugs than rectally administered drugs
  3. May be reduced by increasing first-pass metabolism
  4. Increases in the presence of hepatic enzyme inducers
  5. Is 100% for intravenous ketamine
3321.
  1. True
  2. False
  3. True
  4. False
  5. True
Bioavailability is the fraction of a drug dose that reaches the systemic circulation compared with a standard, usually intravenous, route of administration (A). Thus, any drug administered via the intravenous route has a bioavailability of 100% (E). By plotting a plasma concentration over time curve for the same dose of oral and intravenous drug administration (see Figure 1.4), the bioavailability can be calculated by:
where AUC is area under the curve and IV is intravenous.
zoom view
Figure 1.4: Bioavailability of a drug from oral and intravenous administration.
Bioavailability can be affected by a number of factors, including:
  • First-pass metabolism: drugs absorbed from the gastrointestinal tract enter the portal vein and thence to the liver, where they undergo metabolism prior to reaching the systemic circulation. This is minimised with sublingual and rectal routes of administration which have a higher bioavailability than the oral route (B, C)
  • Hepatic enzymes: induction of hepatic enzymes increases first-pass metabolism thereby reducing bioavailability, while hepatic enzyme inhibition reduces metabolism, thus increasing bioavailability (D)
  • Physicochemical properties: small or liquid drugs are more readily absorbed when compared with large drug preparations. Interaction with other orally delivered substances may reduce absorption, thus reducing oral bioavailability
    34
  • Gastrointestinal properties: any cause for delayed gastric emptying or malabsorption may reduce oral bioavailability
22. The following drugs can be administered transdermally:
  1. Procaine
  2. Glyceryl trinitrate
  3. Clonidine
  4. Alfentanil
  5. Diclofenac
22.
  1. False
  2. True
  3. True
  4. False
  5. True
Transdermal drug delivery depends on drug factors and patient factors as shown in Table 1.5.
Table 1.5   Factors affecting transdermal drug delivery
Drug factors
Patient factors
Lipophilic and hydrophilic
Good regional blood flow
Short half-life
Intact skin
Low melting point
Low molecular weight (<500 Da)
High potency
Unionised
Prilocaine, not procaine, is a component of the topical local anaesthetic cream EMLA (eutectic mixture of local anaesthetic) and can be delivered transdermally (A).
Glyceryl trinitrate is a vasodilator that can be administered as transdermal patches (B).
Clonidine is a versatile α2-adrenoceptor agonist that can be administered orally, transdermally, intramuscularly, intravenously or intrathecally (C).
Alfentanil is a synthetic piperidine opioid receptor agonist that is only delivered via the intravenous route (D).
Diclofenac is a non-steroidal anti-inflammatory drug that can be administered orally, intravenously, rectally or transdermally (E).
Commonly used transdermally applied drugs include:
  • Fentanyl
  • Buprenorphine
  • Diclofenac
  • EMLA (2.5% lidocaine, 2.5% prilocaine)
  • Amethocaine
23. The volume of distribution:
  1. Is greater for fentanyl than morphine
  2. Of propofol is greater than total body water
  3. Is greater than 1 L/kg for most non-depolarising neuromuscular blocking drugs
  4. Is higher for highly protein-bound drugs
  5. Is the apparent volume that a drug disperses into to produce observed plasma concentrations
23.
  1. True
  2. True
  3. False
    35
  4. True
  5. True
The volume of distribution (Vd) is the theoretical volume in which a drug would have to disperse in order to achieve observed plasma concentrations (E). Because it is only a theoretical volume, it may potentially be larger than total body water. Highly protein bound drugs also have a high volume of distribution, while highly polar or charged drugs do not cross membranes easily and stay within the central compartment, leading to small volumes of distribution (D).
The Vd of fentanyl is 4.0 L/kg, while that of morphine is 3.5 L/kg, reflecting its lipid solubility that is 600 times greater than that of morphine (A).
As propofol is highly lipid soluble, it has a volume of distribution of 4 L/kg, which is greater than total body water (B).
Non-depolarising neuromuscular blocking drugs all have volumes of distribution of <0.30 L/kg because they are highly polar drugs (C).
24. The following drugs are excreted in the urine predominantly unchanged:
  1. Diclofenac
  2. Epinephrine
  3. Lithium
  4. Ephedrine
  5. Digoxin
24.
  1. False
  2. False
  3. True
  4. True
  5. True
Diclofenac is significantly metabolised in the liver by phase I and II metabolism, while exogenous epinephrine (adrenaline) is metabolised in the liver by catechol-O-methyl transferase to metadrenaline and metnoradrenaline (A, B).
Lithium, ephedrine and digoxin are all renally excreted without undergoing a significant degree of metabolism (C, D, E).
The following drugs are excreted predominantly unchanged in the urine and can be remembered by the mnemonic ACED LMNOP G:
  • Aminoglycosides
  • Cephalosporins
  • Ephedrine
  • Digoxin
  • Lithium
  • Milrinone/Mannitol
  • Neostigmine
  • Oxytetracycline
  • Penicillins
Drugs that are excreted unchanged in the urine are likely to need dose adjustment in renal failure.
25. The ideal intravenous anaesthetic agent would have the following properties:
  1. High lipid solubility
  2. Lipid-soluble formulation
  3. Long half-life
  4. Analgesic at sub-anaesthetic concentrations
  5. Pre-prepared solution
3625.
  1. True
  2. False
  3. False
  4. True
  5. True
The ideal intravenous anaesthetic agent would have the following properties:
  • High lipid solubility (A)
  • Water-soluble formulation (B)
  • Short half-life (C)
  • Analgesic properties at low doses (D)
  • Pre-prepared solution (E)
  • Rapid recovery, with no accumulation after infusion
  • Minimal cardiovascular and respiratory depression
  • Antiemetic properties
  • Painless on injection
  • No interaction with other drugs
  • No histamine release
  • Long shelf life at room temperature
  • No histamine release
  • No hypersensitivity reactions
26. Thiopentone:
  1. Is formulated as a sulphur salt
  2. Forms a neutral solution when dissolved in water
  3. Is stored in nitrogen
  4. Once administered, 80% is immediately available at pH 7.4
  5. Is safe to use in patients with porphyria
26.
  1. False
  2. False
  3. True
  4. False
  5. True
Thiopentone is the sulphur analogue of the oxybarbiturate pentobarbitone. It is formulated as a sodium salt, sodium thiopentone and appears as a yellow powder in a glass vial (A).
Sodium thiopentone is a weak acid with a pKa value of 7.6. When dissolved in water, it forms an alkaline solution (B).
The free acid that is formed in solution is highly insoluble and would precipitate out of solution. To prevent the formation of the free acid, sodium thiopentone is stored in glass vials containing nitrogen, while sodium carbonate is added to react with water to produce hydroxide ions (C). This forms an alkaline solution, thus preventing the accumulation of hydrogen ions and therefore the undissociated acid.
At physiological pH, only 12% of the administered sodium thiopentone is actually available in the active form, which is non-protein bound and unionised (D). The rapid onset is due to the high lipid solubility of the drug and the fact that the brain receives a relatively large blood flow. Emergence after a single bolus dose is due to 37rapid redistribution into well-perfused tissues such as the liver followed by the skin and muscle.
Sodium thiopentone may precipitate an acute porphyric crisis and is absolutely contraindicated in patients suffering from porphyria (E).
27. Xenon:
  1. Is a colourless gas with a pungent odour
  2. Is produced by fractional distillation of air
  3. Increases cardiac output by sympathetic stimulation
  4. Has a slower onset and offset time than desflurane
  5. Has a higher density and viscosity than nitrous oxide
27.
  1. False
  2. True
  3. False
  4. False
  5. True
Xenon is a non-toxic inert odourless gas, making up only 0.0000087% of the atmosphere and is produced by the fractional distillation of air (A, B).
It does not alter myocardial contractility and may cause a small reduction in heart rate and hence cardiac output (C). Xenon does not sensitise the myocardium to catecholamines.
Xenon has a minimum alveolar concentration of 71% and has a very low blood:gas partition coefficient of 0.14, as a result its onset and offset of action are faster than desflurane, sevoflurane and nitrous oxide (D).
Xenon is three times denser and nearly twice as viscous as nitrous oxide, although the Fink effect of diffusion hypoxia is not a feature of xenon (E).
Xenon has a molecular weight of 131.2, a boiling point of -108°C, is non-flammable and does not support combustion. It undergoes virtually no metabolism in the body.
28. Tachyphylaxis:
  1. Is synonymous with tolerance
  2. Occurs when larger doses are required to achieve a similar response
  3. Is seen with amphetamine
  4. May be due to a change in receptor structure
  5. Takes place over a short period of time
28.
  1. False
  2. False
  3. True
  4. True
  5. True
Tolerance occurs when larger doses of a drug are required to achieve a given response, often due to altered receptor sensitivity such as in chronic opioid use (A).
Tachyphylaxis, however, is the reversible, acute decrease in response to a given dose of drug after repeated administration and is synonymous with desensitisation, which takes place over a longer period of time (B, E). The underlying mechanisms include:
  • Change in receptor structure (D)
  • Loss of receptors
  • Reduction in mediators
  • Increased breakdown of drug
  • Physiological adaptation
Examples include ephedrine and amphetamine depleting amine stores (C).
29. The following local anaesthetics are esters:
  1. Prilocaine
  2. Etidocaine
  3. Amethocaine
  4. Cocaine
  5. Ropivacaine
3829.
  1. False
  2. False
  3. True
  4. True
  5. False
Local anaesthetics can be classified as esters or amides depending on the intermediate chains (see Figure 1.5). Esters contain the —CO.O— linkage and undergo hydrolysis in the plasma. They include amethocaine, cocaine and procaine (C, D). Amides contain the amide (—NH.CO—) linkage chain and undergo hepatic metabolism. All amides contain an ‘i’ followed by the suffix ‘caine’, including bupivacaine, etidocaine, lignocaine, prilocaine and ropivacaine (A, B, E).
zoom view
Figure 1.5: Chemical structure of ester and amide classes of local anaesthetic.
30. Systemic vascular absorption of local anaesthetic is greater than epidural administration when given:
  1. To the brachial plexus
  2. Caudally
  3. Subcutaneously
  4. To the femoral nerve
  5. Intercostally
30.
  1. False
  2. True
  3. False
  4. False
  5. True
Systemic absorption of local anaesthetics is greater than epidural administration when given via the caudal injection or intercostal injection (B, E). Systemic concentrations are lowest when administered subcutaneously, to plexuses or to large distal nerves (see Figure 1.6).
Systemic vascular absorption is dependent on:
  • Local anaesthetic characteristics
  • Use of vasoconstrictor
  • Location of injection
39
zoom view
Figure 1.6: Systemic absorption of local anaesthetics.
31. The following drugs have a vasodilator action:
  1. Prilocaine
  2. Bendroflumethiazide
  3. Diazoxide
  4. Tramadol
  5. Lidocaine
31.
  1. True
  2. True
  3. True
  4. False
  5. True
Most local anaesthetic agents, including prilocaine and lidocaine, cause vasodilatation at low concentrations with vasoconstriction at higher concentrations (A, E). Cocaine however is a pure vasoconstrictor. Bendroflumethiazide has an antihypertensive effect by inhibiting Na+/Cl- co-transport in the distal convoluted tubule, leading to a diuresis, but also exerts a direct vasodilator effect (B).
Diazoxide is structurally similar to bendroflumethiazide and acts as a vasodilator by modulating cyclic adenosine monophosphate levels in arteriolar smooth muscle cells (C).
Tramadol has no significant vascular effects when administered via the intravenous route (D).
32. Antiarrhythmic drugs:
  1. Are classified according to their site of action
  2. Are classified according to their effects on the action potential
  3. May belong to more than one class in the Vaughan Williams classification
  4. Belong to class I as they slow phase 0 of the action potential
  5. Such as adenosine belong to class III
32.
  1. True
  2. True
  3. True
  4. True
  5. False
Antiarrhythmic drugs can be categorised according to the cardiac tissue that they affect (see Table 1.6) (A). This may be useful when specific arrhythmias need to be treated.
The Vaughan Williams classification is based on the electrophysiological action on isolated cardiac fibres (B). There are four classes in this classification, more recently a 5th class has been added, as shown in Table 1.7.
Some drugs have more than one action and can belong to more than one class, but they all have a dominant effect that allows them to be classed accordingly (C).
Class I antiarrhthymic drugs have local anaesthetic properties that exhibit membrane stabilising activity and affect conduction, refractoriness and the action potential (D). They are further subdivided into Ia, Ib and Ic, but they all slow phase 0 of the action potential to different degrees.
40
Table 1.6   Site of action of antiarrhythmic agents
Site of action
Drugs
Sinuatrial node
Beta-blockers
Class IV drugs
Digoxin
Atrioventricular node
Class Ic and IV drugs
Beta-blockers
Digoxin
Adenosine
Ventricles
Class I and III drugs
Atria
Class Ia and Ic drugs
Beta-blockers
Class III drugs
Accessory pathways
Class Ia and III drugs
Table 1.7   The modified Vaughan Williams classification
Class
Mechanism of action
Drugs
Ia
Block Na+ channels, prolong refractory period
Disopyramide, procainamide, quinidine
Ib
Block Na+ channels, shorten refractory period
Lidocaine, mexiletine, phenytoin
Ic
Block Na+ channels, no effect on refractory period
Flecainide, propafenone
II
β-adrenoceptor blockade
Atenolol, propranolol
III
K+ channel blockade
Amiodarone, bretylium, sotalol
IV
Ca2+ channel blockade
Diltiazem, verapamil
V*
Miscellaneous
Adenosine (E), digoxin, magnesium sulphate
* Antiarrhythmic agents that do not fit into any of the original classes described by Vaughan Williams in 1970 were later classified into class V.
Class Ic has the most potent sodium channel blocking action; therefore, it leads to reduction in phase 0 rapid depolarisation. The weakest sodium channel effects, thus the least effect on phase 0, are from class Ib drugs such as lidocaine.
33. Regarding vasoactive drugs, which of the following is true:
  1. Dopamine is a synthetic analogue of dobutamine
  2. Dopamine is a precursor of noradrenaline
  3. Dobutamine can cause hypotension
  4. Dobutamine acts on dopaminergic receptors
  5. Dopamine causes peripheral dilatation at high doses
33.
  1. False
  2. True
  3. True
  4. False
  5. False
41Dobutamine is a synthetic analogue of dopamine and is a β-adrenoceptor-stimulating agent (A). Dopamine is a catecholamine-like agent used for the treatment of severe heart failure and cardiogenic shock.
Dopamine is the precursor of noradrenaline and releases noradrenaline from stores in the nerve endings in the heart (B). The conversion of dopamine to noradrenaline occurs in granulated vesicles and is dependent on the enzyme dopamine β-hydroxylase.
Dobutamine is a derivative of isoprenaline with predominantly β1-adrenoceptor activity, thus making it a potent inotrope. It does, however, also have β2-stimulating effect, which causes peripheral vasodilatation. This may result in hypotension and a fall in the diastolic pressure with a reflex tachycardia (C).
Dobutamine acts on adrenergic receptors in the following order: β1 > β2 > α. It has no action on dopaminergic receptors (D).
Dopamine stimulates the heart by both β- and α-adrenergic responses and causes vasodilatation through its action on dopaminergic receptors. At high doses (>10 μg/kg/min), dopamine causes predominant α-adrenoceptor stimulation with peripheral vasoconstriction, increased peripheral vascular resistance and reduced renal blood flow (E).
34. The following are true regarding the mechanism of action of nitrates:
  1. They preferentially dilate large coronary arteries and arterioles
  2. They redistribute blood from epicardial to endocardial regions
  3. They have no effect on myocardial oxygen demand
  4. The vascular endothelium should be intact for effective vasodilation
  5. They are better arteriolar than venous dilators
34.
  1. True
  2. True
  3. False
  4. False
  5. False
In preference to small coronary arteries, nitrates dilate the large coronary arteries and arterioles with diameters >100 μm (A).
Nitrates redistribute blood flow along collateral channels and from epicardial to endocardial regions (B). They also relieve coronary spasm and dynamic stenosis, especially at epicardial sites, including the coronary arterial constriction induced by exercise.
Nitrates increase the venous capacitance, causing pooling of blood in the peripheral veins and thereby reducing venous return and ventricular volume. There is less mechanical stress on the myocardial wall and the myocardial oxygen demand is reduced. In addition to this, there is a fall in aortic systolic pressure, which further reduces myocardial oxygen demand (C).
The basic mechanism of nitrates (NO3-) is conversion to nitrites (NO2-) followed by enzyme-mediated release of unstable nitric oxide (NO). For some vasoactive agents an intact vascular endothelium is required, but nitrates vasodilate whether or not the endothelium is physically intact or functional (D). The NO produced then stimulates the enzyme guanylate cyclase to produce cyclic guanosine monophosphate (cyclic GMP). This reduces the levels of calcium in the vascular monocyte and therefore vasodilatation occurs. Nitrates are better venous than arteriolar dilators (E).
35. Suxamethonium:
  1. Is hydrolysed at the neuromuscular junction by cholinesterases
  2. Has active metabolites
  3. Is predominantly excreted in the urine
  4. Produces a phase I block on repeated administration
  5. Consists of two acetylcholine molecules joined by an ester link
4235.
  1. False
  2. True
  3. False
  4. False
  5. False
Suxamethonium is a depolarising muscle relaxant. It is hydrolysed by plasma cholinesterase to choline and succinylmonocholine, leaving only 20% to reach the neuromuscular junction (A). Succinylmonocholine is a weakly active ester of succinic acid with choline (B). Because of the rapid metabolism of suxamethonium, only 10% is excreted via the kidneys in the urine (C).
It produces a ‘phase I’ block in which there is a train-of-four ratio of >0.7, no fade to a 1 Hz stimulus with no post-tetanic facilitation. Repeated administration may produce a ‘phase II’ block similar to that of non-depolarising muscle relaxants, demonstrating a reduced train-of-four ratio of <0.7, fade on 1 Hz stimulation and post-tetanic facilitation present (D).
Suxamethonium has a structure of two acetylcholine molecules joined through acetyl groups (E). By binding to nicotinic acetylcholine receptors, suxamethonium depolarises the membrane; however, due to the lack of plasma cholinesterase at the neuromuscular junction, it remains attached to the acetylcholine receptors thereby preventing any further activation, and thus causing neuromuscular blockade.
36. Bisquaternary aminosteroid drugs include:
  1. Pancuronium
  2. Vecuronium
  3. Mivacurium
  4. Tubocurarine
  5. Rocuronium
36.
  1. True
  2. False
  3. False
  4. False
  5. False
The aminosteroid neuromuscular blocking drugs are large, bulky, polar molecules containing a steroid ring. They can be monoquaternary (containing a single N+—CH3group) such as vecuronium and rocuronium, or bisquaternary (containing two N+—CH3 groups) such as pancuronium. Of note, pancuronium is the bisquaternary analogue of vecuronium thereby making it more potent (A, B). Mivacurium is a benzylisoquinolinium, along with atracurium and cis-atracurium, while tubocurarine is a monoquaternary alkaloid (C, D, E).
37. The following are true of non-steroidal anti-inflammatory drugs:
  1. Paracetamol has anti-inflammatory properties
  2. Aspirin has antipyretic properties
  3. Diclofenac is 10% plasma protein bound
  4. Paracetamol is 90% plasma protein bound
  5. Ibuprofen has greater anti-inflammatory activity than analgesic activity
37.
  1. True
  2. True
  3. False
  4. False
  5. False
43Non-steroidal anti-inflammatory drugs (NSAIDs) are a group of diverse drugs with variable analgesic, antipyretic and anti-inflammatory properties. The extent and effect of common NSAIDs can be seen in Table 1.8.
Table 1.8   Features of non-steroidal anti-inflammatory drugs
Thus, paracetamol has moderate anti-inflammatory properties, while aspirin has significant antipyretic action (A, B). Most NSAIDs are highly protein bound except paracetamol that is only 10% protein bound (C, D). Ibuprofen is an equipotent analgesic and anti-inflammatory agent (E).
38. Morphine:
  1. Is a naturally occurring phenanthrene derivative
  2. Relaxes the gut sphincters and the sphincter of Oddi
  3. Causes nausea and vomiting via serotonergic (5HT) and muscarinic receptors
  4. Can precipitate chest wall rigidity
  5. Inhibits the release of adrenocorticotropic hormone and prolactin
38.
  1. True
  2. False
  3. False
  4. True
  5. True
Morphine is a naturally occurring opiate with a phenanthrene structure comprising three fused benzene rings (A). It causes constriction of the gut sphincters and spastic immobility of the bowel, resulting in constipation. It also causes contraction of the sphincter of Oddi, causing an increase in the intraluminal pressure within the biliary tree (B).
Morphine stimulates the chemoreceptor trigger zone via 5-HT3 and dopaminergic receptors. This explains the nausea-inducing effect of morphine (C). Muscarinic receptors are not activated by morphine to cause nausea and vomiting.
Morphine and other opioids can cause chest wall rigidity (D). This is thought to be as a result of dopaminergic and GABA pathways in the substantia nigra interacting with the opioid receptors.
Morphine inhibits the release of adrenocorticotropic hormone, prolactin and gonadotrophic hormones (E). It may also increase the secretion of vasopressin (antidiuretic hormone, ADH) that can cause water retention and hyponatraemia.
39. Benzodiazepines:
  1. Bind to the α-subunit of gamma-aminobutyric acid (GABA)A receptors
  2. Act via second-messenger systems
  3. Are highly plasma protein bound
  4. Always have active metabolites
  5. Include zopiclone
39.
  1. True
  2. False
    44
  3. True
  4. False
  5. False
Benzodiazepines are hypnotic drugs that act on the α-subunit of GABAA receptors to open the ligand-gated chloride (Cl-) channel causing hyperpolarisation of post-synaptic neuronal membranes predominantly (A). They therefore do not act via second messenger systems (B). Baclofen, however, acts via metabotropic GABABreceptors, activating second messenger systems.
Benzodiazepines are generally highly protein bound and usually have active metabolites; however, lorazepam is glucuronated to inactive metabolites (C, D).
Zopiclone, along with zolpidem and zaleplon, is non-benzodiazepine ‘Z-drug’ that lacks the benzodiazepine chemical structure but binds to GABAA receptors in a similar fashion (E).
40. These diuretics have the following unwanted effects:
  1. Furosemide and hypercalcaemia
  2. Bendroflumethiazide and hypocalcaemia
  3. Spironolactone and Conn's syndrome
  4. Mannitol and an increase in preload and cardiac output
  5. Bendroflumethiazide and leucopaenia and thrombocytopaenia
40.
  1. False
  2. False
  3. False
  4. True
  5. True
The biochemical effects of furosemide include hyponatraemia, hypokalaemia, hypochloraemic alkalosis, hypomagnesaemia and hyperuricaemia. These effects are also caused by the thiazide diuretics, but furosemide can also cause hypocalcaemia by increasing Ca2+ excretion (A).
Bendroflumethiazide causes hypercalcaemia as a result of reduced renal excretion (B).
Spironolactone is a competitive aldosterone antagonist; therefore, it not only reduces the excretion of K+ but also increases the excretion of Na+ and water. As a result of its action, it is used to treat ascites, Conn's syndrome (primary hyperaldosteronism) and nephrotic syndrome (C).
Mannitol is administered as an intravenous infusion and is usually given as a loading dose followed by an infusion. The loading dose for the treatment of raised intracranial pressure is about 1 g/kg, followed by an infusion of between 0.1 and 0.2 g/kg. As a result of these large infusion volumes, the initial circulating volume is increased which increases the preload and cardiac output (D).
The haematological effects of bendroflumethiazide include haemolytic anaemia, aplastic anaemia, leucopaenia, thrombocytopaenia and agranulocytosis (see Table 4.8) (E).
41. The following is true regarding electricity:
  1. Electrons can only pass from one atom to another under a potential difference
  2. Body fluids are generally bad conductors
  3. Insulators bind to their electrons more firmly then semiconductors
  4. Thermistors are insulators
  5. Diodes are semiconductors
41.
  1. False
  2. False
  3. True
    45
  4. False
  5. True
Electricity is the flow of electrons from one atom to another under the influence of a potential difference or under the influence of a changing magnetic field (A).
The electrons in a conductor are loosely bound; therefore, they can readily move when influenced by a potential difference. Bodily fluids are good conductors as they contain positive and negative ions, and these can move when a potential difference is applied to them, thereby conducting electricity (B).
The electrons in insulators are firmly bound to their atoms; therefore, they do not conduct electricity; thermistors are semiconductors (C, D).
The outer electrons in semiconductors are more firmly bound to their atoms than in conductors, but less firmly bound than in insulators. Diodes and transistors are semiconductors (E).
42. Surgical diathermy:
  1. Typically uses frequencies of 1 kHz
  2. Is more likely to cause ventricular fibrillation than a direct current
  3. Relies on two connections to the patient, even with monopolar diathermy
  4. Contains a capacitor
  5. When bipolar, uses higher power than monopolar
42.
  1. False
  2. False
  3. True
  4. True
  5. False
Surgical diathermy typically uses high-frequency currents in the region of 1 MHz (A).
The passage of lower frequency currents through the body can cause muscular contractions and ventricular fibrillation. The lower frequency of alternating current or the use of direct current is more likely to cause ventricular fibrillation than using high-frequency diathermy currents (B).
When using monopolar diathermy two connections are made to the patient: one is the patient plate (or neutral plate) and the other is the active (or cutting) electrode. When using bipolar diathermy, two connections are also made, but this is through the two ends of the forceps; a diathermy plate is not required (C).
Isolating capacitors are placed within the diathermy apparatus to increase safety of the equipment and reduce the risk of accidental burns to the patient in the event of the diathermy pad becoming detached (D).
Bipolar diathermy uses lower power than monopolar diathermy (E).
43. Regarding the hazards of magnetic resonance imaging (MRI), which of the following is true:
  1. MRI involves the use of ionising radiation
  2. There is a risk of causing involuntary muscle contraction
  3. Patients may experience flashing lights and sensations of taste
  4. Patients with metal prosthetic joints should not undergo MRI
  5. Ferromagnetic objects are safe to use within the fringe field of the scanner
43.
  1. False
  2. True
  3. True
  4. False
  5. False
46MRI does not involve the use of ionising radiation (A). There is little evidence of serious adverse effects as a result of the magnetic field. Various recommendations exist, but generally continuous exposure should be limited to a magnetic flux density of 0.2 T and short exposures to 5 T.
If the magnetic field gradient is switched, then eddy currents can be induced and this has the potential of inducing small currents in any biological conductors, especially nerve fibres, which can cause involuntary muscle contraction (B). There is also a small risk of causing breathing difficulties and inducing ventricular fibrillation as a result of these eddy currents.
In the presence of very strong fields, patients may experience flashing lights and various taste sensations (C).
Joint and dental prostheses are safe as they are usually non-ferromagnetic and are firmly fixed in the patient, thus cannot be dislodged by the magnetic field of the scanner (D). They may however distort MRI images.
Ferromagnetic objects are attracted to the scanner even in the fringe field, which can extend for a few metres (E).
44. Regarding the gas laws, which of the following is true:
  1. Boyle's law states that at a constant temperature, the volume of a given mass of gas varies inversely to the absolute pressure
  2. Gay Lussac's law is the third perfect gas law
  3. The second gas law states that at a constant pressure, the volume of a gas is proportional to the absolute temperature
  4. They are corrected to a standard temperature of 273°F
  5. Dalton's law states that PV/T = constant
44.
  1. True
  2. False
  3. True
  4. False
  5. False
Boyle's law is the first perfect gas law. If, at a constant temperature, the volume (V) of a gas is decreased, then the pressure (P) will increase by the same proportion (A). This principle can be used to calculate the volume of gas remaining in a cylinder when the pressure on the pressure gauge is known.
The second perfect gas law is Charles's law or Gay Lussac's law (B). This states that at a constant pressure the volume of a given mass of gas varies directly with the absolute temperature (T) (C). The third perfect gas law states that at a constant volume the absolute pressure of a given mass of gas varies directly with the absolute temperature. Combining the three perfect gas laws gives us the formula:
The perfect gas laws are usually corrected to a standard temperature of 273.15 K (0°C) and a standard pressure of 101 kPa (760 mmHg) (D). This is because gases are affected by temperature and pressure.
Dalton's law of partial pressures is not one of the perfect gas laws but refers to a mixture of gasses within a container (E). It states that in a mixture of gases the pressure exerted by each gas is the same as that which it would exert if it alone were occupying the container.
45. The following statements about simple mechanics are true:
  1. Power is defined as the work done when a force is applied in the direction of the force
  2. Power is measured in joules
  3. When the point of application of a force moves in the direction of the force energy is expended
  4. The power of breathing can be calculated by the area under the pressure/volume curve divided by time
  5. In respiratory muscles, most of the chemical energy is converted to mechanical energy
4745.
  1. False
  2. False
  3. True
  4. True
  5. False
Power is the rate of work, or the amount of work performed over time (A). It is measured in watts (W) with one watt equalling one joule (J) per second(s): 1 W = 1 J/s (B).
Energy is always expended and work is done when the point of application of a force moves in the direction of the force (C). It is measured in joules, the energy expended when applying a force of one newton (N) through a distance of one metre (m).
The area under a pressure/volume curve for ventilation is the work done. If power is the rate of work done, then the power of breathing can be calculated by dividing the work done (the area under the curve) by the time (D).
There are various energy changes that take place during the respiratory cycle. It must be remembered that energy cannot be created or lost, but it is converted from one form to another. So, most of the chemical energy generated by the respiratory muscles is converted to heat energy, with only a small proportion converted to mechanical energy (E).
46. Regarding humidity, which of the following is true:
  1. Relative humidity is expressed in milligrams of water per liter of gas
  2. Humidity may be expressed as the pressure exerted by water vapour in a gas mixture
  3. As humidity increases, hair length shortens
  4. If a fully saturated gas is cooled, both the absolute and the relative humidity will fall
  5. A wet and dry bulb hygrometer measures ambient and relative humidity
46.
  1. False
  2. True
  3. False
  4. False
  5. False
Humidity is the amount of water vapour present in gas, and it may be expressed in many ways. Absolute humidity is the mass of water vapour present in a volume of gas and is expressed in milligrams of water per litre of gas. Relative humidity is the amount of water vapour at a particular temperature, and this is expressed as a percentage of the amount that would be held if the gas were fully saturated (A).
Humidity may also be expressed as the pressure exerted by water vapour in a gas mixture (B).
If gas saturated with water vapour is cooled, then it will condense out water vapour. The amount of condensed water vapour will be the amount held at the original temperature minus the amount it can hold at the new lower temperature. The absolute humidity will therefore fall, but the relative humidity remains at 100% (D).
The hair hygrometer is one method used to measure relative humidity, and it operates on the principle that hair length increases if the humidity increases (C).
48The wet and dry bulb hygrometer allows the measurement of the ambient humidity, while relative humidity is obtained from a set of tables and is not directly measured (E).
47. Regarding heat and temperature, which of the following is true:
  1. Heat is a measure of the tendency of an object to gain or lose heat
  2. The SI unit for temperature is Celsius
  3. Temperature is the energy which can be transferred from a hotter object to a cooler one
  4. Zero Kelvin is absolute zero
  5. The triple point of water is at 273.16 K
47.
  1. False
  2. False
  3. False
  4. True
  5. True
Heat is the energy that can be transferred from a hotter object to a cooler object down this temperature gradient (A).
The SI unit for temperature is the Kelvin (K) and not Celsius (B).
Temperature is a measure of the tendency of an object to gain or lose heat (C).
Absolute zero is the zero reference point on this scale and 0 K is equivalent to -273°C (D).
The triple point of water is the temperature at which it exists as a solid, liquid and gas simultaneously. This is at 273.16 K or 0.01°C (E).
48. Regarding latent heat in anaesthesia:
  1. Liquid ethyl chloride vapourises from the skin causing a cooling sensation
  2. Ethyl chloride is stored as a liquid in a glass ampoule under pressure
  3. Cooling of volatile anaesthetics makes them less volatile
  4. Carbon dioxide is stored in cylinders in gaseous form
  5. As nitrous oxide is emptied from a cylinder, the pressure remains constant until all the liquid has vapourised
48.
  1. True
  2. True
  3. True
  4. False
  5. False
Latent heat is the heat absorbed or released when a substance changes its state at a constant temperature. As liquid ethyl chloride vapourises from the skin surface, it requires the latent heat of vapourisation to do so, which is gained from the skin. This results in cooling of the skin as heat is taken away from its surface (A).
Within glass ampoules ethyl chloride is stored under pressure as a liquid to ensure it does not vapourise until it has left the ampoule and contacts skin (B).
When volatile anaesthetics are vapourised, they lose latent heat and the vapourised liquid takes heat from the remaining fluid as well as the vapouriser. This can cause a fall in temperature of the remaining volatile anaesthetic, which makes them less volatile (C).
Nitrous oxide, carbon dioxide and oxygen may all be stored in liquid form when in cylinders, oxygen being a liquid in the vacuum-insulated evaporator (D).
As nitrous oxide is emptied from a cylinder, the liquid form is converted to gas which requires latent heat of vapourisation. This is taken from the remaining liquid and cylinder walls, which cools and may cause the water vapour in the air to condense outside the cylinder. Because of this cooling, there is a rapid fall in vapour pressure inside the cylinder, which is indicated on the pressure gauge (E).
49. The following are true regarding fluids and flow:
  1. Liquids are the only fluids
  2. During laminar flow, the fluid nearest to the wall has no flow
  3. In the middle of a pipe, the velocity is twice than that of the average velocity across the pipe, during laminar flow
  4. Turbulent flow requires less driving pressures than laminar flow
  5. Turbulent flow wastes more energy than laminar flow
4949.
  1. False
  2. True
  3. True
  4. False
  5. True
Fluids are any form of matter that change shape under shear forces and can either be liquids or gases (A).
The flow of fluids can be described either as laminar or turbulent. Laminar flow is flow in layers, or laminae, of fluid running parallel to each other. In a pipe, the layers nearest to the wall will have the lowest velocity, and the layer closest to the wall will probably have no flow (B). The velocity profile across the pipe during laminar flow is parabolic, with the highest flow in the centre. The peak velocity in the centre is twice the average across the pipe at equilibrium (C). There is minimum energy loss during laminar flow.
During turbulent flow, eddies, vortices and currents are developed which result in energy loss in the form of heat, friction and noise (E). The driving pressure required for turbulent flow is greater than that required for an equivalent laminar flow rate (D).
50. Regarding the principles of ultrasound, which of the following is true:
  1. It is a form of mechanical energy
  2. It is produced from piezoelectric crystals which are compressed and decompressed
  3. The frequency is dependent on the compression pressures
  4. The wavelength is the reciprocal of the frequency
  5. The wave can pass through a vacuum
50.
  1. True
  2. True
  3. False
  4. True
  5. False
Ultrasound is mechanical energy in the form of high-frequency vibrations (A).
It is generated by electrically deforming a piezoelectric crystal, which causes it to compress and decompress the medium to which the crystal is coupled (B). The wavelength is the distance occupied by one oscillation when seen graphically.
The changes in pressure created by the crystal travel through the medium; the distance between the points of maximum pressure is also the wavelength. The frequency is not dependent on the compression pressures; in fact it is the wavelength that is dependent on the compression pressures (C). The wavelength of ultrasound is generally the reciprocal of the frequency (D). The wave is propagated by the movement of particles; therefore, it is unable to travel through a vacuum (E).
51. The following are true regarding hazards of electrical equipment:
  1. Under single fault conditions, type 1 CF equipment should have a leakage current of the order of 4 mA
  2. Type BF equipment is safe because the patient circuit is earthed
  3. Class II equipment has power cables only containing ‘live’ and ‘neutral’ conductors
  4. Class III equipment is defined as that which operates at ‘safety extra low voltage’ of less than 12 V
  5. A current-operated earth-leakage circuit breaker relies on an unacceptable current causing disintegration of a fuse that then breaks the circuit
51.
  1. False
  2. False
  3. True
  4. False
  5. False
50Under single-fault conditions, type 1 CF equipment should have leakage currents under 50 μA (A).
Type BF equipment is floating, and may be either class I, II or III, thus is not, by definition, earthed (B).
Safety classification of equipment can be based on either:
  • The means of protection it provides
    • Class I – any accessible conducting part of the equipment is connected to an earth wire. Live, neutral and earth wires never come in contact with each other
    • Class II – also called ‘double-insulated’ as all accessible parts have two layers of insulation. Power cables only contain ‘live’ and ‘neutral’ conductors with a single fuse (C)
    • Class III – the equipment does not need electrical supply >50 V DC or 24 V AC; called ‘safety extra-low voltage’ (SELV) and still provides a theoretical risk of microshock (D)
  • The maximum leakage currents permissible
    • Type B – safe for external patient connection. Under single-fault conditions, equipment has leakage currents of 500 mA if class I or 100 mA if class II
    • Type BF – like type B, but the part connected to the patient is isolated (floating)
    • Type CF – safe for direct connection to the heart with isolated circuits.
      Leakage currents must be under 50 mA if class I or 10 mA if class II. Includes electrocardiogram leads and pressure transducers
Current-operated earth leakage circuit breakers (COELCB) consist of a live wire and a neutral wire attached to a relay circuit breaker. Faulty currents (e.g. high leakage current) lead to a magnetic field between the live and neutral wires that induce the relay to break the circuit. There is no fuse in COELCBs (E).
52. Nitrous oxide cylinders:
  1. Have blue and white quartered shoulders
  2. Are pressurised to 4.4 bar
  3. Have a filling ratio of 0.75 in the UK
  4. Have pressure within them decreasing linearly
  5. Of size E have a capacity of 1800 L
52.
  1. False
  2. False
  3. True
  4. False
  5. True
Nitrous oxide is stored in cylinders with French blue bodies and shoulders; Entonox is stored in cylinders with white and blue quartered shoulders (A). With a boiling point of -88°C and a critical temperature of 36.5°C, nitrous oxide exists as a liquid in cylinders with a vapour layer above.
It is stored at a pressure of 4400 kPa (44 bar), which decreases slowly until all the liquid is utilised with only vapour remaining at which point the pressure decreases rapidly (B). The slow decrease in cylinder pressures is due to the latent heat of vapourisation cooling the cylinder, thereby reducing the vapour pressure. Thus, cylinder pressures do not decrease in a linear fashion (D).
If nitrous oxide cylinders were full of liquid, then increasing temperature would risk the expansion of liquid, thus creating the risk of potential explosion. Therefore, the 51mass of gas in a cylinder divided by the mass of water required to fill that cylinder is termed the ‘filling ratio’, and in the UK the filling ratio is 0.75 (C).
Size E cylinders are attached directly to the anaesthetic machine and have a capacity of 1800 L, while size J cylinders are used in cylinder manifold and hold 18,000 L of nitrous oxide (E).
53. With regard to vacuum-insulated evaporators:
  1. They store liquid oxygen at a pressure of 400 kPa
  2. They have an internal temperature of -118°C
  3. The mass of oxygen can be measured by weight
  4. If less oxygen is utilised, the internal temperature rises
  5. If pressure exceeds 17 bar, this causes opening of a safety valve
53.
  1. False
  2. False
  3. True
  4. True
  5. True
A vacuum-insulated evaporator supplies oxygen to piped gas supplies in hospitals. It is composed of a double-insulated steel tank with a vacuum to maintain internal temperatures. Oxygen is stored as a liquid at -160°C which is below its critical temperature of -118°C, with an internal pressure of approximately 7 bar (700 kPa), not 400 kPa (A, B).
Up to 1500 L of liquid oxygen may be stored and the mass of the liquid can be measured either by weighing the vessel or by differential pressure gauges comparing pressures at the liquid bottom and vapour top of the cylinder (C).
When oxygen evaporates, the temperature of the vessel reduces due to the latent heat of vapourisation, further contributing to the maintenance of a low temperature. However, with reduced use of oxygen, the contribution of latent heat of vapourisation is reduced, therefore temperatures rise (D). With a rise in temperature, the internal pressure rises, and when this exceeds 17 bar, a safety valve opens allowing escape of vapourised oxygen (E). This leads to a further vapourisation, and thus a desired reduction in the temperature once again.
54. Flowmeters such as rotameters:
  1. Depend on gas density
  2. Rely on laminar flow at low flow rates
  3. Are accurate within 1%
  4. Reduce piped gas pressure
  5. Are inaccurate in the presence of static charge
54.
  1. True
  2. True
  3. False
  4. False
  5. True
Rotameter is the trade name of flowmeters that are constant-pressure, variable orifice devices composed of a conical tube that is wider at the top and a bobbin that varies in height depending on the flow rates. Opening of a needle valve at the bottom of the flowmeter allows gas into the tube, with higher flow rates allowing the bobbin to float higher within the tube. As the bobbin is of fixed dimensions, the distance between the bobbin and the wall of the tube increases as flow increases (see Figure 1.7). Thus, at low flow rates, the orifice is narrow and long allowing laminar flow depending on gas viscosity, while at higher flow rates the orifice is wider and flow is turbulent depending on gas density (A, B).
52
zoom view
Figure 1.7: Rotameter.
Flowmeters are 2.5% accurate but are only calibrated for the individual gases they are to be used for (C). Accuracy can be reduced by dirt within the flowmeter tubes, static electricity build-up (up to 35% inaccurate) and back pressure from the common gas outlet of up to 10% (E). Flowmeters have no effect on piped gas pressures (D).
55. The emergency oxygen flush:
  1. Delivers flow that bypasses the vapourisers
  2. Delivers at a pressure of 137 bar
  3. Has pressure-reducing valves to reduce barotrauma
  4. May lead to awareness
  5. Can deliver flow rates of 40–50 L/min
55.
  1. True
  2. False
  3. False
  4. True
  5. True
The emergency oxygen flush is designed to provide 100% oxygen flow that bypasses flowmeters and vapourisers at piped gas pressures of 4 bar (400 kPa) and flow rates of 35–75 L/min to the common gas outlet (A, B, E). Because it bypasses both flowmeters and vapourisers, it may therefore dilute anaesthetic gases potentially leading to awareness (D). It is a non-locking device that does not have any pressure reducing or regulating valves; therefore, the delivered pressure of 4 bar may potentially lead to barotrauma (C).
56. The following are maximum allowable environmental concentrations in the UK:
  1. Desflurane: 75 ppm
  2. Enflurane: 50 ppm
  3. Halothane: 25 ppm
  4. Isoflurane: 100 ppm
  5. Nitrous oxide: 25 ppm
56.
  1. False
  2. True
  3. False
  4. False
  5. False
In 1996, the Health and Safety Commission issued maximally accepted 8-hour time-weighted average concentrations for the UK:
  • Enflurane: 50 particles per million (ppm) (B)
  • Halothane: 10 ppm (C)
  • Isoflurane: 50 ppm (D)
  • Nitrous oxide: 100 ppm (E)
53These levels vary in different countries. No definite acceptable level was set for desflurane at the time of publication (A).
57. The Lack breathing system:
  1. Is a Mapleson D arrangement
  2. Is most efficient for controlled ventilation
  3. Can be in coaxial or parallel arrangements
  4. Requires a fresh gas flow rate of two to three times the minute ventilation during spontaneous ventilation
  5. Vents exhaled gases through a 14 mm inner tube
57.
  1. False
  2. False
  3. True
  4. False
  5. True
The Lack system is a coaxial version of the Mapleson A (Magill) breathing system that can also be in a parallel arrangement (A, C). It consists of a 14 mm inner tube attached to an adjustable pressure-limiting valve for exhaled gases and a 30 mm outer tube supplying the fresh gas flow (E).
It is efficient for spontaneously ventilating patients, requiring 70 mL/kg/min fresh gas flow, but is inefficient for controlled ventilation requiring a fresh gas flow rate of two to three times the minute ventilation (B, D). The most efficient Mapleson breathing system for controlled ventilation is the Mapleson D system, or Bain circuit.
58. The 12-lead electrocardiogram:
  1. Has a signal output of 1–2 mV
  2. Uses six bipolar leads
  3. Has a frequency response in monitoring mode of between 0 and 100 MHz
  4. Prints from an oscilloscope running at 25 mm/s
  5. CM5 arrangement optimally detects arrhythmias
58.
  1. True
  2. True
  3. False
  4. True
  5. False
The electrocardiogram is a device monitoring cardiac electrical activity using silver/silver chloride gel covered electrodes at specific sites on the skin. At the cardiac level, electrical potentials in the range of 90 mV are generated, but this signal is reduced as it passes through tissues, leaving a signal output of 1–2 mV (A). The initial signal is increased by a differential amplifier that may be set at a monitoring mode (frequency of 0.5–40 Hz) or diagnostic mode (frequency of 0.05–100 Hz) (C). The differential amplifier minimises noise and signal interference using the principles of common mode rejection, whereby interference that is the same between two leads is eliminated. The signal output is displayed via an oscilloscope or on paper running at 25 mm/s with a signal of 1 mV/cm (D).
There are six bipolar leads: I, II, III, aVR, aVL and aVF (note that ‘aV’ stands for ‘augmented voltage’) and there are six unipolar leads: V1–V6 (B). In anaesthetic practice, the configurations of three skin electrodes used include:
  • Lead II: right arm, left arm and indifferent electrodes; arrhythmia detection
  • CM5: right arm electrode on manubrium, left arm electrode on the 5th intercostal space, anterior axillary line and the indifferent electrode is on the left shoulder; sensitive for detecting ischaemia (E)
  • CB5: right arm electrode on right scapula, left arm electrode on the 5th intercostal space, anterior axillary line and the indifferent electrode is on the left shoulder; used in thoracic anaesthesia
59. Regarding statistical tests:
  1. A type I error involves rejecting a true null hypothesis
  2. β-errors are the same as false negatives
  3. The power of a study must be calculated after data collection
  4. A power of 20% is acceptable
  5. The variance is the square root of the standard deviation
5459.
  1. True
  2. True
  3. False
  4. False
  5. False
The null hypothesis (H0) states that no difference exists between two sample groups. The null hypothesis must be set prior to conducting any clinical trial, and if a difference between the two samples is detected, the null hypothesis must be rejected. It is liable to sources of error and two outcomes of error are possible: Type I or type II errors.
A type I error is also known as an α-error or a false positive. A difference is found when one does not truly exist causing incorrect rejection of the null hypothesis (A). It is most commonly due to a high p-value or a small sample size.
A type II error is also known as a β-error or a false negative (B). No difference is found when one does indeed exist causing incorrect acceptance of the null hypothesis. It is most commonly due to small sample sizes.
The power of a study should be calculated prior to conducting the study as it indicates what sample size is required (C). The power measures the probability of a difference being detected when one exists and is calculated as 1 – β. A power of 80% or above (i.e. β <20%) is sufficient for a study to be adequately powered (D).
Standard deviation (SD) is a measure of spread around central tendency of data:
where Σ is the sum of the difference of each value (x) from the mean (X̄) divided by the number of values (n). The mean ±1 SD should include 68% of data points, ±2 SD include 96% of all data points and ±3 SD include 99% of all data points.
Variance (SD2) is thus calculated by (E):
60. The following devices contain differential pressure transducers:
  1. Vacuum-insulated evaporators
  2. Paramagnetic analysers
  3. End-tidal carbon dioxide analysers
  4. Tec 6 vapourisers
  5. Pneumotachographs
60.
  1. True
  2. True
  3. False
  4. True
  5. True
Differential pressure transducers work by detecting a difference in pressure between two sides of a given sensor. Although many devices utilise this principle, it may be argued that all devices measuring pressure are differential pressure transducers, 55giving the difference between atmospheric pressure and device pressure. The overall design is one that compares a measurement sample with a reference sample to give a pressure difference, which can be analysed and interpreted (see Figure 1.8).
zoom view
Figure 1.8: Differential pressure transducer. The transducer detects the difference in pressures between measurement and reference samples.
The vacuum-insulated evaporator uses a differential pressure transducer between the top and bottom of the cylinder to calculate the mass of liquid oxygen remaining within the vessel (A).
Paramagnetic analysers are rendered particularly accurate using the principles of differential pressure (B). A reference gas of 21% oxygen is compared with a measured gas with a set concentration of oxygen. When exposed to a magnetic field, the paramagnetic properties of oxygen cause it to be attracted to the field, leading to a difference in pressure between both chambers that is proportional to the partial pressure of oxygen.
End-tidal carbon dioxide analysers do not incorporate differential pressure transducers in their design (C).
Tec 6 desflurane vapourisers contain a differential pressure transducer between the fresh gas flow channel and the vapourising channel, which is then interpreted and electronically alters the gas flow accordingly (D).
Pneumotachographs can be designed in a number of ways, one which uses the Pitot tube design. One tube faces the direction of gas flow, while the other faces the opposite direction, being the reference tube. The pressure difference between the two tubes is proportional to the flow rate squared (E).
56
Answers: SBAs
61. A 73-year-old man has had an asystolic cardiac arrest. You have successfully intubated him and cardiopulmonary resuscitation is underway.
Which is the most likely finding for this patient?
  1. Blood pressure of 52/26 mmHg
  2. End-tidal CO2 of 0.4 kPa
  3. A pH of 7.38
  4. A standard bicarbonate of 38.1 mmol/L
  5. A serum potassium of 7.8 mmol/L
61. B End-tidal CO2 of 0.4 kPa
In a cardiac arrest, there is no cardiac output, therefore no perfusion of the lungs. This is an extreme form of dead space involving both lungs. Although arterial CO2might be elevated, the end-tidal CO2 will be low if there is effective cardiopulmonary resuscitation (CPR) or zero if there is ineffective CPR. Asystolic patients will have no blood pressure and are more than likely going to have a metabolic and/or respiratory acidosis; therefore, the bicarbonate is likely to be low. Elevated serum potassium may cause cardiac arrest, but this is not necessarily true.
Weil MH, Bisera J, Trevino RP, Racklow EC. Cardiac output and end-tidal carbon dioxide. Crit Care Med 1985; 13:907–909.
62. A 68-year-old woman, who had a grade I intubation, is having an open reduction and internal fixation of an ankle fracture. She is being ventilated at a rate of 14 breaths per minute, with a peak inspiratory pressure of 36 cmH2O, achieving a tidal volume of 210 mL.
Which of the following is the most appropriate statement regarding lung compliance?
  1. The patient may have pulmonary oedema
  2. Normally lung compliance is 500 mL/cmH2O
  3. During an asthma attack lung compliance decreases
  4. The patient may have underlying emphysema
  5. It may be plotted on a flow/volume loop
62. A The patient may have pulmonary oedema
Compliance (C) is defined as the change in volume (ΔV) per unit change in pressure (Δρ) and it reflects the elastic recoil of an organ:
In the lung, ΔP is the difference in pressure between alveolar pressure measured at the mouth when there is no gas flow and intrapleural pressure measured by a balloon in the lower third of the oesophagus. Normal lung compliance (Clung) is 150–200 mL/cmH2O (1.5–2.0 L/kPa). Chest wall compliance (Cchest) is normally 200 mL/cmH2O (2.0 L/kPa), where ΔP is the difference between alveolar pressure and intrapleural pressure. Total thoracic compliance (Ctotal) is 85–100 mL/cmH2O (0.85–1 L/kPa) and is related to both Clung and Cchest:
Repeated pressure and volume readings can be plotted on a pressure/volume loop that is approximately linear during normal tidal volume breathing. However, the curves demonstrate hysteresis, meaning that the inflation and deflation curves are different (see Figure 1.9). This is because of the need to initially overcome surface tension upon inflation or inspiration.
Lung compliance can be either static or dynamic. Static compliance refers to the stiffness of the lung and chest wall, i.e. alveolar stretchability and is measured when there is no gas flow. Dynamic compliance is related to airway resistance during equilibration of gases at end-inspiration or end-expiration. Dynamic compliance is usually less than static compliance.
Because compliance is relative to body size, specific compliance is calculated as:
Pulmonary oedema increases lung compliance due to the interstitial oedema.
57
zoom view
Figure 1.9: Lung pressure/volume loops demonstrating hysteresis.
However, asthma has been shown to reduce lung compliance, a pathology related to a loss of elastic recoil at total lung capacity and increased compliance of airways in spite of airway oedema. Emphysema also increases lung compliance, again due to loss of elastic recoil.
Compliance can be affected by the following factors (see Table 2.1):
  • Increased compliance: surfactant, emphysema, old age, acute asthma (reason unclear)
  • Reduced compliance: pulmonary fibrosis, pulmonary venous engorgement, pulmonary oedema, ARDS, pneumonia, neonates, extremes of lung volumes, chronic bronchitis (dynamic)
Wenzel S. Severe asthma in adults. Am J Respir Crit Care Med 2005; 172:149–160.
West JB. Respiratory physiology: the essentials, 9th edn. Baltimore: Lippincott Williams & Wilkins; 2012.
63. A 23-year-old man has sustained a traumatic intracranial haemorrhage with evidence of raised intracranial pressure (ICP). He is intubated and requires transfer to a specialist neurosurgical centre.
Which of the following is most likely to reduce his ICP?
  1. Increasing his Pao2 from 9.0 to 12 kPa
  2. Sedation with ketamine
  3. Reducing Paco2 from 5.8 to 4.2 kPa
  4. 8 mg of dexamethasone
  5. Fluid restriction
63. C Reducing Paco2 from 5.8 to 4.2 kPa
There are a number of factors affecting intracranial pressure (ICP), and management of traumatic brain injury is aimed at reducing secondary injury. Target Paco2 should be 4.5–5.0 kPa, with a reduction in cerebral blood flow of approximately 2–4% for each 0.13 kPa reduction in Paco2. Avoidance of hypoxia is vital, but the ICP does not significantly increase above a Pao2 of 6.7 kPa. Another management target is avoidance of hypotension and fluid restriction in the immediate phase that reduces mean arterial pressure (MAP), thereby reducing cerebral perfusion pressure (CPP): CPP = MAP – (ICP + CVP). Dexamethasone may reduce cerebral oedema secondary to tumours, but there is no evidence that it affects traumatic brain injury. There are a number of anaesthetic drugs that affect ICP (see Table 1.9).
Curry P, Viernes D, Sharma D. Perioperative management of traumatic brain injury. Int J Crit Illn Inj Sci 2011; 1:27–35.
58
Table 1.9   Anaesthetic drugs affecting intracranial pressure (ICP)
Increased ICP
Decreased ICP
Ketamine
Barbiturates, etomidate, propofol
Volatile agents
Opioids
Suxamethonium (transient)
64. A 75-year-old man in the emergency department has a heart rate of 44 beats per minute and an unrecordable blood pressure. His pulses are not palpable.
What is the first drug this patient should receive?
  1. Atropine
  2. Adrenaline
  3. Amiodarone
  4. Ephedrine
  5. Metaraminol
64. B Adrenaline
This patient is in pulseless electrical activity (PEA) cardiac arrest. The Resuscitation Council (UK) published their latest guidelines in 2010 that modified the application of atropine in cardiac arrest situations. It is no longer recommended for use in PEA or asystolic cardiac arrests due to a poor evidence base. The first-line pharmacological therapy should be administration of 1 mg adrenaline via the intravenous or intraosseous routes.
Nolan J, Soar J, Lockey A. Advanced life support, 6th edn. London: Resuscitation Council; 2011.
65. A 54-year-old woman is having a bunionectomy under general anaesthetic. You perform an ankle block using the landmark technique.
Which of the following nerves could a nerve stimulator be used to block?
  1. Deep peroneal nerve
  2. Saphenous nerve
  3. Superficial peroneal nerve
  4. Sural nerve
  5. Tibial nerve
65. E Tibial nerve
An ankle block may be performed to provide analgesia or anaesthesia for forefoot operations. It requires blockade of the deep and superficial peroneal nerves, sural nerve, saphenous nerve and tibial nerve. As the tibial nerve is the only one that has a motor component beyond the site of block, it is the only nerve that a nerve stimulator may identify. Stimulation of the tibial nerve leads to plantar flexing of the toes via contraction of the flexor digitorum longus and flexor hallucis longus. The tibial nerve also provides sensory supply to the medial aspect of the ankle and foot.
Kopka A, Serpell MG. Distal nerve blocks of the lower limb. Cont Educ Anaesth Crit Care Pain 2005; 5 (5): 166–170.
66. An 8-year-old girl (30 kg body weight) presents for tonsillectomy. You have prepared drugs and equipment.
Which calculation is correct?
  1. Length of a nasal tube at the nares is 19 cm
  2. Endotracheal tube size is 5.0 mm internal diameter
  3. Laryngeal mask size is 2.0
  4. Fentanyl dose is 15 µg
  5. Intravenous paracetamol dose is 800 mg
66. A Length of a nasal tube at the nares is 19 cm
Drug dosing and equipment calculation is an important aspect of preparation for paediatric anaesthesia. The important calculations for equipment and drug dosing are as follows:
Equipment
  • Endotracheal tube internal diameter size = (Age/4) + 4
  • Length of oral endotracheal tube at lips = (Age/2) + 12
  • Length of nasal endotracheal tube at nares = (Age/2) + 15
  • Laryngeal mask sizes as defined by Table 1.10.
Drug dosing: for details of drug dosing for paediatric anaesthesis see Table 1.11.
Therefore, this patient should have a size 6.0 mm endotracheal tube at 16 cm at the lips for an oral tube and 19 cm at the nares for a nasal endotracheal tube. The patient should have a size 2.5 or 3 laryngeal mask airway.
59
Table 1.10   Paediatric laryngeal mask airway (LMA) sizes
LMA size
Patient weight (kg)
1
<5
1.5
5–10
2
10–20
2.5
20–30
3
30–50
Table 1.11   Drug dosing for paediatric anaesthesia
Drug
Dose
Analgesics
Paracetamol
15 mg/kg intravenous (7.5 mg/kg if weight is < 10 kg)
40 mg/kg loading dose per rectum
15 mg/kg every 4–6 hours
Ibuprofen
10 mg/kg
Diclofenac
1 mg/kg
Codeine
0.5 mg/kg
Fentanyl
1 µg/kg
Morphine
0.1–0.2 mg/kg
Antiemetics
Dexamethasone
0.1–0.2 mg/kg
Ondansetron
0.1–0.2 mg/kg
Girgis, Sanders. The BADS Paediatric Dose Table. Journal of One-day Surgery 2004; 14:65–8.
South Thames Retrieval Service. Clinical Guidelines. http://www.strs.nhs.uk/educationandguidelines/guidelines.aspx (Last accessed 01/10/2012).
67. A 34-year-old man has a rapid sequence induction for a perforated duodenal ulcer. Twenty minutes later his heart rate rises to 125 beats per minute, blood pressure decreases to 75/40 mmHg, airway pressures increase to 36 cmH2O and cutaneous urticaria is rapidly spreading.
What is the most likely causative agent?
  1. Atracurium
  2. Succinylcholine
  3. Latex
  4. Morphine
  5. Co-amoxiclav
67. B Succinylcholine
The clinical features described are in keeping with a diagnosis of anaphylaxis. Anaphylaxis is an IgE-mediated type 1 hypersensitivity reaction caused under anaesthesia by the substances shown in Table 1.12.
Thus, in this patient, it is most likely to be due to the use of succinylcholine during rapid sequence induction. However, formal allergy testing would be mandatory.
Association of Anaesthetists of Great Britain and Ireland. Suspected anaphylactic reactions associated with anaesthesia. Anaesthesia 2009; 64:199–211.
60
Table 1.12   Anaesthetic causes of anaphylaxis
Agent
Frequency (%)
Notes
Neuromuscular blocking drugs
60
Most commonly succinylcholine and rocuronium, rarely atracurium or mivacurium
Latex
20
Most commonly in atopic patients, healthcare workers, fruit allergy
Antibiotics
15
Penicillins and cephalosporins account for 70% of antibiotic-related anaphylaxis
Colloids
4
Most commonly to gelatins, rare with starches
Opioids
Uncommon
Usually due to histamine release
Local anaesthetics
Rare
May be related to preservatives
Anaesthetic agents
Very rare
Rare with propofol, very rare with thiopental, never reported with volatile agents
68. A 68-year-old woman with chronic obstructive airways disease and rheumatoid arthritis presents for a total shoulder placement. You perform an interscalene nerve block using a nerve stimulator technique.
What is the most likely complication in this patient?
  1. Recurrent laryngeal nerve palsy
  2. Horner's syndrome
  3. Pneumothorax
  4. Vertebral artery puncture
  5. Phrenic nerve palsy
68. E Phrenic nerve palsy
An interscalene brachial plexus block is indicated for shoulder, humeral or elbow surgery. It may be performed using the landmark technique in the interscalene groove at the level of C6, ultrasound guidance or a nerve stimulator eliciting deltoid and biceps contraction.
Nearly 100% of patients have a phrenic nerve block; therefore, the block should only be performed unilaterally. Other complications include a stellate ganglion block causing Horner's syndrome in up to 25% of patients, recurrent laryngeal nerve palsy in up to 10% of patients and vagal nerve damage. Vertebral artery and other vessel puncture may occur, and intrathecal or epidural injunction may also take place. Pneumothorax is rare but more common in patients with chronic obstructive airways disease, thus care must be taken in this patient.
Nicholls B, Conn D, Roberts A. The Abbott pocket guide to practical peripheral nerve blockade.
Maidenhead: Abbott Anaesthesia; 2010.
Urmey WF, Talts KH, Sharrock NE. One hundred percent incidence of hemidiaphragmatic paresis associated with interscalene brachial plexus anesthesia as diagnosed by ultrasonography. Anesthesia and Analgesia 1991; 72: 498–503.
69. A 44-year-old man (100 kg body weight) in the intensive care unit with acute respiratory distress syndrome is being ventilated with volume-controlled ventilation at tidal volumes of 620 mL, peak inspiratory pressures of 29 cmH2O, positive end-expiratory pressure (PEEP) of 6 cmH2O and a Fio2 of 0.9. His arterial blood gas demonstrates a Pao2 of 6.9 kPa and a Paco2 of 7.3 kPa.
What intervention is most likely to increase his Pao2?
  1. Increase the tidal volume
  2. Increase the Fio2
  3. Increase the expiratory time
  4. Increase the respiratory frequency
  5. Increase the PEEP
69. E Increase the positive end-expiratory pressure (PEEP)
Acute respiratory distress syndrome (ARDS) is traditionally defined by the American-European Consensus Conference (1994) as:
  • Acute onset
  • Bilateral infiltrates on chest X-ray
  • Absence of left atrial hypertension
  • A Pao2:Fio2 ratio of <200 mmHg (26.7 kPa)
A more recent consensus definition of ARDS has now replaced this definition. The Berlin definition (2012) involves:
61
  • Onset of ARDS within 7 days of a defined event (e.g. sepsis)
  • Bilateral opacities consistent with pulmonary oedema on either chest X-ray or CT scan
  • Respiratory failure not fully explained by cardiac failure or fluid overload; objective assessment with echocardiography should be performed.
ARDS is caused by either pulmonary or extrapulmonary pathology, both producing hypoxia due to an increase in both shunt and dead space
Treating hypoxia is best achieved with alveolar recruitment. This can be done by increasing the mean airway pressures, either by increasing the positive end-expiratory pressure (PEEP) or prolonging the inspiratory time. The ARDSnet trial have recommended a lung-protective strategy for ventilating patients with ARDS, and the recommendations include:
  • Use of pressure-controlled ventilation
  • Aim for tidal volumes of 6 mL/kg of ideal body weight
  • Plateau pressures of <30 cmH2O
  • PEEP of 10–12 cmH2O
  • Titrate Fio2 to a Pao2 of 8 kPa
  • Permissive hypercapnia of 8 kPa, increasing the respiratory rate to reduce the Paco2
In this patient, increasing the tidal volumes will increase peak airway pressures and have minimal effect on mean airway pressures. The patient already has an Fio2 of 0.9; thus, increasing the Fio2 will have minimal effect on hypoxia and will not aid alveolar recruitment. Increasing the expiratory time will only shorten the inspiratory time, lead to higher peak airway pressures and reduce alveolar recruitment. Increasing the respiratory rate will have a greater effect on offloading CO2 than oxygenation and will be of limited benefit in this scenario.
The Acute Respiratory Distress Syndrome Network. Ventilation with lower tidal volumes as compared with traditional tidal volumes for acute lung injury and the acute respiratory distress syndrome. N Engl J Med 2000; 342:1301–1308.
The ARDS Definition Task Force. Ranieri VM, Rubenfeld GD, et al. Acute respiratory distress syndrome: the Berlin definition. JAMA 2012; 307:2526–2533.
70. A 22-year-old woman with a past medical history of asthma presents with difficulty in breathing, a respiratory rate of 28 breaths per minute, inability to complete sentences and wheeze. An arterial blood gas breathing room air reveals a Pao2 of 8.5 kPa and a Paco2 of 3.4 kPa. Five milligrams of nebulised salbutamol has been administered.
What is the next step in this patient's management?
  1. Nebulised antimuscarinic agents
  2. Check peak expiratory flow rates
  3. Intubation and ventilation
  4. Intravenous magnesium
  5. Request a chest radiograph
70. A Nebulised antimuscarinic agents
This patient has evidence of acute severe asthma, which is defined by the British Thoracic Society (BTS) as any one of:
  • Peak expiratory flow rate (PEFR) of 33–50% of best or predicted
  • Respiratory rate of 25 breaths per minute or more
  • Heart rate or 110 beats per minute or more
  • Inability to complete sentences in one breath
Life-threatening asthma as defined by BTS (2012) guidelines includes:
  • PEFR of <33% best or predicted
  • Spo2 <92%
  • Quiet chest, cyanosis or reduced respiratory effort
  • Arrhythmia or hypotension
  • Altered consciousness
  • ‘Normal’ Paco2 of 4.6–6.0 kPa
    62
  • Severe hypoxia with Pao2 of <8.0 kPa
  • Acidosis
The immediate treatment of acute severe asthma in adults is:
  • Oxygen to maintain Spo2 94–98%
  • Nebulised salbutamol 5 mg or terbutaline 10 mg (β2-agonists)
  • Nebulised ipratropium bromide 0.5 mg (antimuscarinic)
  • Oral prednisolone 40–50 mg or intravenous hydrocortisone 100 mg
  • Chest radiograph only if pneumothorax or consolidation is suspected or patient requires intubation
  • If life-threatening asthma, then also:
    • Consider ventilation
    • Consider intravenous magnesium sulphate 1.2–2 g via intravenous infusion
    • More frequent salbutamol nebulisers
This patient does not yet require intubation and ventilation, and there is no evidence of requirement for chest radiography at this current stage. PEFR assessment should be performed after immediate treatment as the diagnosis of acute severe asthma is already made. Intravenous magnesium should only be used once conventional therapy has been used and not been successful.
British Thoracic Society/Scottish Intercollegiate Guidelines Network. British Guideline on the Management of Asthma. January 2012 (http://www.brit-thoracic.org.uk/Portals/0/Guidelines/AsthmaGuidelines/sign101%20Jan%202012.pdf) (Last accessed 01/10/2012).
71. While trying to attach a cylinder to the anaesthetic machine, you realise it does not fit appropriately. The pin index shows positions 3 and 5.
Which is the most likely cylinder?
  1. Nitrous oxide
  2. Oxygen
  3. Air
  4. Carbon dioxide
  5. Entonox
71. A Nitrous oxide
The pin index system is a safety mechanism designed to avoid attachment of the incorrect cylinder to the wrong yoke on the anaesthetic machine. Each gas has a specific pin index to ensure the valve block holes correspond to the machine yoke (see Figure 1.10). The pin index system positions for the common gases are shown in Table 1.13.
zoom view
Figure 1.10: The pin index system showing pin positions.
63
Table 1.13   Pin index system positions for the common gases
Gas
Pin index system positions
Nitrous oxide
3 and 5
Oxygen
2 and 5
Air
1 and 5
Carbon dioxide
1 and 6
Entonox
7
Al-Shaikh B, Stacey S. Essentials of anaesthetic equipment, 3rd edn. Edinburgh: Churchill Livingstone; 2007.
72. A 36-year-old woman has oxygenation saturations of 96% on pulse oximetry but 88% on arterial blood gas oximetry analysis.
What is the most likely cause for this discrepancy?
  1. Hyperbilirubinaemia
  2. Indocyanine green dye
  3. Methaemoglobinaemia
  4. Carboxyhaemoglobin
  5. Presence of nail varnish
72. D Carboxyhaemoglobin
Pulse oximetry uses the principle of light absorption based on Beer's and Lambert's laws. They are accurate above 70% to ±2%, below which the saturation is extrapolated from studies. Pulse oximeters under read in presence of:
  • Indocyanine green dye
  • Methaemoglobinaemia
  • Methylene blue dye
  • Nail varnish
It over-reads, as in this patient, in the presence of carboxyhaemoglobin because the absorption coefficient is similar to that of oxyhaemoglobin, causing the readings to be 96%.
Pedersen T, Moller AM, Pedersen BD. Pulse oximetry for perioperative monitoring: Systematic review of randomized, controlled trials. Anesth Analg 2003; 96:426–431.
Al-Shaikh B, Stacey S. Essentials of anaesthetic equipment, 3rd edn. Edinburgh: Churchill Livingstone; 2007.
73. While floating a pulmonary artery catheter, a pulmonary capillary wedge pressure of 15 mmHg is shown on the monitor.
This represents the pressure in which of the following?
  1. Right atrium
  2. Right ventricle
  3. Left atrium
  4. Left ventricle
  5. Pulmonary artery
73. C Left atrium
The tip of the catheter is placed in the pulmonary artery tree where it is wedged. Once wedged the pulsatile waveform is lost to continuous low pressure reading; this is the pulmonary artery occlusion pressure or the pulmonary capillary wedge pressure; and it is an accurate representation of the left atrial pressure. However, the presence of the mitral valve means that the pressure analysed is more directly related to the left atrium, although the left ventricular pressure can be inferred from this. Because the pressure being measured is beyond the right atrium, ventricle and pulmonary artery, it is not related to these three pressures.
Levick JR. An introduction to cardiovascular physiology, 5th edn. London: Hodder Arnold; 2010.
74. Using the flow of water through a pipe, driven by a mechanical pump as an analogy for electricity, the pressure difference between two points in the pipe would correspond to which of the following?
  1. Current
  2. Work
  3. Voltage
  4. Resistance
  5. Power
74. C Voltage
The water pressure difference between two points would correspond to the voltage difference in an electrical circuit. If there is a water pressure difference between two points, then water will flow from one point to the other and this would allow the 64water to do work, such as rotate the blades in a turbine. Similarly, work is done by the flow of an electric current in a circuit driven by a voltage difference; the current which is generated could then provide power to an electrical device such as a light bulb. If the water pump is not working, then there will be no pressure difference generated so the turbine blades will not rotate, similarly if there is no flow of current then the bulb will not light up.
Davis PD, Kenny GNC. Basic physics and measurement in anaesthesia, 5th edn. Boston: Butterworth-Heinemann; 2003.
75. A 20-year-old woman has arrived in the anaesthetic room for elective orthopaedic surgery. She is needle phobic and is insisting on a gaseous induction of anaesthesia.
Which of the following would most efficiently and rapidly allow you to perform a gas induction?
  1. Bain breathing system
  2. Mapleson A breathing system
  3. Mapleson B breathing system
  4. Mapleson C breathing system
  5. Mapleson D breathing system
75. B Mapleson A breathing system
Mapleson A breathing systems are the most efficient for spontaneous ventilation, and the fresh gas flows required are equal to alveolar minute ventilation. This system is not efficient for controlled ventilation, as it would require fresh gas flows three times the minute ventilation. Mapleson B and C systems are not efficient for spontaneous or controlled ventilation; however, the B system is more efficient than the A and C systems for controlled ventilation.
The Bain system is a coaxial version of the Mapleson D breathing system. This system is not efficient for spontaneous ventilation, requiring a fresh gas flow of about twice the alveolar minute ventilation, but is efficient during controlled ventilation.
Mapleson WW. Editorial I: Fifty years after – reflections on ‘The elimination of rebreathing in various semi-closed anaesthetic systems’. Br J Anaesth 2004; 93:319–321.
76. A 75-year-old man has had emergency vascular surgery. He presented with a full stomach, therefore required a rapid sequence induction to secure his airway followed by prolonged surgery requiring significant blood transfusion.
Which of the following extubation strategies is least appropriate?
  1. Deep extubation
  2. Laryngeal mask exchange
  3. Airway exchange catheter
  4. Remifentanil technique
  5. Awake extubation
76. A Deep extubation
This patient would be classed as an ‘at risk’ extubation; he has a full stomach, has undergone emergency surgery taking several hours and lost significant amount of blood requiring blood transfusion. The patient may not be able to maintain his own airway after the tracheal tube has been removed. An ‘at-risk’ extubation is characterised by the concern that once extubated, airway management may be difficult. A deep extubation should not be performed in such a patient. Deep extubation should only be performed in ‘low-risk’ extubation cases; these are routine extubations where reintubation could be managed without difficulty if required.
All the other options are appropriate techniques for ‘at-risk’ extubations, but must be performed by clinicians who are experienced in the technique.
Difficult Airway Society Extubation Guidelines Group. Popat M, Mitchell V, et al. Difficult Airway Society Guidelines for the management of tracheal extubation. Anaesthesia 2012; 67:318–340.
77. A 35-year-old man requires surgery to repair a lacerated biceps tendon. He had been assaulted and sustained a left-sided pneumothorax and a 5 cm stab wound to his right antecubital fossa.
Which of the following ultrasound-guided upper limb blocks would be the most appropriate for surgery?
  1. Interscalene
  2. Supraclavicular
  3. Infraclavicular
  4. Axillary
  5. Peripheral nerve block
77. D Axillary
This patient has a pneumothorax on the contralateral side to where the block will be performed. Either a supraclavicular, infraclavicular or axillary nerve block will provide appropriate analgesic coverage for the area where surgery will be performed. The dermatomal distribution for this is region is C5, C6 and T1 where the sensory innervation will be from the median and ulnar nerves.
65For a supraclavicular block, the brachial plexus is blocked at the level of the divisions and when performed under ultrasound guidance, the risk of pneumothorax is reduced, but not completely eliminated, even by experienced practitioners. Therefore, there is still a risk of pneumothorax, which must be avoided as the patient already has a pneumothorax on the contralateral side.
An interscalene block would not provide adequate cover for an incision over the antecubital fossa. There is also a high chance of phrenic nerve palsy on the ipsilateral side, thus the patient would be at risk of respiratory distress as he already has a pneumothorax on the left and a phrenic nerve palsy on the right could further compromise ventilation. This is a further reason for not choosing this block.
Axillary and infraclavicular blocks both cover a similar area, which will be adequate for surgery in the antecubital fossa. But there is still some associated risk of pneumothorax when performing an infraclavicular block, even if guided by ultrasound.
Therefore, the most appropriate block would be an axillary nerve block, this will provide adequate coverage to the antecubital fossa and has no risk of pneumothorax. A peripheral nerve block is less likely to be as effective as an axillary block in this circumstance.
Perlas A, Lobo G, Lo N, et al. Ultrasound-guided supraclavicular block: outcome of 510 consecutive cases. Reg Anesth Pain Med 2009; 34:171–176.
78. The components of a circle breathing system attached to a standard anaesthetic machine include soda lime in a canister.
Which of the following is incorrect regarding soda lime?
  1. It contains a pink dye
  2. The dye changes to white when the soda lime is exhausted
  3. The size of the granules should be 4–8 mesh
  4. It consists of 95% sodium hydroxide
  5. Silica is added
78. D It consists of 95% sodium hydroxide
In the circle breathing system soda lime is added to absorb exhaled carbon dioxide. As a result fresh gas flows may be low and, compared with all the Mapleson breathing systems, this makes the circle breathing system a very efficient and economical one. It also causes minimal pollution.
The soda lime is placed in a canister which has two ports: one to deliver fresh gases to the patient and another to receive exhaled gases via unidirectional valves. Soda lime is made of 94% calcium hydroxide, 5% sodium hydroxide and a small amount of potassium hydroxide.
A dye is added which indicates when the soda lime is exhausted; this can either be from pink to white or white to violet. Silica is added to stop the granules from disintegrating into dust particles, which could be inhaled by the patient. The soda lime granules are 4–8 mesh in size, but they can also be made into spherical shapes of 3–4 mm in size.
Al-Shaikh B, Stacey S. Essentials of anaesthetic equipment, 3rd edn. Edinburgh: Churchill Livingstone; 2007.
79. A 35-year-old woman is having laparoscopic gynaecological surgery. She is anaesthetised and has a blood pressure of 110/60 mmHg and heart rate of 60 beats per minute as the surgeon is insufflating the peritoneum with gas. There is an immediate complication as a result of the pneumoperitoneum.
Which of the following emergency drugs would you first administer?
  1. Adrenaline
  2. Atropine
  3. Ephedrine
  4. Suxamethonium
  5. Metaraminol
79. B Atropine
The patient has a low heart rate at this point, and one of the commonest complications of pneumoperitoneum is bradycardia as a result of stimulation of the Vagus nerve due to rapid peritoneal stretching. The combination of a pre-existing 66low heart rate and vagal stimulation will most likely cause a bradycardia requiring immediate atropine administration and temporary cessation of air insufflation.
Metaraminol would make the bradycardia worse, so should not be administered in this scenario.
Ephedrine and adrenaline could be administered, but there is nothing to suggest that the blood pressure has been compromised, and therefore the first line of treatment should be atropine, followed by ephedrine if the blood pressure is compromised.
There are many complications associated with gas insufflation for laparoscopic surgery including:
  • Trauma to intra-abdominal viscera and great vessels
  • Gas embolus if gas is insufflated into a blood vessel
  • Pneumothorax
  • Pneumomediastinum
  • Subcutaneous emphysema
  • Caval compression
Perrin M, Fletcher A. Laparoscopic abdominal surgery. Contin Educ Anaesth Crit Care Pain 2004; 4:107–110.
80. You have chosen to use an infusion of remifentanil and propofol to induce and maintain anaesthesia for a patient undergoing elective surgery for a plastics procedure.
Which of the following is least likely to guarantee drug delivery using this technique?
  1. Use of non-return valves
  2. Keeping the cannulation site visible at all times
  3. Being aware of the uses and limitations of the infusions pumps
  4. Infusing intravenous fluids through the same cannula as the drugs
  5. Using a large proximal vein for infusion
80. E Using a large proximal vein for infusion
The Safe Anaesthesia Liaison Group has published guidelines in 2009 to help guarantee the administration of drugs during total intravenous anaesthesia. This was after a number of incidences of awareness as a result of inadequate drug delivery.
Non-return or one-way valves prevent the backflow of the anaesthetic drugs into the intravenous fluid lines, therefore guaranteeing the delivery of these drugs to the patient.
Monitoring the cannulation site at regular intervals is a simple way to reduce the risk of inadequate drug delivery by ensuring that accidental disconnection, backtracking of anaesthetic drugs or displacement of the cannula are noted early.
Infusing fluids through the same cannula as the anaesthetic drugs will help ensure their delivery because if the fluid infusion abruptly stops or suddenly flows at a very fast rate, occlusion or disconnection of the cannula could be the cause and this can then be investigated and resolved.
The use of a large vein as opposed to a smaller vein does not further guarantee drug delivery.
Safe Anaesthesia Liaison Group. Guaranteeing drug delivery in total intravenous anaesthesia. London: National Patient Safety Agency; October 2009.
81. A 46-year-old man is being anaesthetised for functional endoscopic sinus surgery to remove nasal polyps as a day case procedure.
Which of the following is most likely to reduce intraoperative bleeding?
  1. Hypotensive anaesthesia
  2. Normocarbia
  3. Reverse Trendelenburg tilt
  4. Use of Moffat's solution
  5. Normothermia
81. A Hypotensive anaesthesia
Intraoperative bleeding is one of the major problems of this type of surgery and can lead to complications such as impairing the surgeon's vision, increasing the duration of surgery and increasing the risk of intracranial and orbital complications. Blood 67loss is primarily dependent on cut vessels, but can be reduced considerably by a reduction in the mean arterial pressure. Controlled hypotension deliberately and predictably decreases mean arterial pressure to limit intraoperative blood loss and is the method most likely to reduce intraoperative bleeding.
Normocarbia will prevent increased blood flow to the head and neck region as a result of vasodilation which would otherwise occur with hypercarbia. The reverse Trendelenburg position helps reduce local blood flow to the capillary bed by having the patient in a slightly head-up position. However, neither of these techniques is as effective as hypotensive anaesthesia.
Moffat's solution is a sprayed onto the nasal mucosa to induce vasoconstriction, reduce intraoperative bleeding, reduce congestion of the nose and act as a local anaesthetic. It consists of a combination of cocaine, adrenaline and sodium bicarbonate. This is often required within a multimodal approach for generating a bloodless field but is usually insufficient in isolation. Surgeons often supplement Moffat's solution with lignocaine and adrenaline injections intraoperatively because it rarely guarantees a bloodless field.
Hypothermia has been shown to increase intraoperative blood loss and the need for blood transfusion due to hypothermia-induced impairment of platelet function; this is more likely to occur with a core temperature of 35–35.7°C, therefore will have less influence on this short duration surgery.
Allman KG, Wilson IH. Oxford handbook of anaesthesia, 3rd edn. Oxford: Oxford University Press; 2011.
82. You are about to perform a fibreoptic intubation in a patient with a known difficult airway.
Which of the following would be the most appropriate method of preventing cross-contamination with a fibreoptic scope?
  1. Cleaning
  2. Sterilisation
  3. Disinfection
  4. Decontamination
  5. Autoclaving
82. D Decontamination
Decontamination is the process of removing contaminants such that they are unable to reach a site in sufficient quantities to initiate an infection or other harmful reaction. The process of decontamination always starts with cleaning and is followed by disinfection or sterilisation. Fibrescopes are classed as intermediate risk equipment (do not penetrate the skin or enter sterile cavities) so decontamination by cleaning followed by disinfection is an acceptable method of cleaning this type of equipment.
Cleaning is the process of physically removing foreign material from an object without necessarily destroying any infective material. An example of manual cleaning is washing, and automated cleaning is performed using an ultrasonic bath or low-temperature steam.
Disinfection is the process of rendering an object free from all pathological organisms except bacterial spores. Chemicals used for disinfection are glutaraldehyde 2%, alcohol 60–80%, chlorhexidine 0.5–5% and hydrogen peroxide. Pasteurisation is the process of using heat to destroy pathogenic organisms without destroying bacterial spores. It is an alternative to chemical disinfection.
Sterilisation is the process of rendering an object completely free of all visible infectious agents including bacterial spores. This can be done using chemicals such 68as ethylene oxide and glutaraldehyde 2%, using radiation like gamma irradiation, or using heat such as the autoclave.
Sabir N, Ramachandra V. Decontamination of anaesthetic equipment. Contin Educ Anaesth Crit Care Pain 2004; 4:134–135.
83. A 43-year-old woman having a laparoscopic cholecystectomy has an oxygen saturation which rapidly decreases to 80% on 100% inspired oxygen, peak airway pressures increase to 38 cmH2O, and blood pressure reduces to 72/42 mmHg. Examination demonstrates absent breath sounds and a hyper-resonant percussion note on the left side of the chest.
What is the most appropriate immediate management?
  1. Increase tidal volumes
  2. 50–100 µg of adrenaline
  3. Needle decompression
  4. Increase respiratory rate
  5. Fluid bolus
83. C Needle decompression
This patient is demonstrating clinical features of a tension pneumothorax with hypoxia, elevated airway pressures and hypotension. This is a recognised risk of laparoscopic surgery and requires immediate recognition and treatment with insertion of a cannula in the second intercostal space, midclavicular line on the affected side. An intercostal chest drain should then be inserted for this life-threatening emergency.
Adrenaline may be used but would not treat the underlying cause. Optimising ventilatory parameters would not treat the cause and may worsen any tension by increasing the size of the pneumothorax. Administration of intravenous fluid boluses would be of benefit but would not be addressing the underlying cause.
Hayden P, Cowman S. Anaesthesia for laparoscopic surgery. Contin Educ Anaesth Crit Care Pain 2011; 11:177–180.
84. A 68-year-old woman presents with shortness of breath and weakness. A 12-lead electrocardiogram shows a rate of 42 beats per minute with complete dissociation of P waves from QRS complexes.
What is the most likely jugular venous pressure (JVP) waveform that will be seen?
  1. Absent JVP waveforms
  2. Absent a waves
  3. Raised JVP with normal waveforms
  4. Large v waves
  5. Cannon a waves
84. E Cannon a waves
The jugular venous pressure (JVP) is assessed by viewing the height and pulsations of the jugular veins with the patient sitting at 45° and the head turned slightly away. It has a characteristic double-pulsatile waveform pattern, distinguishing it from arterial pulsations (see Figure 1.11):
  • a wave: atrial contraction
  • c wave: bulging of the closed tricuspid valve at the start of isovolumetric right ventricular contraction
  • x descent: atrial relaxation
  • v wave: venous return against a closed tricuspid valve
  • y descent: atrial relaxation while emptying into the right ventricle
zoom view
Figure 1.11: The jugular venous pressure (JVP) waveform.
69Abnormalities of the JVP waveform may reflect underlying pathology:
  • Absent JVP may be present in patients who are intravascularly deplete
  • Elevated JVP (>4 cm above the sternal angle) with normal waveforms may reflect right heart failure and fluid overload
  • Elevated JVP with absent waveforms may be caused by superior vena cava obstruction
  • Cannon a waves are caused by atrial contraction against a closed tricuspid valve such as in complete heart block
  • Absent a waves may be seen in atrial fibrillation
  • Large v waves is a feature of tricuspid regurgitation
  • Elevated JVP with deep x and y descents may be seen with constrictive pericarditis
The electrocardiographic findings in this patient are consistent with complete atrioventricular dissociation, as is seen in complete heart block. Thus, the most likely JVP waveforms will be cannon a waves.
Levick JR. An introduction to cardiovascular physiology, 5th edn. London: Hodder Arnold; 2010.
85. A 45-year-old man presents with a productive cough, shortness of breath and pyrexia. He has saturations of 91% on room air. An arterial blood gas sample (ABG) is performed but the plastic syringe is left for more than 60 minutes at room temperature prior to analysis.
What is the most likely erroneous result on the ABG analysis?
  1. Reduced pH
  2. Elevated Pao2
  3. Reduced Paco2
  4. Increased glucose
  5. Reduced lactate
85. A Reduced pH
Once an arterial blood gas (ABG) sample is collected, it should be analysed as soon as possible. Within the ABG syringe cellular metabolism continues. Dissolved oxygen is utilised; however, this is only of a limited quantity and anaerobic metabolism ensues and carbon dioxide is produced, thereby increasing the Paco2. Anaerobic metabolism persists utilising glucose stores and producing lactate. The overall effect of delayed analysis of an ABG sample can be summarised as follows:
  • Reduced pH
  • Reduced Pao2
  • Increased Paco2
  • Reduced glucose
  • Increased lactate
Other errors in analysis of an ABG sample include:
  • Air bubbles in syringe: may incorrectly increase Pao2, saturation and pH of the sample
  • Clotting of the sample: may incorrectly increase the potassium concentration of the sample
  • Dilution of the sample with arterial line saline flush or liquid heparin: may incorrectly increase Pao2 while decreasing Paco2, glucose, lactate, haemoglobin and potassium
  • Accidental venepuncture during ABG sample collection: may incorrectly decrease Po2 and saturations, while increasing Pco2
Wennecke G, Juel G. Avoiding pre-analytical errors in blood gas testing. Radiometer Medical ApS; 2008.
86. A 46-year-old woman is undergoing an open reduction and internal fixation of a tibial plateau fracture. She is breathing spontaneously through a laryngeal mask airway. Ten minutes following prosthesis insertion, her respiratory rate increases to 34 breaths per minute, oxygen saturations decrease to 88% with an Fio2 of 0.5, end-tidal CO2 reduces to 2.4 kPa and a petechial rash is noted on her chest.
What is the most likely diagnosis?
  1. Haemorrhage
  2. Anaphylaxis
  3. Fat embolism syndrome
  4. Venous thromboembolism
  5. Myocardial ischaemia
86. C Fat embolism syndrome
This patient has clinical features of fat embolism syndrome. This condition occurs when fat from the bone marrow enters the circulation and restricts right ventricular 70outflow by increasing pulmonary artery pressures. It most frequently occurs after major trauma, but bone fixation may also cause it. The classical triad of clinical features includes:
  • Respiratory symptoms: tachypnoea, dyspnoea, pulmonary oedema, hypoxia
  • Neurological features: confusion, drowsiness
  • Petechial rash: usually on chest wall, axilla or conjunctiva
Other clinical features include tachycardia, pyrexia, retinal emboli on fundoscopy, renal dysfunction, thrombocytopaenia, anaemia, increased erythrocyte sedimentation rate and fat macroglobulinaemia. Treatment is predominantly supportive.
Haemorrhage would present with cardiovascular as well as respiratory changes, and a rash is not a common presentation for acute haemorrhage. Although anaphylaxis may have a similar presentation, the rash is not typically petechial but urticarial. In addition, cardiovascular changes usually predominate in anaphylaxis. Venous thromboembolism is possible, yet in the time frame described it is less likely. Finally myocardial ischaemia does not usually present with a rash.
Gupta A, Reilly CS. Fat embolism. Contin Educ Anaesth Crit Care Pain 2007; 7:148–151.
87. A 28-year-old man with a traumatic brain injury is to be transferred to a neurosurgical centre 90 minutes away. He is intubated and being ventilated with an Fio2 of 1.0 and a minute ventilation of 10 L per minute.
What is the minimum number of full size E oxygen cylinders that would be needed for transfer?
  1. One size E oxygen cylinders
  2. Two size E oxygen cylinders
  3. Three size E oxygen cylinders
  4. Four size E oxygen cylinders
  5. Five size E oxygen cylinders
87. B Two size E oxygen cylinders
Calculation of the volume of oxygen cylinders required for a transfer depends on two factors: the oxygen delivered and the duration of the transfer.
Oxygen delivered = Fio2× minute volume
Oxygen delivered = 1.0 ×10 L/min
Oxygen delivered = 10 L/min
A size E oxygen cylinder contains a volume of 680 L of oxygen, thus would last for 68 minutes in total. A journey of 90 minutes would therefore require a minimum of 2 size E oxygen cylinders, as this would provide enough oxygen to last 136 minutes.
Shouman YM. More information from the pressure gauge of oxygen cylinders. Anesth Analg 2004; 99;307–308.
88. A 44-year-old woman is undergoing free flap surgery for breast reconstruction. The surgeons are concerned about microvascular perfusion in the donor tissue. The blood pressure decreases to 85/44 mmHg.
What is the most important variable to consider in order to optimise flap perfusion?
  1. Blood pressure
  2. Heart rate
  3. Blood viscosity
  4. Vessel calibre
  5. Temperature
88. D Vessel calibre
Blood flow to the flap is determined predominantly by the Hagen-Poiseuille equation:
where η is fluid viscosity, l is length of the tube, r is the radius of the tube and ΔP is pressure gradient along the tube.
Although the blood pressure and viscosity play an important role in microvascular perfusion, a change in the calibre or radius of vessels leads to a change in blood flow to the power of 4. Therefore, it is the vessel radius that is the most important variable.
71Temperature may affect a combination of variables including viscosity and vessel radius; however, it is ultimately the radius that is vital.
Although heart rate may affect the cardiac output, it may not necessarily increase flap perfusion to the extent of vessel diameter.
Adams J, Charlton P. Anaesthesia for microvascular free tissue transfer. BJA CEPD Reviews 2003; 3:33–37.
89. A 65-year-old man presents with a 12-hour history of fatigue, confusion and chest pain with a plasma normal troponin concentration. A 12-lead electrocardiogram shows a heart rate of 32 beats per minute with complete dissociation of P waves from QRS complexes.
Which drug is most likely to increase the heart rate in this patient?
  1. Isoprenaline
  2. Atropine
  3. Phenylephrine
  4. Salbutamol
  5. Noradrenaline
89. A Isoprenaline
This patient has symptomatic complete heart block that is due to complete atrioventricular (AV) dissociation. The ideal pharmacological agent for this patient would be one with positive chronotropic effects, mediated via β1-adrenoceptors. Anticholinergic drugs such as atropine would be ineffective because they would act to increase the atrial rate but have no effect on ventricular rate because of the AV dissociation.
Isoprenaline is a potent synthetic β1- and β2-adrenoceptor agonist with no α-adrenoceptor effects. Thus, it has positive inotropic and chronotropic effects, increasing the cardiac output. It was thought to be the ideal drug for patients with complete heart block.
Phenylephrine is a pure α1-adrenoceptor agonist with no β-effects, therefore would not have any effect on heart rate or contractility in this patient.
Salbutamol is predominantly a β2-adrenoceptor agonist; thus, it induces smooth muscle relaxation and vasodilatation. It will have no direct effects to increase myocardial contractility or heart rate.
Noradrenaline predominantly acts via α1-adrenoceptors to causes peripheral vasoconstriction, hypertension and, potentially, a reflex bradycardia, although this reflex may be absent in a patient with complete AV dissociation. Noradrenaline has minimal influence on increasing heart rate in patients with complete heart block, and therefore would not be indicated in this patient.
Sasada M, Smith S. Drugs in anaesthesia and intensive care, 4th edn. Oxford: Oxford University Press; 2011.
90. A 66-year-old man with severe chronic obstructive airways disease presents for a laparoscopic hernia repair as a day case.
Which is the most appropriate anaesthetic agent to use in this patient?
  1. Bupivacaine
  2. Desflurane
  3. Sevoflurane
  4. Remifentanil
  5. Lidocaine
90. C Sevoflurane
Laparoscopic hernia repairs are most frequently done under general anaesthetic, although there a few reports of regional anaesthetic techniques being used but this is not common practice. General anaesthesia is required to ensure optimal ventilation, avoidance of excessive build-up of CO2 and abdominal discomfort despite adequate regional blockade. Therefore, lidocaine and bupivacaine would not be ideal agents in this patient.
Under general anaesthesia, the ideal agent in this patient would be sevoflurane because it provides bronchodilator properties that would be of benefit to patients with chronic obstructive airways disease (COAD). Desflurane is irritative to airways and can precipitate bronchospasm, particularly in patients with COAD.
72Although a remifentanil infusion would provide intraoperative analgesia and blood pressure control, it is not an anaesthetic agent in itself, and therefore cannot be used in isolation.
Molinelli BM, Tagliavia A, Bernstein D. Total extraperitoneal preperitoneal laparoscopic hernia repair using spinal anesthesia. JSLS 2006; 10:341–344.